Maternity HESI Questions

Lakukan tugas rumah & ujian kamu dengan baik sekarang menggunakan Quizwiz!

A client who is in active labor receives a prescription for oxytocin 10 milliunits/min intravenously (IV). The IV bag contains 20 units in lacated Ringers 1000 mL. How many mL/hour should the nurse program the infusion pump to deliver? (Enter numerical value only.)

.

A woman who is trying to get pregnant tells the nurse that she was very disappointed several months ago when she was informed that her positive pregnancy test was a false positive. Which method provides the greatest degree of accuracy? A. Visualization of implantation by vaginal ultrasound. B. Presence of amenorrhea for 2 months. C. Maternal blood serum tests positive for alpha-fetoprotein. D. Complaints of feeling tired all of the time.

...

The healthcare provider prescribes zidovudine 100 mg by mouth five times daily for a pregnant woman who is HIV positive. The drug is available in a 240 mL bottle labeled, "50 mg/5mL." How many mL should the nurse administer? (Enter numeric value only.)

10 mL.

Ampicillin 1 gram intravenously every 4 hours is prescribed for a 38-week multipara who is positive for Group B streptococcus. The drug is available via the Pyxis MedStation medication dispensing unit and is diluted in 50 mL of normal saline. To administer the medication over 30 minutes, the nurse should set the infusion pump to deliver how many mL/hour? (Enter numerical value only.)

100 mL/hr.

A loading dose of terbutaline (Brethine) 250 mcg IV is prescribed for a client in preterm labor. Brethine 20 mg is added to 1,000 mL D5W. How many mL of the solution should the nurse administer? (Enter numeric value only)

12.5 mL

In evaluating the respiratory effort of a one-hour-old infant using the Silverman-Anderson Index, the nurse determines the infant has synchronized chest and abdominal movement, just visible lower chest retractions, just visible xiphoid retractions, minimal and transient nasal flaring, and an expiratory grunt heard only on auscultation. What Silverman-Anderson score should the nurse assign to this infant? (Enter numeral value only.)

4 A Silverman-Anderson Index has five categories with scores of 0, 1, or 2. The total score ranges from 0 to 10. Four of the these assessment findings should receive a score of 1, and the 5th finding (synchronized chest and abdominal movement) receives a score of 0. Therefore, the total score is 4. A total score of 0 means the infant has no dyspnea, a total score of 10 indicates maximum respiratory distress.

Bottlefeeding, when should she expect period to return?...

6-8 weeks after delivery.

A 34-week primigravida woman with preeclampsia is receiving Lactated Ringer's 500 mL with magnesium sulfate 20 grams at the rate of 3 grams/hr. How many mL/hr should the nurse program the infusion pump? (Enter numeric value only.)

75 mL/hr.

The nurse is monitoring a client who is in the active stage of labor. The nurse documents that the client is experiencing labor dystocia. The nurse determines that which risk factors in the client's history placed her at risk for this complication? Select all that apply. A. Age 54 years. B. Body mass index of 28. C. Previous difficulty with fertility. D. Administration of oxytocin for induction. E. Potassium level of 3.6 mEq/L (3.6 mmol/L).

A, B, C Risk factors that increase a woman's risk for dysfunctional labor include the following: advanced maternal age, being overweight, electrolyte imbalances, previous difficulty with fertility, uterine overstimulation with oxytocin, short stature, prior version, masculine characteristics, uterine abnormalities, malpresentations and position of the fetus, cephalopelvic disproportion, maternal fatigue, dehydration, fear, administration of an analgesic early in labor, and use of epidural analgesia. Age 54 years is considered advanced maternal age, and a body mass index of 28 is considered overweight. Previous difficulty with fertility is another risk factor for labor dystocia. A potassium level of 3.6 mEq/L (3.6 mmol/L) is normal, and administration of oxytocin alone is not a risk factor; risk exists only if uterine hyperstimulation occurs. (Labor dystocia, also known as difficult labor, is a prolonged or abnormal labor that fails to progress normally, resulting in a prolonged or difficult delivery.)

Which findings of depression in the postpartum client require additional action by the nurse? (Select all that apply.) A. Feelings of sadness. B. Return of lochia rubra. C. Decreased appetite. D. Engorged, painful breasts. E. Trouble falling asleep.

A, C, E - HESI

Which finding(s) is (are) of most concern to the nurse when caring for a woman in the first trimester of pregnancy? (Select all that apply.) A. Cramping with bright red spotting. B. Extreme tenderness of the breast. C. Lack of tenderness of the breast. D. Increased amounts of discharge. E. Increased right-side flank pain.

A, C, E A. Cramping with bright red spotting. C. Lack of tenderness of the breast. E. Increased right-side flank pain. (A and C) are signs of a possible miscarriage. Cramping with bright red bleeding is a sign that the client's menstrual cycle is about to begin. A decrease of tenderness in the breast is a sign that hormone levels have declined and that a miscarriage is imminent. (E) could be a sign of an ectopic pregnancy, which could be fatal if not discovered in time before rupture. (B and D) are normal signs during the first trimester of a pregnancy.

The client comes to the hospital assuming she is in labor. Which assessment finding(s) by the nurse would indicate that the client is in true labor? (Select all that apply.) A. Pain in the lower back that radiates to abdomen. B. Contractions decreased in frequency with ambulation. C. Progressive cervical dilation and effacement. D. Discomfort localized in the abdomen. E. Regular and rhythmic painful contractions.

A, C, E A. Pain in the lower back that radiates to abdomen. C. Progressive cervical dilation and effacement. E. Regular and rhythmic painful contractions. These are all signs of true labor (A, C, and E). The others are signs of false labor (B and D).

During labor, the nurse determines that a full-term client is demonstrating late decelerations. In which sequence should the nurse implement these nursing actions? (Arrange in order.) A. Reposition the client. B. Call the healthcare provider. C. Increase IV fluid. D. Provide oxygen via face mask

A, D, C, B 1 - A) Reposition the client. 2 - D) Provide oxygen via face mask. 3 - C) Increase IV fluid. 4 - B) Call the healthcare provider. To stabilize the fetus, intrauterine resuscitation is the first priority, and to enhance fetal blood supply, the laboring client should be repositioned (1) to displace the gravid uterus and improve fetal perfusion. Secondly, to optimize oxygenation of the circulatory blood volume, oxygen via face mask (2) should be applied to the mother. Next, the IV fluids should be increased (3) to expand the maternal circulating blood volume. Then, the primary healthcare provider should be notified (4) for additional interventions to resolve the fetal stress.

A healthcare provider informs the charge nurse of a labor and delivery unit that a client is coming to the unit with suspected abruptio placentae. What findings should the charge nurse expect the client to demonstrate? (Select all that apply.) A. Dark, red vaginal bleeding. B. Lower back pain. C. Premature rupture of membranes. D. Increased uterine irritability. E. Bilateral pitting edema. F. A rigid abdomen.

A, D, F. The symptoms of abruptio placentae include dark red vaginal bleeding (A), increased uterine irritability (D), and a rigid abdomen (F). (B, C, and E) are findings not associated with abruptio placentae.

Which finding(s) is (are) most critical for the nurse to report to the primary health care provider when caring for the client during the last trimester of her pregnancy? (Select all that apply.) A. Increased heartburn that is not relieved with doses of antacids. B. Increase of the fetal heart rate from 126 to 156 beats/min from the last visit. C. Shoes and rings that are too tight because of peripheral edema in extremities. D. Decrease in ability for the client to sleep for more than 2 hours at a time. E. Chronic headache that has been lingering for a week behind the client's eyes.

A, E A. Increased heartburn that is not relieved with doses of antacids. E. Chronic headache that has been lingering for a week behind the client's eyes. (A and E) are possible signs of preeclampsia or eclampsia but can also be normal signs of pregnancy. These signs should be reported to the health care provider for further evaluation for the safety of the client and the fetus. (B, C, and D) are all normal signs during the last trimester of pregnancy.

Which statement made by the client indicates that the mother understands the limitations of breastfeeding her newborn? A. "Breastfeeding my infant consistently every 3 to 4 hours stops ovulation and my period." B. "Breastfeeding my baby immediately after drinking alcohol is safer than waiting for the alcohol to clear my breast milk." C. "I can start smoking cigarettes while breastfeeding because it will not affect my breast milk." D. "When I take a warm shower after I breastfeed, it relieves the pain from being engorged between breastfeedings."

A. "Breastfeeding my infant consistently every 3 to 4 hours stops ovulation and my period. Continuous breastfeeding on a 3- to 4-hour schedule during the day will cause a release of prolactin, which will suppress ovulation and menses, but is not completely effective as a birth control method (A). (B) is incorrect because alcohol can immediately enter the breast milk. Nicotine is transferred to the infant in breast milk (C). Taking a warm shower will stimulate the production of milk, which will be more painful after breastfeedings (D).

A nurse is talking with a client during her initial prenatal visit. The client reports a history of trisomy 13 syndrome in her family and is concerned her fetus might be at risk. Which of the following statements should the nurse provide? A. "If you sign an informed consent form, we can perform genetic screening to see if your baby has this disorder." B. "If the genetic screening shows that your baby has this disorder, I can provide you with information about an abortion clinic." C. "Screening for trisomy 14 syndrome and other chromosomal disorders is done automatically for clients at increased risk." D. "I can provide you with information about sterilization so that the disorder is not passed to your future children."

A. "If you sign an informed consent form, we can perform genetic screening to see if your baby has this disorder." Genetic screening has multiple legal and ethical considerations that must be addressed prior to testing. The client will need to sign an informed consent from prior to the screening.

A nurse is providing discharge instructions to a client who is breastfeeding her newborn. Which of the following statements should the nurse include? A. "Notify your provider if you notice cracking on your nipples." B. "Notify your provider if you have had a bowel movement within 5 days." C. "Notify you provider if your breasts leak when you shower." D. "Notify your provider if your vaginal discharge is brownish-red color."

A. "Notify your provider if you notice cracking on your nipples." The client should notify the provider of cracking, bleeding, or blistered nipples since this increases the client's risk of infection.

A new mother asks the nurse about an area of swelling on her baby's head near the posterior fontanel that lies across the suture line. How should the nurse respond? A. "That is called caput succedaneum. It will absorb and cause no problems." B. "That is called a cephalhematoma. It can cause jaundice as it is absorbed." C. "That is called a cephalhematoma. It will cause no problems." D. "That is called caput succedaneum. It will have to be drained."

A. "That is called caput succedaneum. It will absorb and cause no problems." - HESI An area of swelling on her baby's head near the posterior fontanel that crosses a suture line is caput succedaneum. Caput succedaneum is a common and usually benign condition that occurs due to pressure on the baby's head during delivery. Caput succedaneum typically resolves on its own within a few days without causing any long-term complications.

A nurse is providing discharge teaching to the parent of a newborn. Which of the following statements should the nurse include in the teaching? A. "your baby should be rear-facing in a car seat until 2 years of age." B. "Cover your baby with a light blanket during naps." C. "Set your hot water heater to no more than 140 degrees Fahrenheit." D. Ensure your baby's crib has side rails that can be lowered."

A. "your baby should be rear-facing in a car seat until 2 years of age." The parent should ensure the baby rides in a rear-facing car seat until at least 2 years of age, or longer if recommended by the car seat manufacturer.

The nurse is counseling a couple who has sought information about conceiving. For teaching purposes, the nurse should know that ovulation usually occurs A. 2 weeks before menstruation. B. immediately after menstruation. C. immediately before menstruation. D. 3 weeks before menstruation.

A. 2 weeks before menstruation. Ovulation occurs 14 days before the first day of the menstrual period (A). While ovulation can occur in the middle of the cycle, or 2 weeks after menstruation, this is only true for a woman who has a perfect 28-day cycle. For many women, the length of their menstrual cycle varies.

A woman who thinks she could be pregnant calls her neighbor, a nurse, to ask when she could use a home pregnancy test to diagnose pregnancy. Which response is best? A. A home pregnancy test can be used right after your first missed period. B. These tests are most accurate after you have missed your second period. C. Home pregnancy tests often give false positives and should not be trusted. D. The test can provide accurate information when used right after ovulation.

A. A home pregnancy test can be used right after your first missed period. Home urine tests are based on the chemical detection of human chorionic gonadotrophin, which begins to increase 6 to 8 days after conception, and is best detected at 2 weeks gestation or immediately after the first missed period (A). (B and D) provide inaccurate information. Although home tests are accurate, they have more false negatives than false positives (C), usually because they are used too early.

A 38-week primigravida is admitted to labor and delivery after a non-reactive result on a non-stress test (NST). The nurse begins a contraction stress test (CST) with an oxytocin infusion. Which finding is most important for the nurse to report to the health care provider? A. A pattern of fetal late decelerations. B. Fetal heart rate accelerations with fetal movement. C. Absence of uterine contractions within 20 minutes. D. Spontaneous rupture of membranes.

A. A pattern of fetal late decelerations. Since the nurse has already started an oxytocin infusion to initiate uterine contractions for the CST, the absence of uterine contractions within 20 minutes after the oxytocin infusion would be expected and already reported to the provider.

A nurse is caring for a newborn who is premature at 30 weeks of gestation. Which of the of following findings should the nurse expect? A. Abundant lanugo. B. Good flexion. C. Heel creases covering the bottom of the feet. D. Dry, parchment-like skin.

A. Abundant lanugo Newborns who are premature have abundant lanugo (fine hair), especially over their back. A full-term newborn typically has minimal lanugo present only on the shoulders, pinna, and forehead. (It helps protect them and keeps them warm while they grow in utero.)

A nurse in a provider's office is caring for a client who is in the first trimester of pregnancy. Which of the following psychological tasks should the nurse expect the client to accomplish during this trimester? A. Accepting the pregnancy. B. Preparing for the end of pregnancy. C. Preparing for parenthood. D. Accepting the baby.

A. Accepting the pregnancy. Accepting the pregnancy is a psychological tasks that the client is expected to accomplish during the first trimester.

The nurse in the labor room is caring for a client in the active stage of the first phase of labor. The nurse is assessing the fetal patterns and notes a late deceleration on the monitor strip. What is the most appropriate nursing action? A. Administer oxygen via face mask. B. Place the mother in a supine position. C. Increase the rate of the oxytocin (Pitocin) intravenous infusion. D. Document the findings and continue to monitor the fetal patterns.

A. Administer oxygen via face mask. Late decelerations are due to uteroplacental insufficiency and occur because of decreased blood flow and oxygen to the fetus during the uterine contractions. Hypoxemia results; Oxygen at 8 to 10 L/minute via face mask is necessary. The supine position is avoided because it decreases uterine blood flow to the fetus. The client should be turned onto her side to displace pressure of the gravid uterus on the inferior vena cava. An intravenous oxytocin infusion is discontinued when a late deceleration is noted. The oxytocin would cause further hypoxemia because of increased uteroplacental insufficiency resulting from stimulation of contractions by this medication. Although the nurse would document the occurrence, D would delay necessary treatment. - Saunders Comprehensive Review for the NCLEX-RN® Examination 7th Edition

The nurse is performing a gestational age assessment on a full-term newborn during the first hour of transition using the Ballard (Dubowitz) scale. Based on this assessment, the nurse determines that the neonate has a maturity rating of 40- weeks. What findings should the nurse identify to determine if the neonate is small for gestational age (SGA)? (Select all that apply.) A. Admission weight of 4 pounds, 15 ounces (2244 grams). B. Head to heel length of 17 inches (42.5 cm). C. Frontal occipital circumference of 12.5 inches (31.25 cm). D. Skin smooth with visible veins and abundant vernix. E. Anterior plantar crease and smooth heel surfaces. F. Full flexion of all extremities in resting supine position.

A. Admission weight of 4 pounds, 15 ounces (2244 grams). B. Head to heel length of 17 inches (42.5 cm). C. Frontal occipital circumference of 12.5 inches (31.25 cm). Correct choices are (A, B, and C). The normal full-term, appropriate for gestational age (AGA) newborn should fall between the measurement ranges of weight, 6-9 pounds (2700-4000 grams); length, 19-21 inches (48-53 cm); FOC, 13-14 inches (33-35 cm). This neonate's parameters (A, B, and C) plot below the 10% percentile, which indicate that the infant is SGA. (D and E) are criteria for a pre-term neonate. (F) is a criteria for physical maturity score (full-term, 40-weeks) on the Ballard (Dubowitz) scale.

A woman who gave birth 48 hours ago is bottle-feeding her infant. During assessment, the nurse determines that both breasts are swollen, warm, and tender upon palpation. What action should the nurse take? A. Apply cold compresses to both breasts for comfort. B. Instruct the client run warm water on her breasts. C. Wear a loose-fitting bra to prevent nipple irritation. D. Express small amounts of milk to relieve pressure.

A. Apply cold compresses to both breasts for comfort. The client is experiencing engorgement even though she is bottle-feeding her infant, and cold compresses (A) may help reduce discomfort. Lactation begins about the third day after delivery, so the mother should avoid any breast stimulation, such as (B or D), which further stimulates milk production. To aid in suppressing lactation, a well-fitting bra, not (C), should be worn to support and bind the breasts.

A multigravida full term, laboring client complains of back labor. Vaginal examine reveals that the client is 3cm with 50% effacement, and the fetal head is at -1 station. What action should the nurse implement first? A. Apply counter - pressure to the sacral area. B. Turn the client lateral position. C. Notify the scrub nurse to prepare the OR. D. Ambulate the client between contractions.

A. Apply counter - pressure to the sacral area. The nurse should first apply counter-pressure to the sacral area to relieve the pressure on the sacral nerve roots and help alleviate the back pain associated with back labor.

A nurse is caring for a client who reports intestinal gas pain following cesarean section. Which of the following actions should the nurse take? A. Assist the client to ambulate in the hallway. B. Instruct the client to splint the incision with a pillow. C. Have the client drink fluids through a straw. D. Encourage the client to drink carbonated beverages.

A. Assist the client to ambulate in the hallway. Walking can help stimulate peristalsis, which will promote the expulsion of gas. Splinting the incision with a pillow will assist with incisional pain relief but will not assist with relieving discomfort from gas pains. Drinking carbonated beverages and through a straw and can cause the client to ingest air and increase gas production.

The nurse is teaching a new mother about diet and breastfeeding. Which instruction is most important to include in the teaching plan? A. Avoid alcohol because it is excreted in breast milk. B. Eat a high-roughage diet to help prevent constipation. C. Increase caloric intake by approximately 500 cal/day. D. Increase fluid intake to at least 3 quarts each day.

A. Avoid alcohol because it is excreted in breast milk. Alcohol should be avoided while breastfeeding because it is excreted in breast milk (A) and may cause a variety of problems, including slower growth and cognitive impairment for the infant. (B, C, and D) should also be included in diet teaching for a breastfeeding mother; however, because these do not involve safety of the infant, they do not have the same degree of importance as (A).

The nurse is interacting with a female client who is diagnosed with postpartum depression. Which finding should the nurse document as an objective signs of depression? (Select all that apply.) A. Avoids eye contact. B. Interacts with a flat affect. C. Reports feeling sad. D. Expresses suicidal thoughts. E. Has a disheveled appearance.

A. Avoids eye contact. B. Interacts with a flat affect. C. Reports feeling sad. D. Expresses suicidal thoughts.

A vaginally delivered infant of an HIV positive mother is admitted to the newborn nursery. What intervention should the nurse perform first? A. Bathe the infant with an antimicrobial soap. B. Measure the head and chest circumference. C. Obtain the infant's footprints. D. Administer vitamin K (AquaMEPHYTON).

A. Bathe the infant with an antimicrobial soap. To reduce direct contact with the human immuno-virus in blood and body fluids on the newborn's skin, a bath (A) with an antimicrobial soap should be administered first. (B, C, and D) should be implemented after the neonate's skin is cleansed of blood and body fluids.

The nurse is planning care for a client at 30-weeks gestation who is experiencing preterm labor. What maternal prescription is most important in preventing this fetus from developing respiratory distress syndrome? A. Betamethasone (Celestone) 12mg deep IM. B. Butorphanol 1mg IV push q2h PRN pain. C. Ampicillin 1g IV push q8h. D. Terbutaline (Brethine) 0.25mg subcutaneously q15 minutes x3.

A. Betamethasone. The nurse should plan to administer betamethasone IM, a glucocorticoid, to stimulate fetal lung maturity and prevent respiratory depression. Butorphanol is an opioid analgesic used for pain relief during labor. Ampicillin is an antibiotic used to treat or prevent infections. Terbutaline is a beta-agonist that can be used to suppress uterine contractions.

The nurse is teaching a woman how to use her basal body temperature (BBT) pattern as a tool to assist her in conceiving a child. Which temperature pattern indicates the occurrence of ovulation, and therefore, the best time for intercourse to ensure conception? A. Between the time the temperature falls and rises. B. Between 36 and 48 hours after the temperature rises. C. When the temperature falls and remains low for 36 hours. D. Within 72 hours before the temperature falls.

A. Between the time the temperature falls and rises. - HESI In most women, the BBT drops slightly 24 to 36 hours before ovulation and rises 24 to 72 hours after ovulation, when the corpus luteum of the ruptured ovary produces progesterone. (Your body temperature falls slightly just before your ovary releases an egg and rises 24 hours after the egg's release.)

A multigravida client at 41-weeks gestation presents in the labor and delivery unit after a non-stress test indicated that the fetus is experiencing some difficulties in utero. Which diagnostic test should the nurse prepare the client for additional information about fetal status? A. Biophysical profile (BPP). B. Ultrasound for fetal anomalies. C. Maternal serum alpha-fetoprotein (AF) screening. D. Percutaneous umbilical blood sampling (PUBS).

A. Biophysical profile (BPP). BPP (A) provides data regarding fetal risk surveillance by examining 5 areas: fetal breathing movements, fetal movements, amniotic fluid volume, and fetal tone and heart rate. The client's gestation has progressed past the estimated date of confinement, so the major concern is fetal well-being related to an aging placenta, not screening for fetal anomalies (B). Maternal serum AF screening is generally checked between 15 and 22 weeks to detect neural tube defects (C). Although PUBS is performed to determine a number of at-risk fetal conditions, the BPP determines current fetal risk (D).

A breastfeeding postpartum client is diagnosed with mastitis, and antibiotic therapy is prescribed. Which instruction should the nurse provide to this client? A. Breastfeed the infant, ensuring that both breasts are completely emptied. B. Feed expressed breast milk to avoid the pain of the infant latching onto the infected breast. C. Breastfeed on the unaffected breast only until the mastitis subsides. D. Dilute expressed breast milk with sterile water to reduce the antibiotic effect on the infant.

A. Breastfeed the infant, ensuring that both breasts are completely emptied. Breastfeeding should be encouraged, as it helps to ensure that the breast is completely emptied and can help to promote drainage of the infected area. Avoiding breastfeeding or feeding expressed milk exclusively can lead to a decrease in milk supply and make the condition worse. Diluting breast milk with sterile water is not recommended as it can increase the risk of infection for the infant.

Twenty-four hours after admission to the newborn nursery, a full-term male infant develops localized swelling on the right side of his head. The nurse knows that, in the newborn, an accumulation of blood between the periosteum and skull which does not cross the suture line is a newborn variation known as: A. Cephalhematoma, which is caused by forceps trauma and may last up to 8 weeks. B. Subarachnoid hematoma, which requires immediate drainage to prevent further complications. C. Molding, which is caused by pressure during labor and will disappear within 2-3 days. D. Subdural hematoma, which can result in lifelong damage.

A. Cephalhematoma, which is caused by forceps trauma and may last up to 8 weeks. A cephalhematoma is a slight abnormal variation of the newborn which usually arises within the first 24 hours after delivery and is more likely to occur by trauma or if birth assisting tools were utilized during the delivery. The trauma causes capillary bleeding between the periosteum and the skull.

While caring for a laboring client on continuous fetal monitoring, the nurse notes a fetal heart rate pattern that falls and rises abruptly with a "V" shaped appearance. What action should the nurse take first? A. Change the maternal position. B. Administer oxygen at 10/L by mask. C. Prepare for a potential cesarean. D. Allow the client to begin pushing.

A. Change the maternal position. The appropriate action for the nurse to take first when noting a fetal heart rate pattern that falls and rises abruptly with a "V" shaped appearance is to change the maternal position. Fetal heart rate decelerations can be caused by compression of the umbilical cord, and changing the maternal position can alleviate that compression. Administering oxygen and preparing for a potential cesarean delivery may be necessary depending on the severity and persistence of the decelerations, but changing the maternal position should be the initial intervention.

One day after vaginal delivery of a full-term baby, a postpartum client's white blood cell (WBC) count is 15,000/mm³. What action should the nurse take first? A. Check the differential, since the WBC is normal for this client. B. Assess the client's temperature, pulse, and respirations q4h. C. Notify the healthcare provider (HCP), since this finding is indicative of infection. D. Assess the client's perineal area for signs of a perineal hematoma.

A. Check the differential, since the WBC is normal for this client.

A full-term infant is admitted to the newborn nursery and, after careful assessment, the nurse suspects that the infant may have an esophageal atresia. Which symptoms is this newborn likely to have exhibited? A. Choking, coughing, and cyanosis. B. Projectile vomiting and cyanosis. C. Apneic spells and grunting. D. Scaphoid abdomen and anorexia.

A. Choking, coughing, and cyanosis. (A) includes the "3 Cs" of esophageal atresia caused by the overflow of secretions into the trachea. (B) is characteristic of pyloric stenosis in the infant. (C) could be due to prematurity or sepsis, and grunting is a sign of respiratory distress. (D) is characteristic of diaphragmatic hernia. (Esophageal atresia is a birth defect in which part of a baby's esophagus (the tube that connects the mouth to the stomach) does not develop properly thus secretions make their way into the trachea causing choking, coughing, and cyanosis.)

A client at 28-weeks gestation calls the antepartal clinic and states that she is experiencing a small amount of vaginal bleeding which she describes as bright red. She further states that she is not experiencing any uterine contractions or abdominal pain. What instruction should the nurse provide? A. Come to the clinic today for an ultrasound. B. Go immediately to the emergency room. C. Lie on your left side for about one hour and see if the bleeding stops. D. Bring a urine specimen to the lab tomorrow to determine if you have a urinary tract infection.

A. Come to the clinic today for an ultrasound. Third trimester painless bleeding is characteristic of a placenta previa. Bright red bleeding may be intermittent, occur in gushes, or be continuous. Rarely is the first incidence life-threatening, nor cause for hypovolemic shock. Diagnosis is confirmed by transabdominal ultrasound (A). Bleeding that has a sudden onset and is accompanied by intense uterine pain indicates abruptio placenta, which IS life-threatening to the mother and fetus--then (B) would be appropriate. (C) does not take the symptoms seriously. The woman is not describing symptoms of a UTI (D).

A multigravida client arrives at the labor and delivery unit and tells the nurse that her bag of water has broken. The nurse identifies the presence of meconium fluid on the perineum and determines the fetal heart rate is between 140 to 150 beats/minute. What action should the nurse implement next? A. Complete a sterile vaginal exam. B. Take maternal temperature every 2 hours. C. Prepare for an immediate cesarean birth. D. Obtain sterile suction equipment.

A. Complete a sterile vaginal exam. A vaginal exam should be performed after the rupture of membranes to determine the presence of a prolapsed cord. (B and D) can be implemented after the completion of (A). (C) is not indicated at this time since the fetal heart rate is within normal limits. (FHR is normal - elevation indicates fever.)

A pregnant client with mitral stenosis Class III is prescribed complete bedrest. The client asks the nurse, "Why must I stay in bed all the time?" Which response is best for the nurse to provide this client? A. Complete bedrest decreases oxygen needs and demands on the heart muscle tissue. B. We want your baby to be healthy, and this is the only way we can make sure that will happen. C. I know you're upset. Would you like to talk about some things you could do while in bed? D. Labor is difficult and you need to use this time to rest before you have to assume all child-caring duties.

A. Complete bedrest decreases oxygen needs and demands on the heart muscle tissue. To help preserve cardiac reserves, the woman may need to restrict her activities and complete bedrest is often prescribed (A). (B) offers false reassurance. (C) does not answer the woman's question. While (D) may be true, it is not the most important reason for bedrest.

The health care provider prescribes 10 units per liters of oxytocin via IV drip to augment a client's labor because she's experiencing a prolonged active phase. Which finding would cause the nurse to immediately discontinue the oxytocin? A. Contraction duration of 100 seconds. B. For contractions in 10 minutes. C. Uterus is soft. D. Early deceleration of fetal heart rate.

A. Contraction duration of 100 seconds. Contractions that occur more frequently than every 2 minutes or last longer than 90 seconds, or if there are more than 5 contractions in 10 minutes can cause decreased blood flow to the placenta, which can lead to fetal distress and compromise. Early decelerations of the fetal heart rate may be a sign of fetal head compression, but it is not an immediate indication for discontinuing the oxytocin infusion unless other signs of fetal distress are present.

A woman with Type 2 diabetes mellitus becomes pregnant, and her oral hypoglycemic agents are discontinued. Which intervention is most important for the nurse to implement? A. Describe diet changes that can improve the management of her diabetes. B. Inform the client that oral hypoglycemic agents are teratogenic during pregnancy. C. Demonstrate self-administration of insulin. D. Evaluate the client's ability to do glucose monitoring.

A. Describe diet changes that can improve the management of her diabetes. Diet modifications are effective in managing Type 2 diabetes during pregnancy and describing the necessary diet changes is the most important intervention for the nurse to implement with this client. (B, C, and D) are interventions that should also be implemented, but do not have the priority of (A).

At 5-weeks gestation, the rubella titer of a client indicates she is non-immune. When is the best time to administer a rubella vaccine to this client? A. Early postpartum, within 72 hours of delivery. B. After the client stops breastfeeding. C. After the client reaches 20-week gestation. D. Immediately, at 6-weeks gestation, to protect this fetus.

A. Early postpartum, within 72 hours of delivery. The rubella vaccine is a live vaccine that is contraindicated during pregnancy due to the potential risk to the fetus. The best time to administer a rubella vaccine to a pregnant client who is non-immune is after she gives birth and before she leaves the hospital or at her postpartum checkup. Rubella is a highly contagious viral infection that can lead to serious complications if contracted during pregnancy so it is important for the client to receive the vaccine to gain immunity. The client should avoid pregnancy for at least 28 days after vaccination.

An ambulatory client at 39-weeks gestation presents to the emergency center with an obvious injury to her arm that occurred as the result of a fall. Which concurrent symptom is a priority for the nurse? A. Ecchymotic knees. B. Dribbling urine. C. 1+ pedal edema. D. Pain in the forearm.

A. Ecchymotic knees. Ecchymotic knees could indicate a possible pelvic fracture or other significant injury that could impact the client's ability to safely deliver her baby.

The nurse is caring for a woman with a previously diagnosed heart disease who is in the second stage of labor. Which assessment findings are of greatest concern? A. Edema, basilar rales, and an irregular pulse. B. Increased urinary output and tachycardia. C. Shortness of breath, bradycardia, and hypertension. D. Regular heart rate and hypertension.

A. Edema, basilar rales, and an irregular pulse. Edema, basilar rales, and an irregular pulse indicate cardiac decompensation and require immediate intervention. Though (B, C, and D) are cardiac symptoms, they require less emergency intervention than (A).

A full term infant is admitted to the newborn nursery 2 hours after delivery. The delivery record reports that the mother is positive for HIV and received AZT intravenously during labor. What action should the nurse implement first? A. Ensure that AZT is given within 6 hours after birth. B. Collect venous specimen for serum glucose level. C. Asses for the presence of the Moro reflex. D. Obtain consent for the Hep B vaccine

A. Ensure that AZT is given within 6 hours after birth. AZT (zidovudine) is an antiretroviral medication used to prevent mother-to-child transmission of HIV. It is typically given to the mother during labor and to the infant within 6 hours of birth, with additional doses given for the first 6 weeks of life. This helps to reduce the risk of vertical transmission of the virus from mother to infant.

Calculated by Naegele's rule, a primigravida client is at 28 weeks gestation. She is moderately obese and carrying twins and the nurse measures her fundal height at 27 cm. During the previous visit 3 weeks ago, the fundal height measured at 28 cm. Based on these findings, what should the nurse conclude? A. Fundal height measurement may indicate intrauterine growth retardation. B. The healthcare provider needs to be notified immediately since this fundal height measurement is greater than expected. C. Confirm the fundal height measurement with another nurse. D. Recognize this as a reasonable fundal height measurement for this client.

A. Fundal height measurement may indicate intrauterine growth retardation. The fundal height measurement is usually within 2 cm of the gestational age in centimeters. In this case, the fundal height has decreased by 1 cm in 3 weeks, indicating a slowing or cessation of fetal growth. This is a sign of intrauterine growth retardation and should be further investigated by the healthcare provider.

A nurse is reviewing risk factors for postpartum depression with a newly licensed nurse. Which of the following risk factors should the nurse include? A. Gestational diabetes. B. Planned pregnancy. C. Being married. D. Post-term birth.

A. Gestational diabetes. Gestational diabetes increases the risk postpartum depression. Other risk factors include infertility treatment, pregnancy complications, preterm birth, and a history of mood disorder.

The healthcare provider prescribes terbutaline (Brethine) for a client in preterm labor. Before initiating this prescription, it is most important for the nurse to assess the client for which condition? A. Gestational diabetes. B. Elevated blood pressure. C. Urinary tract infection. D. Swelling in lower extremities.

A. Gestational diabetes. The nurse should evaluate the client for gestational diabetes (A) because terbutaline (Brethine) increases blood glucose levels. (B) could be related to the client being in preterm labor, however, terbutaline (Brethine) can cause a decrease in blood pressure. (C) can cause uterine irritability, which can result in preterm labor that should be treated by first resolving the infection rather than by administering a tocolytic agent such as terbutaline (Brethine). (D) is a common pregnancy complaint. (Terbutaline is a medication used to delay preterm labor by which helping prevent and slow contractions of the uterus.)

The nurse attempts to help an unmarried teenager deal with her feelings following a spontaneous abortion at 8-weeks gestation. What type of emotional response should the nurse anticipate? A. Grief related to her perceptions about the loss of this child. B. Relief of ambivalent feelings experienced with this pregnancy. C. Shock because she may not have realized that she was pregnant. D. Guilt because she had not followed her healthcare provider's instructions.

A. Grief related to her perceptions about the loss of this child. Grief/loss response occurs at all stages of pregnancy loss. Ambivalence toward the pregnancy normally occurs up to 20-weeks and contributes to guilt experienced following pregnancy loss (B). Shock due to denial of pregnancy might be a factor with this client, but it is not likely to influence the grieving process (C). Although data was not provided to support (D), compliance with medical instructions does not prevent guilt that can be associated with other behaviors the client may have exhibited (such as smoking) during the first trimester.

Which action should the nurse implement when preparing to measure the fundal height of a pregnant client? A. Have the client empty her bladder. B. Request the client lie on her left side. C. Perform Leopold's maneuvers first. D. Give the client some cold juice to drink.

A. Have the client empty her bladder. To accurately measure the fundal height, the bladder must be empty to avoid elevation of the uterus. Fundal height is not measured with the client lying on her side (B). Leopold's maneuvers are performed to assess fetal position and the expected location of the point of maximal impulse (PMI) for fetal heart rate (C). Cold juice (D) does not affect the fundal height measurement, but may be given to arouse the fetus if the fetus appears to be sleeping during a non-stress test.

A nurse is caring for a client who is scheduled to receive a spinal anesthetic. Which of the following actions should the nurse plan to perform? A. Infuse a 500 mL bolus of 0.9% sodium chloride immediately prior to the procedure. B. Assess the fetal hear rate pattern for 10 min prior to the procedure. C. Position the client upright and erect on the edge of the bed prior to the procedure. D. Monitor vital signs every 15 min after the anesthetic is placed.

A. Infuse a 500 mL bolus of 0.9% sodium chloride immediately prior to the procedure. The nurse should infuse a fluid bolus of 500 to 1000 mL of 0.9% sodium chloride or located Ringer's 15-30 minutes before the procedure to offset the potential complication of hypotension.

Immediately after birth a newborn infant is suctioned, dried, and placed under a radiant warmer. The infant has spontaneous respirations and the nurse assesses an apical heart rate of 80 beats/minute and respirations of 20 breaths/minute. What action should the nurse perform next? A. Initiate positive pressure ventilation. B. Intervene after the one minute Apgar is assessed. C. Initiate CPR on the infant. D. Assess the infant's blood glucose level.

A. Initiate positive pressure ventilation. The nurse should immediately begin positive pressure ventilation (A) because this infant's vital signs are not within the normal range, and oxygen deprivation leads to cardiac depression in infants. (The normal newborn pulse is 100 to 160 beats/minute and respirations are 40 to 60 breaths/minute.) Waiting until the infant is 1 minute old to intervene may worsen the infant's condition. According to neonatal resuscitation guidelines, CPR is not begun until the heart rate is 60 or below or between 60 and 80 and not increasing after 20 to 30 seconds of PPV. (D) can be checked after treating the respiratory rate.

Immediately after birth a newborn infant is suctioned, dried, and placed under a radiant warmer. The infant has spontaneous respirations and the nurse assesses an apical heart rate of 80 beats/minute and respirations of 20 breaths/ minute. What action should the nurse perform next? A. Initiate positive pressure ventilation. B. Intervene after the one minute Apgar is assessed. C. Initiate CPR on the infant. D. Assess the infant's blood glucose level.

A. Initiate positive pressure ventilation. The nurse should immediately begin positive pressure ventilation because this infant's vital signs are not within the normal range, and oxygen deprivation leads to cardiac depression in infants. (The normal newborn pulse is 100 to 160 beats/minute and respirations are 40 to 60 breaths/minute.) Waiting until the infant is 1 minute old to intervene may worsen the infant's condition. According to neonatal resuscitation guidelines, CPR is not begun until the heart rate is 60 or below or between 60 and 80 and not increasing after 20 to 30 seconds of PPV.

A one-day-old neonate develops a cephalahematoma. The nurse should closely assess this neonate for which common complication? A. Jaundice. B. Poor appetite. C. Brain damage. D. Hypoglycemia

A. Jaundice. A cephalohematoma is a collection of blood between the skull bone and its periosteum (the covering of the bone). Since the blood cannot be reabsorbed immediately, the hematoma gradually increases in size, typically over the first few days of life. The breakdown of the blood leads to the release of bilirubin, which can result in jaundice.

A 16-year-old gravida 1, para 0 client has just been admitted to the hospital with a diagnosis of eclampsia. She is not presently convulsing. Which intervention should the nurse plan to include in this client's nursing care plan? A. Keep airway equipment at the bedside/Keep an airway at the bedside. B. Allow liberal family visitation. C. Monitor blood pressure, pulse, and respirations every 4 hours. D. Assess temperature every hour.

A. Keep airway equipment at the bedside/Keep an airway at the bedside.

Which nursing intervention is most helpful in relieving postpartum uterine contractions or "afterpains?" A. Lying prone with a pillow on the abdomen. B. Using a breast pump. C. Massaging the abdomen. D. Giving oxytocic medications.

A. Lying prone with a pillow on the abdomen. Lying prone (A) keeps the fundus contracted and is especially useful with multiparas, who commonly experience afterpains due to lack of uterine tone. (B and D) stimulate uterine contractions. (C) may contract the uterus temporarily and then encourage more afterpains later.

The nurse is caring for a one-year-old child following surgical correction of hypospadias. The nursing action has the highest priority? A. Monitor urinary output. B. Auscultate bowel sounds. C. Observe appearance of stool. D. Record percent of diet eaten.

A. Monitor urinary output. Hypospadias is a birth defect in boys in which the opening of the urethra is not located at the tip of the penis.

When explaining "postpartum blues" to a client who is 1 day postpartum, which symptoms should the nurse include in the teaching plan? (Select all that apply.) A. Mood swings. B. Panic attacks. C. Tearfulness. D. Decreased need for sleep. E. Disinterest in the infant.

A. Mood swings. C. Tearfulness. Correct choices are (A and C). "Postpartum blues" is a common emotional response related to the rapid decrease in placental hormones after delivery and include mood swings (A), tearfulness (C), feeling low, emotional, and fatigued. (B, D, and E) are more characteristic of postpartum depression that typically occurs 3 to 7 days later than postpartum blues.

A nurse is assessing a client who reports that she might be pregnant. Which of the following findings should the nurse identify as a presumptive sign of pregnancy? A. Nausea in the morning. B. Positive home pregnancy test. C. Increased sensitivity of the cervix noted upon examination. D. Gestational sec observed by transvaginal ultrasound.

A. Nausea in the morning. Nausea is a presumptive sign of pregnancy-that is, a subjective symptom reported by the mother that could have a cause other than pregnancy. (Presumptive, probable, and positive signs of pregnancy.)

The nurse has performed a nonstress test on pregnant client and is reviewing the fetal monitor strip. The nurse interprets the test as reactive. How should the nurse document this finding? A. Normal. B. Abnormal. C. The need for further evaluation. D. That findings were difficult to interpret.

A. Normal. A reactive nonstress test indicates that the fetal heart rate is within normal limits and shows adequate variability with two or more accelerations in a 20-minute period. This indicates that the fetus is receiving sufficient oxygen and is not experiencing distress, which is considered a normal finding.

The nurse is monitoring a client in active labor and notes that the client is having contractions every 3 minutes that last 45 seconds. The nurse notes that the fetal heart rate between contractions is 100 beats/minute. Which nursing action is most appropriate? A. Notify the health care provider (HCP). B. Continue monitoring the fetal heart rate. C. Encourage the client to continue pushing with each contraction. D. Instruct the client's coach to continue to encourage breathing techniques.

A. Notify the health care provider (HCP). A normal fetal heart rate is 110 to 160 beats/minute, and the fetal heart rate should be within this range between contractions. Fetal bradycardia between contractions may indicate the need for immediate medical management, and the HCP needs to be notified. Options B, C, and D are inappropriate nursing actions in this situation and delay necessary intervention. - Saunders Comprehensive Review for the NCLEX-RN® Examination 7th Edition

On admission to the prenatal clinic, a 23-year-old woman tells the nurse that her last menstrual period began on February 15, and that previously her periods were regular. Her pregnancy test is positive. This client's expected date of delivery (EDD) is: A. November 22. B. November 8. C. December 22. D. October 22.

A. November 22. (A) correctly applies Nägele's rule for estimating the due date by counting back 3 months from the first day of the last menstrual period (January, December, November) and adding 7 days (15+7=22).

A 30-year-old primigravida delivers a 9-pound (4082 gram) infant vaginally after a 30-hour labor. What is the priority nursing action for this client? A. Observe for signs of uterine hemorrhage. B. Encourage direct contact with the infant. C. Assess the blood pressure for hypertension. D. Gently massage fundus every four hours.

A. Observe for signs of uterine hemorrhage.

A newborn infant, diagnosed with developmental dysplasia of the hip (DDH), is being prepared for discharge. Which nursing intervention should be included in this infant's discharge teaching plan? A. Observe the parents applying a Pavlik harness. B. Provide a referral for an orthopedic surgeon. C. Schedule a physical therapy follow-up home visit. D. Teach the parents to check for hip joint mobility.

A. Observe the parents applying a Pavlik harness. It is important that the hips of infants with hip dysplasia are maintained in an abducted position, which can be accomplished by using the Pavlik harness (A); this keeps the hips and knees flexed, the hips abducted, and the femoral head in the acetabulum. Early treatment often negates the need for surgery, and (B) is not indicated until approximately 6 months of age. (C) is not indicated for hip dysplasia. It is best for the pediatrician to monitor hip joint mobility, and teaching the parents to perform this technique is likely to increase their anxiety (D).

A diabetic client delivers a full term large for gestational age infant who is jittery. What action should the nurse take first? A. Obtain a blood glucose level. B. Administer oxygen. C. Feed the infant glucose water (10%). D. Decrease environmental stimuli.

A. Obtain a blood glucose level. The jitteriness in the large for gestational age infant suggests hypoglycemia, which is a common problem in infants of diabetic mothers due to hyperinsulinemia caused by exposure to high levels of maternal glucose in utero. Therefore, the first action the nurse should take is to obtain a blood glucose level to confirm the diagnosis of hypoglycemia and to determine the severity of the condition.

One hour after delivery, the nurse is unable to palpate the uterine fundus of a client who had an epidural and notes a large amount of lochia on the perineal pad. The nurse massages at the umbilicus and obtains current vital signs. Which intervention should the nurse implement next? A. Palpate the suprapubic area for bladder distention. B. Increase the rate of the oxytocin infusion. C. Provide bedpan to void if unable to ambulate. D. Document number of pad changes in the last hour.

A. Palpate the suprapubic area for bladder distention. The first intervention to implement when the uterus is not contracting adequately is to assess for bladder distention, as a full bladder can prevent the uterus from contracting properly.

A client in active labor is admitted with preeclampsia. Which assessment finding is most significant in planning this client's care? A. Patellar reflex 4+. B. Blood pressure 158/80. C. Four-hour urine output 240 mL. D. Respiration 12/minute.

A. Patellar reflex 4+. A 4+ reflex in a client with pregnancy-induced hypertension (A) indicates hyperreflexia, which is an indication of an impending seizure. Although (B) is significant, some individuals have preeclampsia superimposed on chronic hypertension, and an elevated blood pressure alone is not as significant a finding as (A). (C and D) are important, but these findings are within normal range.

A nurse is caring for a client who is recieving oxytocin to induce labor. which of the following actions should the nurse take? A. Perform continuous FHR monitoring. B. Measure maternal temp every hour. C. Evaluate the maternal contraction pattern every hour. D. Check blood pressure every 5 min.

A. Perform continuous FHR monitoring. When oxytocin is administered to an antepartum client, the fetal monitor must be used to monitor the FHR and maternal contractions continuously.

The nurse is caring for a postpartum patient who is exhibiting symptoms of spinal headaches 24 hours following delivery of a normal newborn. Prior to anesthesiologist's arrival on the unit, which action should the nurse perform? A. Place procedure equipment at bedside. B. Apply an abdominal binder. C. Cleanse the spinal injection site. D. Insert an indwelling foley catheter.

A. Place procedure equipment at bedside.

A nurse is caring for a client who recently had birth and plans to breastfeed. Which of the following actions should the nurse take? A. Place the unwrapped newborn on the mother's bare chest. B. Feed the infant 5-15mL 5% glucose water to assess the suck/swallow reflex. C. Bathe the newborn under running warm water before feeding. D. Administer vitamin K and eye prophylaxis prior to feeding.

A. Place the unwrapped newborn on the mother's bare chest. Skin-to-skin contact will maintain the newborn's temperature and illicit instinctive newborn feeding behaviors.

In preparing a gravid client for a triple screen analysis, which action should the nurse take? A. Prepare to draw blood for analysis. B. Encourage the client to drink 8 oz of water. C. Assist the client to left lateral tilt position. D. Apply an external fetal monitor to the abdomen.

A. Prepare to draw blood for analysis.

A primipara has delivered a stillborn fetus at 30-weeks gestation. To assist the parents with the grieving process, which intervention is most important for the nurse to implement? A. Provide an opportunity for the parents to hold their infant in privacy. B. Assist the couple in completing a request for autopsy. C. Encourage the couple to seek family counseling within the next few weeks. D. Explain the possible causes of fetal demise.

A. Provide an opportunity for the parents to hold their infant in privacy. Allowing the parents to hold their stillborn infant provides them with an opportunity to bond with their child and begin the grieving process. This intervention is often recommended by healthcare providers to help parents cope with the loss of their child. While the other options may be appropriate, providing an opportunity for the parents to hold their infant in privacy is the most important intervention for the nurse to implement at this time.

Which type of anesthesia, used with a client in labor, produces a loss of sensation only to the vagina and perineum? A. Pudendal block. B. Epidural block. C. Saddle block. D. Paracervical block.

A. Pudendal block. -HESI

A client receiving epidural anesthesia begins to experience nausea and becomes pale and clammy. What intervention should the nurse implement first? A. Raise the foot of the bed. B. Assess for vaginal bleeding. C. Evaluate the fetal heart rate. D. Take the client's blood pressure.

A. Raise the foot of the bed. These symptoms are suggestive of hypotension which is a side effect of epidural anesthesia. Raising the foot of the bed will increase venous return and provide blood to the vital areas. Increasing the IV fluid rate using a balanced non-dextrose solution and ensuring that the client is in a lateral position are also appropriate interventions. (B and C) will not raise the maternal blood pressure. Since the symptoms are common side effects of epidural anesthesia and suggest hypotension, (D) can wait until (A) is implemented.

The nurse is providing discharge teaching for a client who is 24 hours postpartum. The nurse explains to the client that her vaginal discharge will change from red to pink and then to white. The client asks, "What if I start having red bleeding after it changes?" What should the nurse instruct the client to do? A. Reduce activity level and notify the healthcare provider. B. Go to bed and assume a knee-chest position. C. Massage the uterus and go to the emergency room. D. Do not worry as this is a normal occurrence.

A. Reduce activity level and notify the healthcare provider. Lochia should progress in stages from rubra (red) to serosa (pinkish) to alba (whitish), and not return to red. The return to rubra usually indicates subinvolution or infection. If such a sign occurs, the mother should notify the clinic/healthcare provider and reduce her activity to conserve energy (A). Going to bed, or resting might be helpful, but (B) is not indicated. (C) would be an over-reaction and the uterus might not be palpable at that time. This is not a normal occurrence (D).

A nurse is caring for a newborn whose mother received magnesium sulfate to treat preterm labor. Which of the following clinical manifestations in the newborn indicates toxicity due to the magnesium sulfate therapy? A. Respiratory depression. B. Hypothermia. C. Hypoglycemia. D. Jaundice.

A. Respiratory depression. Magnesium sulfate can cause respiratory and neuromuscular depression in the newborn. The nurse should monitor the newborn for clinical manifestations of respiratory depression.

A postpartum client who is Rh-negative refuses to receive Rho(D) immune globulin (RhoGAM) after delivery of an infant who is Rh-positive. Which information should the nurse provide this client? A. RhoGam prevents maternal antibody formation for future Rh-positive babies. B. RhoGAM is not necessary unless all her pregnancies are Rh-positive. C. The R-positive factor from the fetus threatens her blood cells. D. The mother should receive RhoGAM when the baby is Rh-negative.

A. RhoGam prevents maternal antibody formation for future Rh-positive babies

A nurse is performing an initial physical assessment of a newborn following a vaginal birth. Which of the following findings should the nurse report to the provider? A. Small, pinpoint, reddish-purple spots on the chest. B. Bluish coloring of the feet. C. Overlapping suture lines. D. White, cheese-like substance covering the skin.

A. Small, pinpoint, reddish-purple spots on the chest. These marks are petechiae, which are commonly found above the neck if the umbilical cord was around the newborn's neck at birth. Petechiae in any other circumstance should be reported because this finding can indicate infection or a low platelet count.

Client teaching is an important part of the maternity nurse's role. Which factor has the greatest influence on successful teaching of the gravid client? A. The client's readiness to learn. B. The client's educational background. C. The order in which the information is presented. D. The extent to which the pregnancy was planned.

A. The client's readiness to learn. When teaching any client, readiness to learn is the most important criterion. For example, the client with severe morning sickness in the first trimester may not be "ready to learn" about labor and delivery, but is probably very "ready to learn" about ways to relieve morning sickness. (B and C) are factors that may influence learning, but they are not as influential as (A). Even if a pregnancy is planned and very desirable (D), the client must be ready to learn the content presented.

The nurse is teaching a postpartum client about breast-feeding. Which instruction should the nurse include? A. The diet should include additional fluids. B. Prenatal vitamins should be discontinued. C. Soap should be used to cleanse the breasts. D. Birth control measures are unnecessary while breast-feeding.

A. The diet should include additional fluids. Breastfeeding mothers require additional fluids to stay hydrated and to support milk production. Prenatal vitamins should be taken as prescribed and soap should not be used on the breast because it removes natural oils which increases the chance of cracked nipples. Breast-feeding is not a sole method of contraception, so birth control measures should be resumed.

When assessing a multiparous woman who has just given birth to an 8-pound boy, the nurse notes that the woman's fundus is firm and has become globular in shape. A gush of dark red blood comes from her vagina. The nurse concludes that: A. The placenta has separated. B. A cervical tear occurred during the birth. C. The woman is beginning to hemorrhage. D. Clots have formed in the upper uterine segment.

A. The placenta has separated. Placental separation is indicated by a firmly contracting uterus, a change in the uterus from a discoid to a globular ovoid shape, a sudden gush of dark red blood from the introitus, an apparent lengthening of the umbilical cord, and a finding of vaginal fullness. Cervical tears that do not extend to the vagina result in minimal blood loss. Signs of hemorrhage are a boggy uterus, bright red vaginal bleeding, alterations in vital signs, pallor, lightheadedness, restlessness, decreased urinary output, and alteration in the level of consciousness. If clots have formed in the upper uterine segment, then the nurse would expect to find the uterus boggy and displaced to the side.

With regard to a woman's intake and output during labor, the nurse should be aware that: A. The tradition of restricting the laboring woman to clear liquids and ice chips is being challenged because regional anesthesia is used more often than general anesthesia and studies are not showing harm from drinking fluids in labor. B. Intravenous (IV) fluids usually are necessary to ensure that the laboring woman stays hydrated. C. Routine use of an enema empties the rectum and is very helpful for producing a clean, clear delivery. D. When a nulliparous woman experiences the urge to defecate, it often means birth will follow quickly.

A. The tradition of restricting the laboring woman to clear liquids and ice chips is being challenged because regional anesthesia is used more often than general anesthesia and studies are not showing harm from drinking fluids in labor. Women are awake with regional anesthesia and are able to protect their own airway, which reduces the worry over aspiration. Routine IV fluids during labor are unlikely to be beneficial and may be harmful. The routine use of an enema is, at best, ineffective and may be harmful. Having the urge to defecate followed by the birth of her fetus is true for a multiparous woman but not for a nulliparous woman.

A 42-week gestational client is receiving an intravenous infusion of oxytocin (Pitocin) to augment early labor. The nurse should discontinue the oxytocin infusion for which pattern of contractions? A. Transition labor with contractions every 2 minutes, lasting 90 seconds each. B. Early labor with contractions every 5 minutes, lasting 40 seconds each. C. Active labor with contractions every 31 minutes, lasting 60 seconds each. D. Active labor with contractions every 2 to 3 minutes, lasting 70 to 80 seconds each.

A. Transition labor with contractions every 2 minutes, lasting 90 seconds each. Contraction pattern (A) describes hyperstimulation and an inadequate resting time between contractions to allow for placental perfusion. The oxytocin infusion should be discontinued. There is an appropriate resting period between contractions for clients who are experiencing contraction patterns (B and D). Oxytocin can benefit the client in contraction pattern (C) by causing the contractions to come closer together and to become more efficient.

A client at 20-weeks gestation comes to the antepartum clinic complaining of vaginal warts (human papillomavirus). What information should the nurse provide this client? A. Treatment options, while limited due to the pregnancy, are available. B. The client should be treated with Penicillin G. C. This client should be treat with acyclovir (Zovirax). D. Termination of the pregnancy should be considered.

A. Treatment options, while limited due to the pregnancy, are available. Some treatments may be contraindicated during pregnancy due to potential risks to the developing fetus. Acyclovir is an antiviral medication that is primarily used to treat herpes simplex virus infections.

Which nursing intervention is helpful in relieving "afterpains" (postpartum uterine contractions)? A. Using relaxation breathing techniques. B. Using a breast pump. C. Massaging the abdomen. D. Giving oxytocic medications.

A. Using relaxation breathing techniques. Periodic contraction and relaxation of the uterus causes "afterpains." Relaxation breathing techniques (A) provide distraction, reducing the perception of pain. (B) stimulates uterine contractions. (C) may contract the uterus temporarily and then encourage more afterpains later. (D) stimulates afterpains/uterine contractions.

The nurse caring for the woman in labor should understand that maternal hypotension can result in: A. Uteroplacental insufficiency. B. Spontaneous rupture of membranes. C. Fetal dysrhythmias. D. Early decelerations.

A. Uteroplacental insufficiency.

A client at 33-weeks gestation is admitted with a moderate amount of vaginal bleeding and no contractions are noted on the external monitor. Which intervention should the nurse implement? A. Weight perineal pads. B. Weight daily. C. Measure intake and output. D. Ambulate 15 minutes QID.

A. Weight perineal pads. The nurse should weigh perineal pads to determine the amount of blood loss. This will help determine the severity of the situation and inform the health care provider of the need for further interventions or treatment.

The nurse is assessing a 3-day old infant with a cephalohematoma in the newborn nursery. Which assessment finding should the nurse report to the healthcare provider? A. Yellowish tinge to the skin. B. Babinski reflex present bilaterally. C. Pink papular rash on the face. D. Moro reflex noted after a loud noise.

A. Yellowish tinge to the skin. Cephalohematomas are characterized by bleeding between the bone and its covering, the periosteum. Due to the breakdown of the red blood cells within a hematoma, the infant is at a greater risk for jaundice, so (A) should be reported. (B, C, and D) are expected assessment findings in a newborn.

Twenty-four hours after admission to the newborn nursery, a full-term male infant develops localized edema on the right side of his head. The nurse knows that, in the newborn, an accumulation of blood between the periosteum and skull which does not cross suture lines is a newborn variation known as: A. a cephalhematoma, caused by forceps trauma and may last up to 8 weeks. B. a subarachnoid hematoma, which requires immediate drainage to prevent further complications. C. molding, caused by pressure during labor and will disappear within 2 to 3 days. D. a subdural hematoma which can result in lifelong damage.

A. a cephalhematoma, caused by forceps trauma and may last up to 8 weeks. Cephalhematoma (A), a slight abnormal variation of the newborn, usually arises within the first 24 hours after delivery. Trauma from delivery causes capillary bleeding between the periosteum and the skull. (C) is a cranial distortion lasting 5 to 7 days and is caused by pressure on the cranium during vaginal delivery--it is a normal finding, or a common variation of the newborn. (B and D) both involve intracranial bleeding, and could not be detected by physical assessment alone.

During a 26 week gestation prenatal exam, a client reports occasional dizziness. What intervention is best for the nurse to recommend the client? A. Elevate the head with two pillows while sleeping. B. Lie on the left or right side when sleeping or resting. C. Increase intake of foods that are high in iron. D. Decrease the amount of carbohydrates in the diet.

A/B

In assessing a client diagnosed with preeclampsia who is receiving magnesium sulfate, the nurse determines that her deep tendon reflexes are 1+, respiratory rate is 12 breaths/minute, urinary output is 50 mL in 4 hours, magnesium sulfate level is 9 mEq/L (4.5 mmol/L). Based on these findings, what intervention should the nurse implement? A. Stop the magnesium sulfate infusion immediately. B. Administer calcium gluconate immediately. C. Continue the magnesium sulfate infusion as prescribed. D. Decrease the magnesium sulfate infusion by one-half.

A/B The assessment findings suggest that the client is experiencing magnesium sulfate toxicity, which can lead to respiratory depression, decreased urinary output, and decreased deep tendon reflexes. The therapeutic level of magnesium sulfate is 4-8 mEq/L (2-4 mmol/L), and a level greater than 8 mEq/L (4 mmol/L) is considered toxic. a?

The nurse teaching a preconception preparation class is discussing ways to improve dietary folic acid intake. Which evening snack contains the most folic acid? A. Fresh strawberries. B. Roasted peanuts in shell. C. Unflavored yogurt. D. Vanilla milkshake with protein supplement. Toasted white bread with butter. Whole grain cereal and milk. Hard-boiled egg and juice

A/B (Whole grain cereal and milk)?

Following the vaginal delivery of a large for gestation age (LGA) infant, a woman is admitted to the ICU due to post-partum hemorrhaging. The client's medical record describes Jehovah's Witness notes as her religion. What action should the nurse take next? A. Inform the client of the critical need for a blood transfusion. B. Obtain consent from the family to infuse packed red blood cells. C. Clarify the clients wishes about receiving blood products. D. Prepare to infuse multiple units of fresh frozen plasma.

A/C

When assessing a pregnant woman who is 39 weeks gestation who is admitted to labor and delivery, which finding is most important to report the health care provider? A. + 1 proteinuria. B. 130/70 blood pressure. C. 102 oral temperature. D. +1 pedal edema.

A/C

What is the priority nursing assessment immediately following the birth of an infant with esophageal atresia and a tracheoesophageal (TE) fistula? A . Body temperature. B. Level of pain. C. Time of first void. D. Number of vessels in the cord

A/C Body temp?

A multiparous women at 38 weeks gestation with a history of rapid progression of labor is admitted for induction due to signs and symptoms of pregnancy induced hypertension (PIH). One hour after the oxytocin infusion is initiated she complains of a headache. Her contractions are occurring every 1-2 mins, lasting 60-75 seconds and a vaginal exam reveals that her cervix is 90% and dilated 6 cm. What intervention is most important for the nurse to implement? A. Prepare for immediate delivery. B. Measure deep tendon reflexes. C. Discontinue the Pitocin infusion. D. Turn the client to her left side.

A/C/D?

A neonate who has congenital adrenal hypoplasia (CAH) presents with ambiguous genitalia. What is the primary nursing consideration when supporting the parents of a child with this anomaly? A. Discuss the need for cortisol and aldosterone replacement therapy after discharge. B. Support the parents in their decision to assign sex of their child according to their preference. C. Offer information about ultrasonography and genotyping to determine sex assignment. D. Explain that corrective surgical procedures consistent with sex assignment can be delayed.

A/C?

A full term, 24-hour-old infant in the nursery regurgitates and suddenly turns cyanotic. What should the nurse do first? A. Suction the oral and nasal passages. B. Give oxygen by positive pressure. C. Stimulate the infant to cry. D. Turn the infant onto the right side.

A/C? A?

After placing a 36-week-gestation newborn in an isolette and drying the infant with several blankets, what should the nurse implement next? A. Place erythromycin ophthalmic ointment in both eyes. B. Open the isolette door to assess the infant's vital signs. C. Administer the vitamin K injection. D. Remove the wet blankets and linens from the isolette.

A/D D?

An unlicensed assistive personnel (UAP) reports to the charge nurse that a client who delivers a 7-pound infant 12 hours ago is reporting a severe headache. The client blood pressure is 110/70 mmHg, respiratory rate is 18 breaths/minute, heart rate is 74 bpm, and temperature is 98.6F. The client's fundus is firm and one fingerbreadth above the umbilicus. Which action should the charge nurse implement first? A. Notify the healthcare provider of the assessment findings. B. Obtain a STAT hemoglobin and hematocrit. C. Assign a practical nurse (PN) to reassess the client's vital signs. D. Determine if the client received anesthesia during delivery.

A/D? A. Notify the healthcare provider of the assessment findings.

A toddler with a history of a cyanotic defect is admitted to the pediatric intensive care. Respiration rate 60 beats / min and heart 150 beats/ min. What action should the nurse take first? A. Obtain a pulse ox reading. B. Assess child's blood pressure. C. Perform a neurological assessment. D. Initiate peripheral intravenous access.

A?

The nurse is using the Silverman-Anderson index to assess an infant with respiratory distress and determines that the infant is demonstrating marked nasal flaring, an audible expiratory grunt, and just visible intercostal and xiphoid retractions. Using this scale, which score should the nurse assign?A. 3B. 4C. 5D. 8

ANS: CThe Silverman-Anderson index is an assessment scale that scores a newborn's respiratory status as grade 0, 1, or 2 for each component; it includes synchrony of the chest and abdomen, retractions, nasal flaring, and expiratory grunt. No respiratory distress is graded 0 and a total of 10 indicates maximum respiratory distress. This infant is demonstrating respiratory distress with maximal effort, so a grade 2 is assigned for marked nasal flaring, grade 2 for an audible expiratory grunting, plus grade 1 for just visible retractions, which is a total score of 5 (C). (A, B, and D) are not accurate.

A female client with insulin-dependent diabetes arrives at the clinic seeking a plan to get pregnant in approximately 6 months. She tells the nurse that she want to have an uncomplicated pregnancy and a healthy baby. What information should the nurse share with the client? A. Your current dose of Insulin should be maintained throughout your pregnancy. B. Maintain blood sugar levels in a constant range within normal limits during pregnancy. C. The course and outcome of your pregnancy is not an achievable goal with diabetes. D. Expect an increase in insulin dosages by 5 units/week during the first trimester.

B) Maintain blood sugar levels in a constant range within normal limits during pregnancy. Maintaining blood sugar within a normal range during pregnancy has a strong correlation with a good outcome. Insulin requirements normally change during pregnancy (A). Active participation of the client with her diabetes management during pregnancy is associated with better outcomes, not (C). Insulin needs are individually determined by blood glucose values, not a set schedule, not (D).

A 30-year-old gravida 2, para 1 client is admitted to the hospital at 26-weeks gestation in preterm labor. She is started on an IV solution of terbutaline (Brethine). Which assessment is the highest priority for the nurse to monitor during the administration of this drug? A) Maternal blood pressure and respirations. B) Maternal and fetal heart rates. C) Hourly urinary output. D) Deep tendon reflexes.

B) Maternal and fetal heart rates. Monitoring maternal and fetal heart rates (B) is most important when terbutaline is being administered. Terbutaline acts as a sympathomimetic agent that stimulates both beta 1 receptors (causing tachycardia, a side effect of the drug) and stimulation of beta 2 receptors (causing uterine relaxation, a desired effect of the drug). While monitoring (A, C, and D) is helpful, these do not have the priority of monitoring (B) when a beta- adrenergic agonists is administered. (Terbutaline may cause tachycardia.)

A client who is anovulatory and has hyperprolactinemia is being treated for infertility with metformin, menotropins (Repronex, MENOPUR®), and human chorionic gonadotropin(hCG). Which side effects should the nurse tell the client to report immediately? A) Episodes of headache and irritability B) Nausea and vomiting C) Rapid increase in abdominal girth D) Persistent daytime fatigue

B) Nausea and vomiting - HESI

A nurse is teaching a parent of a newborn how to care for the newborn's umbilical cord stump. Which of the following instructions should the nurse include? A. "Cover the cord with the edge of the diaper." B. "Clean the cord stump with tap water." C. "Apply a damp cloth over the cord stump once each day." D. "You should gently tug on the cord stump in 5 days if it has not yet fallen off."

B. "Clean the cord stump with tap water." The nurse should instruct the parent to cleanse around the cord stump with tap water to promote healing and prevent infection.

A client in the first trimester of pregnancy arrives at a health care clinic and reports that she has been experiencing vaginal bleeding. A threatened abortion is suspected, and the nurse instructs the client regarding management of care. Which statement made by the client indicates a need for further instruction? A. "I will watch for the evidence of the passage of tissue." B. "I will maintain strict bed rest throughout the remainder of the pregnancy." C. "I will count the number of perineal pads used on a daily basis and note the amount and color of blood on the pad." D. "I will avoid sexual intercourse until the bleeding has stopped, and for 2 weeks following the last evidence of bleeding."

B. "I will maintain strict bed rest throughout the remainder of the pregnancy." Strict bed rest throughout the remainder of the pregnancy is not required for a threatened abortion. The client should watch for the evidence of the passage of tissue. The client is instructed to count the number of perineal pads used daily and to note the quantity and color of blood on the pad. The client is advised to curtail sexual activities until bleeding has ceased and for 2 weeks after the last evidence of bleeding or as recommended by the health care provider. - Saunders Comprehensive Review for the NCLEX-RN Examination

A nurse is caring for a primigravida client who is at 8 weeks gestation with twins. The client states that even though she and her husband planned this pregnancy, she is experiencing ambivalent feelings about it. Which of the following responses should the nurse make? A. "Have you told your husband about these feelings? B. "These feelings are quite normal at the beginning of pregnancy." C. "Perhaps you should see a counselor to discuss these feelings." D. "I am quite concerned about these feelings. Could you explain more?

B. "These feelings are quite normal at the beginning of pregnancy." The client needs reassurance that these feelings are normal and that there is no reason for concern.

The nurse notes hypotonia, irritability, and a poor sucking reflex in a full-term newborn on admission to the nursery. The nurse suspects fetal alcohol syndrome and is aware that which additional sign would be consistent with this syndrome? A. Length of 19 inches. B. Abnormal palmar creases. C. Birth weight of 6 lb, 14 oz 4. D. Head circumference appropriate for gestational age.

B. Abnormal palmar creases. Abnormal palmar creases in FAS are one of the many physical signs and symptoms of the condition. Alcohol exposure during fetal development can lead to incomplete or fused creases, or a single deep crease on the palm.

The nurse can help their clients by keeping them informed about the distinctive stages of labor. What description of the phases of the first stage of labor is accurate? A. Latent: Mild, regular contractions; no dilation; bloody show; duration of 2 to 4 hours. B. Active: Moderate, regular contractions; 4- to 7-cm dilation; duration of 3 to 6 hours. C. Lull: No contractions; dilation stable; duration of 20 to 60 minutes. D. Transition: Very strong but irregular contractions; 8- to 10-cm dilation; duration of 3-4 hours.

B. Active: Moderate, regular contractions; 4- to 7-cm dilation; duration of 3 to 6 hours.

A nurse is planning care for a client who is positive for HIV. Which of the following actions should the nurse take after the baby is born? A. Encourage the mother to breastfeed. B. Administer the hepatitis B vaccine prior to discharge. C. Implement contact and droplet precautions when providing care for the infant. D. Collect a cord blood specimen to test for the presence of HIV.

B. Administer the hepatitis B vaccine prior to discharge. Infants who are exposed to HIV should receive all routine vaccinations. Infants who are infected with HIV can received all inactivated vaccinations. (Baby is immunocompromised.)

The nurse caring for a laboring client encourages her to void at least q2h, and records each time the client empties her bladder. What is the primary reason for implementing this nursing intervention? A. Emptying the bladder during delivery is difficult because of the position of the presenting fetal part. B. An over-distended bladder could be traumatized during labor as well as prolong the progress of labor. C. Urine specimens for glucose and protein must be obtained at certain intervals throughout labor. D. Frequent voiding minimizes the need for catheterization which increases the chance of bladder infection.

B. An over-distended bladder could be traumatized during labor as well as prolong the progress of labor. A full bladder can impair the efficiency of the uterine contractions and impede descent of the fetus during labor (B). Also, because of the close proximity of the bladder to the uterus, the bladder can be traumatized by the descent of the fetus. It is not difficult to empty the bladder during delivery (A). Urine specimens are obtained only by special order (C). There is danger of infection due to catheterization (D), but this is not the primary reason for encouraging the client to void during labor.

A nurse in a prenatal clinic is performing telephone triage for several clients. Which of the following client reports should the nurse identify as an expected physiological adaptation to pregnancy? A. Spotting with urination. B. Breast tenderness. C. Thick, white vaginal discharge. D. Facial swelling.

B. Breast tenderness Breast tenderness is common during the first and third trimester of pregnancy. The nurse should explain that this is expected and that the client should wear a well-fitting, supportive bra to help alleviate the tenderness.

A client with gestational hypertension is in active labor and receiving an infusion of magnesium sulfate. Which drug should the nurse have available for signs of potential toxicity? A. Oxytocin (Pitocin). B. Calcium gluconate. C. Terbutaline (Brethine). D. Naloxone (Narcan).

B. Calcium gluconate. The antidote for magnesium sulfate is calcium gluconate (B), which should be readily available if the client manifest signs of toxicity. (A, C, and D) are not effective in the reversal of magnesium sulfate.

A multiparous client at 36 hours postpartum reports increased bleeding and cramping. On examination the nurse finds the uterine fundus 2 centimeters above the umbilicus. Which action should the nurse take first? A. Increase the intravenous fluid to 150ML/hr. B. Call the health care provider. C. Encourage the client to void. D. Administer ibuprofen 800 milligrams by mouth.

B. Call the health care provider.

A nurse is assessing a newborn. Which of the following findings should the nurse report to the provider? A. Anterior fontanel of 5 cm. B. Central cyanosis. C. Edematous scrotum. D. Capillary refill of under 2 seconds.

B. Central cyanosis Central cyanosis is an indication of compromised cardiorespiratory status. Other manifestations include tachypnea, nasal flaring, retractions, and grunting

A pregnant woman in the first trimester of pregnancy has hemoglobin of 8.6 mg/dL and a hematocrit of 25.1%. What food should the nurse encourage this client to include in her diet? A. Carrots. B. Chicken. C. Yogurt. D. Cheese.

B. Chicken. A pregnant woman in the first trimester of pregnancy with low hemoglobin and hematocrit levels should be encouraged to eat foods rich in iron to help increase the levels. Therefore, the nurse should encourage the client to include chicken in her diet. While carrots, yogurt, and cheese are nutritious foods, they are not particularly high in iron.

A client at 35 weeks gestation complains of a "pain whenever the baby moves." On assessment, the nurse notes the client's temperature to be 101.2 F (38.4 C), with severe abdominal and uterine tenderness on palpation. The nurse knows that these findings are indicative of what condition? A. Round ligament strain. B. Chorioamnionitis. C. Abruptio placenta. D. Viral infection.

B. Chorioamnionitis. The findings of a fever, severe abdominal and uterine tenderness on palpation, and pain with fetal movement are indicative of chorioamnionitis, which is an infection of the amniotic fluid and membranes. Chorioamnionitis can cause complications for both the mother and the fetus, and may lead to preterm labor or other serious infections.

The nurse is conducting postpartum teaching with a mother who is breastfeeding her infant. When discussing birth control, which method should the nurse recommend to this client as best for her to use in preventing an unwanted pregnancy? A. Breastfeed exclusively at least every 3-4 hours. B. Condoms and contraceptive foam or gel. C. Rhythm method (natural family planning). D. Combined estrogen-progesterone oral contraceptives.

B. Condoms and contraceptive foam or gel. Combined estrogen-progesterone oral contraceptives are contraindicated while breastfeeding because it (estrogen) decreases milk supply.

A new mother, who is lacto-ovo vegetarian, plans to breastfeed her infant. What information should the nurse provide prior to discharge? A. Avoid using lanolin-based nipple cream or ointment. B. Continue prenatal vitamins with B12 while breast feeding. C. Offer iron- fortified supplemental formula daily. D. Weigh the baby weekly to evaluate the newborns growth.

B. Continue prenatal vitamins with B12 while breast feeding. Lacto-ovo vegetarians may not consume enough vitamin B12, which is essential for the infant's growth and development. Prenatal vitamins containing vitamin B12 should be continued during breastfeeding. (A lacto-ovo vegetarian is someone who does not eat meat, poultry, or fish, but still consumes dairy products (lacto) and eggs (ovo).)

Following a traumatic delivery an infant receives an initial Apgar score of 3. Which intervention is most important for the nurse to implement? A. Page the pediatrician STAT. B. Continue resuscitative efforts. C. Repeat the Apgar assessment in 5 minutes. D. Inform the parents of the infant's condition.

B. Continue resuscitative efforts.

A newborn with a respiratory rate of 40 breaths per minute at one minute after birth is demonstrating cyanosis of the hands and feet. What action should the nurse take? A. Assess bowel sounds. B. Continue to monitor. C. Assist with intubation. D. Rub the infant's back.

B. Continue to monitor. Cyanosis of the hands and feet are common, especially 1 minute after birth. Respirations are normal.

A nurse is assessing a client who is at 12 weeks gestation and has a hydatidiform mole. Which of the following findings should the nurse expect? A. Hypothermia. B. Dark brown vaginal discharge. C. Decreased urinary output. D. Fetal heart tones.

B. Dark brown vaginal discharge. A hydatidiform mole (a molar pregnancy) is a benign proliferative growth of the chronic villi that gives rise to multiple cysts. The products of conception transform into a large number of edematous, fluid-filled vesicles. As cells slough off the uterine wall, vaginal discharge is usually dark brown and can contain grape-like clusters.

A nurse is caring for a client who is in labor. The nurse decides to switch from intermittent auscultation to continuous fetal monitoring. Which of the following data can only be obtained from continuous electronic fetal monitoring? A. Determination of a baseline. B. Determination of variability. C. Presence of accelerations. D. Presence of decelerations.

B. Determination of variability. Continuous electronic fetal monitoring is required to determine variability since the nurse needs a monitor tracing to quantify variability.

The nurse is assisting a client undergoing induction of labor at 41 weeks' gestation. The client's contractions are moderate and occurring every 2 to 3 minutes, with a duration of 60 seconds. An internal fetal heart rate monitor is in place. The baseline fetal heart rate has been 120 to 122 beats/minute for the past hour. What is the priority nursing action? A. Notify the health care provider. B. Discontinue the infusion of oxytocin (Pitocin). C. Place oxygen on at 8 to 10 L/minute via face mask. D. Contact the client's primary support person(s) if not currently present.

B. Discontinue the infusion of oxytocin (Piton). The priority nursing action is to stop the infusion of oxytocin. Oxytocin can cause forceful uterine contractions and decrease oxygenation to the placenta, resulting in decreased variability. After stopping the oxytocin, the nurse should reposition the laboring mother. Applying oxygen, increasing the rate of the intravenous (IV) fluid (the solution without the oxytocin), and notifying the health care provider are also actions that are indicated in this situation. Contacting the client's primary support person(s) is not the priority action at this time.

A 25-year-old client who had a severe postpartum hemorrhage following the vaginal birth of twins is transferred to the postpartum unit. The nurse knows that assessment for what complication has the highest priority for this client? A. Hard, painful uterine afterpains. B. Disseminated intravascular coagulation. C. Postpartum psychosis. D. Placenta accreta.

B. Disseminated intravascular coagulation. - HESI Following a severe postpartum hemorrhage, the client is at risk for developing disseminated intravascular coagulation (DIC), a potentially life-threatening condition characterized by abnormal blood clotting and bleeding. DIC is a medical emergency that requires immediate treatment to prevent further complications. Placenta accreta, is a potential complication following childbirth, but it is typically identified and managed during the antenatal period, before delivery. If a client has already delivered twins, it is unlikely that placenta accreta is the complication that has the highest priority for the postpartum period. Instead, the client is at a higher risk for complications such as postpartum hemorrhage and disseminated intravascular coagulation.

A client who has an autosomal dominant inherited disorder is exploring family planning options and the risk of transmission of the disorder to an infant. The nurse's response should be based on what information? A. Males inherit the disorder with a greater frequency than females. B. Each pregnancy carries a 50% chance of inheriting the disorder. C. The disorder occurs in 25% of pregnancies. D. All children will be carriers of the disorder.

B. Each pregnancy carries a 50% chance of inheriting the disorder. According to the laws of inheritance, an autosomal dominant disorder has a 50% chance of being transmitted with each pregnancy (B), and if transmitted, the disorder will appear in the child. Males do not inherit autosomal dominant disorders more frequently than females (A). There is a 25% chance of receiving an affected gene in autosomal recessive (C), not autosomal dominant disorders. (D) is incorrect.

A 28-year-old client in active labor complains of cramps in her leg. What intervention should the nurse implement? A. Massage the calf and foot. B. Extend the leg and dorsiflex the foot. C. Lower the leg off the side of the bed. D. Elevate the leg above the heart.

B. Extend the leg and dorsiflex the foot. Dorsiflexing the foot by pushing the sole of the foot forward or by standing (if the client is capable) (B), and putting the heel of the foot on the floor is the best means of relieving leg cramps. (A) is ineffective for leg cramps caused by phosphorous/calcium imbalances and may dislodge small thrombus. (C) would not be helpful. (D) is used to promote venous return, but is not indicated for leg cramps.

A client in active labor complains of cramps in her leg. What intervention should the nurse implement? A. Ask the client if she takes a daily calcium tablet. B. Extend the leg and dorsiflex the foot. C. Lower the leg off the side of the bed. D. Elevate the leg above the heart.

B. Extend the leg and dorsiflex the foot. Dorsiflexing the foot by pushing the sole of the foot forward or by standing (if the client is capable) (B), and putting the heel of the foot on the floor is the best means of relieving leg cramps. (A) is not related to leg cramps caused by reduced circulation to the foot. (C) is not likely to be helpful. (D) is used to promote venous return, but is not indicated for leg cramps.

While assessing a 40-week gestation primigravida in active labor, the client's membranes rupture spontaneously and the nurse notices that the amniotic fluid is meconium stained. Which additional finding is most important for the nurse to report to the healthcare provider? A. Maternal blood pressure of 130/85 mmHg. B. Fetal heart rate of 100 to 110 bpm. C. Vaginal exam reveals a cervix 6cm dilated. D. Contractions occurring every 2-3 minutes.

B. Fetal heart rate of 100 to 110 bpm.

The nurse is performing an assessment of a client who is scheduled for a cesarean delivery. Which assessment finding would indicate the need to contact the health care provider? A. Hemoglobin of 11 g/dL. B. Fetal heart rate of 180 beats/minute. C. Maternal pulse rate of 85 beats/minute. D. White blood cell count of 12,000 cells/mm3.

B. Fetal heart rate of 180 beats/minute. A fetal heart rate of 180 beats per minute is considered elevated and requires further assessment and possible intervention by the health care provider. The other assessment findings are within normal limits or do not indicate a need for immediate contact with the health care provider.

On the first postpartum day, the nurse examines the breasts of the new mother. Which condition is the nurse most likely to: A. Slightly firm with immediate let down response B. Filing and secreting colostrum C. Soft, with no change from before delivery D. Firm, larger very tender to touch

B. Filing and secreting colostrum Colostrum is excreted during the first few days after giving birth, typically between the second and fifth day. Colostrum is considered the first milk that a mother produces for her newborn.

Through vaginal examination the nurse determines that a woman is 4 cm dilated, and the external fetal monitor shows uterine contractions every 3.5 to 4 minutes. The nurse would report this as: A. First stage, latent phase. B. First stage, active phase. C. First stage, transition phase. D. Second stage, latent phase.

B. First stage, active phase. The first stage, active phase of maternal progress indicates that the woman is in the active phase of the first stage of labor. (4 cm dilated with contractions every 3.5 to 4 minutes.) During the latent phase of the first stage of labor, the expected maternal progress would be 0 to 3 cm dilation with contractions every 5 to 30 minutes. During the transition phase of the first stage of labor, the expected maternal progress is 8 to 10 cm dilation with contractions every 2 to 3 minutes. During the latent phase of the second stage of labor, the woman is completely dilated and experiences a restful period of laboring down.

The nurse is performing a newborn assessment. Which symptoms, if present in a newborn, would indicate respiratory distress? A. Shallow and irregular respirations. B. Flaring of the nares. C. Abdominal breathing with synchronous chest movement. D. Respiratory rate of 50 breaths per minute.

B. Flaring of the nares. The five symptoms of respiratory distress are retractions (pulling in of the chest wall), tachypnea (rate >60), dusky color or circumoral cyanosis, expiratory grunt, and flaring nares.

A newborn's assessment reveals spina bifida occulta. Which maternal factors should nurse identify as having the greatest impact on the development of this newborn complication? A. Short interval pregnancy B. Folic acid deficiency C. Preeclampsia D. Tobacco use

B. Folic acid deficiency.

At 20-weeks gestation, a client who has gained 20 pounds during this pregnancy tells the nurse that she is feeling fetal movement. Fundal height measurement is 20 cm, and the only complaint is that her breasts are leaking clear fluid. Which assessment finding warrants further evaluation? A. Presence of fetal movement. B. Gestational weight gain. C. Fundal height measurement. D. Leakage from breasts.

B. Gestational weight gain. - HESI

A 3-hour old male infant's hands and feet are cyanotic, and he has an axillary temperature of 96.5 F, a respiratory rate of 40 breaths/min, and a heart rate of 165 beats/min. Which nursing intervention is best for the nurse to implement? A. Perform a heel-stick to monitor blood glucose level. B. Gradually warm the infant under a radiant heat source. C. Administer oxygen by mask at 2L/minute. D. Notify the pediatrician of the infants unstable vital signs.

B. Gradually warm the infant under a radiant heat source. The infant is showing signs of hypothermia. The best intervention is to gradually warm the infant under a radiant heat source. This can help to stabilize the infant's vital signs and prevent further complications.

Which maternal behavior is the nurse most likely to see when a new mother receives her infant for the first time? A. She eagerly reaches for the infant, undresses the infant, and examines the infant completely. B. Her arms and hands receive the infant and she then traces the infant's profile with her fingertips. C. Her arms and hands receive the infant and she then cuddles the infant to her own body. D. She eagerly reaches for the infant and then holds the infant close to her own body.

B. Her arms and hands receive the infant and she then traces the infant's profile with her fingertips. Attachment/bonding theory indicates that most mothers will demonstrate behaviors described in (B) during the first visit with the newborn, which may be at delivery or later. After the first visit, the mother may exhibit greater affection such as eagerly reaching, hugging, etc. (A, C, and D).

A primigravida at 40-weeks gestation is receiving oxytocin (Pitocin) to augment labor. Which adverse effect should the nurse monitor for during the infusion of Pitocin? A. Dehydration. B. Hyperstimulation. C. Galactorrhea. D. Fetal tachycardia.

B. Hyperstimulation. Pitocin causes the uterine myofibril to contract, so unless the infusion is closely monitored, the client is at risk for hyperstimulation (B) which can lead to tetanic contractions, uterine rupture, and fetal distress or demise. Dehydration (A) and galactorrhea (C) are not adverse effects associated with the administration of Pitocin. Fetal tachycardia (D) is an initial response to any stressor, including an increase in maternal temperature or intrauterine infection, but fetal decelerations indicate distress following tetanic contractions.

A nurse is teaching a parent how to care for his newborn's circumcision site. Which of the following client statements indicates an understanding of the teaching? A. I should clean the circumcision site with half-strength hydrogen peroxide twice a day. B. I should apply the diaper loosely until the circumcision site is healed. C. I should notify the doctor if yellow discharge forms on the head of the penis. D. Newborns typically do no experience any pain from this procedure.

B. I should apply the diaper loosely until the circumcision site is healed. This prevents pressure on the circumcision site, thereby decreasing pain and irritation.

A primigravida, when returning for the results of her multiple marker screening (triple screen), asks the nurse how problems with her baby can be detected by the test. What information will the nurse give to the client to describe best how the test is interpreted? A. If MSAFP (maternal serum alpha-fetoprotein) and estriol levels are high and the human chorionic gonadotropin (hCG) level is low, results are positive for a possible chromosomal defect. B. If MSAFP and estriol levels are low and the hCG level is high, results are positive for a possible chromosomal defect. C. If MSAFP are within normal limits, there is a guarantee that the baby is free of all structural anomalies. D. If MSAFP, estriol, and hCG are absent in the blood, the results are interpreted as normal findings.

B. If MSAFP and estriol levels are low and the hCG level is high, results are positive for a possible chromosomal defect. Low levels of MSAFP and estriol and elevated levels of hCG found in the maternal blood sample are indications of possible chromosomal defects (B). High levels of MSAFP and estriol in the blood sample after 15 weeks of gestation can indicate a neural tube defect, such as spina bifida and anencephaly, not chromosomal defects (A). One of the limitations of the multiple marker screening is that any defects covered by skin will not be evident in the blood sampling (C). After 15 weeks of gestation, there will be traces of MSAFP, estriol, and hCG in the blood sample (D).

A full term infant is transferred to the nursery from L&D. Which info is most important for the nurse to receive when planning immediate care for the newborn? A. Length of labor and delivery method. B. Infant's condition at birth and treatment received. C. Feeding method chosen by parents. D. History of drugs given to mother during labor.

B. Infant's condition at birth and treatment received. Immediate care is most dependent on the infant's current status (i.e., Apgar scores at 1 and 5 minutes) and any treatment or resuscitation that was indicated. The transitional care nurse needs the information listed in the choices (A, C, and D), but the priority is (B).

A male infant with a 2-day- history of fever and diarrhea is brought to the clinic by his mother who tells the nurse that the child refuses to drink anything. The nurse determines that the child has a weak cry with no tears. Which prescription is more important to implement? A. Provide a bottle of electrolyte solution. B. Infuse normal saline intravenously. C. Administer an antipyretic rectally. D. Apply external cooling blanket.

B. Infuse normal saline intravenously.

A client who is attending antepartum classes asks the nurse why her healthcare provider has prescribed iron tablets. The nurse's response is based on what knowledge? A. Supplementary iron is more efficiently utilized during pregnancy. B. It is difficult to consume 18 mg of additional iron by diet alone. C. Iron absorption is decreased in the GI tract during pregnancy. D. Iron is needed to prevent megaloblastic anemia in the last trimester.

B. It is difficult to consume 18 mg of additional iron by diet alone. Consuming enough iron-containing foods to facilitate adequate fetal storage of iron and to meet the demands of pregnancy is difficult so iron supplements are often recommended. Dietary iron (A) is just as "good" as iron in tablet form. Iron absorption occurs readily during pregnancy, and is not decreased within the GI tract (C). Megaloblastic anemia (D) is caused by folic acid deficiency.

Following a precipitous labor, a postpartum client has a continuous trickling of bright red blood from her vagina. Her uterus is firm and her vital signs are within normal limits. The nurse determines that this sign may indicate which condition? A. Early postpartum hemorrhage. B. Laceration on the cervix. C. Expected course in the fourth stage of labor. D. A full urinary bladder.

B. Laceration on the cervix. Uterus is firm and vital signs are within normal limits.

For women who have a history of sexual abuse, a number of traumatic memories may be triggered during labor. The woman may fight the labor process and react with pain or anger. Alternately she may become a passive player and emotionally absent herself from the process. The nurse is in a unique position of being able to assist the client to associate the sensations of labor with the process of childbirth and not the past abuse. The nurse can implement a number of care measures to help her client view the childbirth experience in a positive manner. Which intervention would be key for the nurse to use while providing care? A. Telling the client to relax and that it won't hurt much. B. Limiting the number of procedures that invade her body. C. Reassuring the client that as the nurse you know what is best. D. Allowing unlimited care providers to be with the client.

B. Limiting the number of procedures that invade her body. The number of invasive procedures such as vaginal examinations, internal monitoring, and intravenous therapy should be limited as much as possible. The nurse should always avoid words and phrases that may result in the client's recalling the phrases of her abuser (e.g., "Relax, this won't hurt" or "Just open your legs.") The woman's sense of control should be maintained at all times. The nurse should explain procedures at the client's pace and wait for permission to proceed. Protecting the client's environment by providing privacy and limiting the number of staff who observe the client will help to make her feel safe.

During a prenatal visit, the nurse discusses with a client the effects of smoking on the fetus. When compared with nonsmokers, mothers who smoke during pregnancy tend to produce infants who have: A. Lower Apgar scores. B. Lower birth weights. C. Respiratory distress. D. A higher rate of congenital anomalies.

B. Lower birth weights. Smoking is associated with low-birth-weight infants. Mothers are encouraged not to smoke during pregnancy. To date, significant relationships have not been found between smoking and options (A, C, or D).

The nurse is planning care for a newborn of a mother with diabetes mellitus. What is the priority nursing consideration for this newborn? A. Developmental delays because of excessive size. B. Maintaining safety because of low blood glucose levels. C. Choking because of impaired suck and swallow reflexes. D. Elevated body temperature because of excess fat and glycogen.

B. Maintaining safety because of low blood glucose levels. The newborn of a diabetic mother is at risk for hypoglycemia because they have been exposed to high levels of glucose during fetal development and their bodies may overproduce insulin after birth in response to the sudden drop in glucose levels so maintaining safety because of low blood glucose levels would be a priority.

A pregnant client tells the nurse that the first day of her last menstrual period was August 2, 2006. Based on Nägele's rule, what is the estimated date of delivery? A. April 25, 2007. B. May 9, 2007. C. May 29, 2007. D. June 2, 2007.

B. May 9, 2007. Since this woman's first day of her last normal menstrual period occurred on August 2, 2006, the estimated date of delivery is May 9, 2007 (B). Nägele's rule is used to calculate the expected date of delivery, and is obtained by subtracting 3 months and adding 7 days beginning from the first day of the last normal menstrual period. (A, C, and D) are incorrect calculations.

A nurse is assessing a client who is 3 days postpartum. When examining the client's uterus, which of the following techniques should the nurse use? A. Press down and forward with the hand that is placed on the base of the uterus. B. Measure the height of the fundus in finger-breaths in relation to the umbilicus. C. Place the client in a semi-fowlers position prior to checking the uterus. D. place the client in a supine position prior to checking the uterus.

B. Measure the height of the fundus in finger-breaths in relation to the umbilicus. The nurse should measure the height of the fundus in finger breaths and should expect the height to decrease 1 finger breath in height daily after birth. The fundus should be about 3 finger-breaths below the umbilicus by the third day postpartum. A deviation from the expected measurement could indicate a potential complication, such as retained placental fragments, uterine atony, or excessive bleeding.

A 36 week primigravida is admitted to labor and delivery with severe abdominal pain and bright red vaginal bleeding. Her abdomen is rigid and tender to touch. The fetal heart rate is 90 beats/min, and the maternal heart rate is 120 beats/min. What action should the nurse implement first? A. Alert the neonatal team and prepare for neonatal resuscitation. B. Notify the healthcare provider from the client's bedside. C. Obtain written consent for an emergency cesarean section. D. Draw a blood sample for stat hemoglobin and hematocrit.

B. Notify the healthcare provider from the client's bedside.

The nurse identifies crepitus when examining the chest of a newborn who was delivered vaginally. Which further assessment should the nurse perform? A. Elicit a positive scarf sign on the affected side. B. Observe for an asymmetrical Moro (startle) reflex. C. Watch for swelling of fingers on the affected side. D. Note paralysis of affected extremity and muscles.

B. Observe for an asymmetrical Moro (startle) reflex. The most common neonatal birth trauma due to a vaginal delivery is fracture of the clavicle. Although an infant may be asymptomatic, a fractured clavicle should be suspected if an infant has limited use of the affected arm, malposition of the arm, anasymmetric Moro reflex (B), crepitus over the clavicle, focal swelling or tenderness, or cries when the arm is moved. Eliciting (A) (extending arm across the chest toward the opposite shoulder) is contraindicated if a fractured clavicle is present. (C and D) on the affected side require follow-up, but are not indicative of a fractured clavicle.

The nurse observes a new mother avoiding eye contact with her newborn. Which action should the nurse take? A. Ask the mother why she won't look at the infant. B. Observe the mother for other attachment behaviors. C. Examine the newborn's eyes for the ability to focus. D. Recognize this as a common reaction in new mothers.

B. Observe the mother for other attachment behaviors. Parent-infant bonding or attachment is based on a mutual relationship between parent and infant and is commonly established by the "enface position," which is demonstrated by the mother's and infant's eyes meeting in the same plane. To assess for other attachment behaviors, continued observation of the new mother's interactions with her infant helps the nurse determine problems in attachment. (A) may cause undue confusion, stress, or impact the mother's self-confidence. (C) is not indicated. The "enface position" is a significant, early behavior that leads to the formation of affectional ties and should be encouraged (D).

A client who is 24-weeks gestation arrives to the clinic reporting swollen hands. On examination the nurse notes the clients as had a rapid weight gain over six weeks. Which action should the nurse implement next? A. Review previous blood pressures in the chart. B. Obtain the client's blood pressure. C. Observe and time the client's contractions. D. Examine the client for pedal edema.

B. Obtain the client's blood pressure. Sudden weight gain and swelling in the hands can be signs of preeclampsia.

The nurse is assessing a patient who is 36 hours post delivery. Which finding should the nurse report to the healthcare provider? A. White blood cell count 19,000. B. Oral temperature of 100.6. C. Fundus deviated to the right. D. Breast are firm when palpated.

B. Oral temperature of 100.6. A temperature of 100.4 degrees Fahrenheit (38 degrees Celsius) or higher is considered a fever for a pregnant woman. It is important for pregnant women to report any fever to their healthcare provider as it can indicate an underlying infection or other medical issue that may require treatment. The other options are normal or expected findings for a patient 36 hours post delivery.

The nurse in a labor room is performing a vaginal assessment on a pregnant client in labor. The nurse notes the presence of the umbilical cord protruding from the vagina. What is the first nursing action with this finding? A. Gently push the cord into the vagina. B. Place the client in Trendelenburg's position. C. Find the closest telephone and page the health care provider stat. D. Call the delivery room to notify the staff that the client will be transported immediately.

B. Place the client in Trendelenburg's position. When cord prolapse occurs, prompt actions are taken to relieve cord compression and increase fetal oxygenation. The mother should be positioned with her hips higher than her head to shift the fetal presenting part toward the diaphragm.

A nurse is caring for a client who is in labor. The nurse observes late decelerations on the fetal monitor. Which of the following actions should the nurse take? A. Decrease the rate of the client's maintenance IV fluid. B. Place the client in a left lateral position. C. Apply oxygen at 2 L/min via nasal cannula. D. Prepare the client for an amniocentesis.

B. Place the client in a left lateral position. The nurse should identify that decelerations of the fetal heart rate with an onset beginning after a contraction has started that persist beyond the end of the contraction are considered late decelerations. Later decelerations indicate an interruption in fetal oxygenation. A lateral position improves blood glucose to the uterus and intervillous spaces. Repositioning that client is a component of intrauterine resuscitation.

A mother spontaneously delivers a newborn infant in the taxicab while on the way to the hospital the emergency room nurse reported the mother has active herpes (H5V III) lesions on the vulva. Which intervention should the nurse implement first when admitting the neonate to the nursery? A. Documents the temperature on the flow sheet. B. Place the newborn in the isolation area of the nursery. C. Obtain blood specimen for serum glucose level. D. Administer the vitamin K injection.

B. Place the newborn in the isolation area of the nursery.

The nurse has been working with a laboring client and notes that she has been pushing effectively for 1 hour. What is the client's primary physiological need at this time? A. Ambulation. B. Rest between contractions. C. Change positions frequently. D. Consume oral food and fluids.

B. Rest between contractions. Pushing requires a significant amount of energy and effort, and the client may become fatigued. Resting between contractions allows the client to conserve energy and regain strength for the next contraction.

A nurse is planning care for a client who is postpartum. Which of the following strategies should the nurse include in the plan to prevent bladder distention? A. Withhold analgesics to prevent urinary retention. B. Run water in the sink while the client sits on the toilet. C. Perform Crede's maneuver every 4 hours. D. Restrict oral hydration.

B. Run water in the sink while the client sits on the toilet. Running water in the sink, placing the client's hand in warm water, and using a squeeze bottle to run water over the client's perineum can assist with spontaneous voiding.

The nurse should explain to a 30-year-old gravid client that alpha fetoprotein testing is recommended for which purpose? A. Detect cardiovascular disorders. B. Screen for neural tube defects. C. Monitor the placental functioning. D. Assess for maternal pre-eclampsia.

B. Screen for neural tube defects Alpha-fetoprotein (AFP) is a screening test used in pregnancy. Elevated AFP may indicate an increased risk of neural tube defects (B) such as anencephaly and spinal bifida. AFP does not apply in (A, C, or D).

A nurse receives a shift change report for a newborn who is 12 hours post-vaginal delivery. In developing a plan of care the nurse should give the highest priority to which finding? A. Cyanosis of the hands and feet. B. Skin color that is slightly jaundiced. C. Tiny white papules on the nose or chin. D. Red patches on the cheeks and trunk.

B. Skin color that is slightly jaundiced. Jaundice, a yellow skin coloration, is caused by elevated levels of bilirubin, which should be further evaluated in a newborn less than 24 hours old (B). Acrocyanosis (blue color of the hands and feet) is a common finding in newborns; it occurs because the capillary system is immature (A). Milia (C) are small white papules present on the nose and chin that are caused by sebaceous gland blockage, which disappear in a few weeks. Small red patches on the cheeks and trunk (D) are called erythema toxicum neonatorum, a common finding in newborns.

A nurse is speaking with a client who is addicted to heroin and who just learned that she is pregnant. The client states, "I just started taking methadone. Is there anything else I can do to make sure my baby is healthy?" Which information should the nurse provide? A. Discontinue the methadone right away. B. Start a prenatal care plan as soon as possible. C. Sign up for group therapy sessions. D. Describe genetic testing protocols.

B. Start a prenatal care plan as soon as possible. Starting a prenatal care plan as soon as possible is essential for ensuring the health of both the mother and the baby.

The nurse expects to administer an oxytocic (e.g., Pitocin, Methergine) to a woman after expulsion of her placenta to: A. Relieve pain. B. Stimulate uterine contractions. C. Prevent infection. D. Facilitate rest and relaxation.

B. Stimulate uterine contractions. Oxytocics stimulate uterine contractions, which reduce blood loss after the third stage of labor. Oxytocics are not used to treat pain or prevent infection. They cause the uterus to contract, which reduces blood loss. Oxytocics do not facilitate rest and relaxation.

The nurse is performing an assessment on a client who has just been told that a pregnancy test is positive. Which assessment finding indicates that the client is at risk for preterm labor? A. The client is a 35-year-old primigravida. B. The client has a history of cardiac disease. C. The client's hemoglobin level is 13.5 g/dL (135 mmol/L). D. The client is a 20-year-old primigravida of average weight and height.

B. The client has a history of cardiac disease. A history of cardiac disease is a risk factor for preterm labor because it can lead to maternal stress, reduced oxygen delivery to the fetus, and increased risk of complications such as preeclampsia, gestational diabetes, and premature rupture of membranes. Additionally, some cardiac medications may increase the risk of preterm labor. It is important to receive regular prenatal care and monitoring to reduce the risk of preterm labor and other complications. Preterm labor occurs after the 20th week but before the 37th week of gestation. Several factors are associated with preterm labor, including a history of medical conditions, present and past obstetric problems, social and environmental factors, and substance abuse. Other risk factors include a multifetal pregnancy, which contributes to overdistention of the uterus; anemia, which decreases oxygen supply to the uterus; and age younger than 18 years or first pregnancy at age older than 40 years.

The nurse is preparing to administer oxytocin intravenously (20 units of oxytocin in 1,000 mL Lactated Ringer's) to a client after the delivery of her infant. What is the purpose of administering oxytocin at this time? A. To stimulate the let-down reflex. B. To stimulate uterine contractions. C. To return the uterus to its prepregnancy size. D. To aid in the expulsion of the placenta.

B. To stimulate uterine contractions. The purpose of administering oxytocin intravenously after the delivery of the infant is to stimulate uterine contractions and prevent postpartum hemorrhage. While oxytocin can help to expel the placenta by stimulating uterine contractions, the expulsion of the placenta is a natural process that occurs as the uterus continues to contract after the delivery of the infant and is not the primary purpose of administering oxytocin intravenously after the delivery of the infant.

The nurse is caring for a 35-week gestation infant delivered by cesarean section 2 hours ago. The nurse observes the infant's respiratory rate is 72 breaths/minute with nasal flaring, grunting, and retractions. The nurse should recognize these findings indicate which complication? A. Persistent pulmonary hypertension of the newborn. B. Transient tachypnea of the newborn. C. Meconium aspiration syndrome. D. Bronchopulmonary dysplasia.

B. Transient tachypnea of the newborn. (TTN)

After breast-feeding 10 minutes at each breast, a new mother calls the nurse to the postpartum room to help change the newborns diaper. As the mother begins the diaper change, the newborn spits up the breast milk. What action should the nurse implement first? A. Wipe away the spit-up and assist the mother with the diaper change. B. Turn the newborn to the side and bulb suction the mouth and nares. C. Sit the newborn up and burp by rubbing or patting the upper back. D. Place the newborn in a position with the head lower than the feet.

B. Turn the newborn to the side and bulb suction the mouth and nares. - HESI Turning the newborn to the side and bulb suctioning the mouth and nares is the priority intervention to prevent aspiration of the spit-up. Sitting the newborn up and burping is an appropriate intervention to prevent further episodes of spitting up, but does not address the immediate risk of aspiration.

A nurse is assessing a client who is receiving magnesium sulfate as a treatment for pre-eclampsia. Which of the following clinical findings is the nurse's priority? A. Respirations 16/min. B. Urinary output 40 mL in 2 hr. C. Reflexes +2. D. Fetal heart rate 158/min.

B. Urinary output 40 mL in 2 hr. Urinary output is critical to the excretion of magnesium from the body. The nurse should discontinue the magnesium sulfate if the hourly output is <30mL/hr.

Vaginal examinations should be performed by the nurse under all of these circumstances EXCEPT: A. An admission to the hospital at the start of labor. B. When accelerations of the fetal heart rate (FHR) are noted. C. On maternal perception of perineal pressure or the urge to bear down. D. When membranes rupture.

B. When accelerations of the fetal heart rate (FHR) are noted. An accelerated FHR is a positive sign; however, variable decelerations merit a vaginal examination. Vaginal examinations should be performed when the woman is admitted, when she perceives perineal pressure or the urge to bear down, when her membranes rupture, when a significant change in her uterine activity has occurred, or when variable decelerations of the FHR are noted.

A new mother asks the nurse "How do I know that my daughter is getting enough breast milk?" Which explanation will the nurse provide? A. Weigh the baby daily, and if she is gaining weight, she is eating enough. B. Your milk is sufficient if the baby is voiding pale straw-colored urine 6-10 times a day. C. Offer the baby extra bottle milk after her feeding, and see if she is still hungry. D. If you're concerned, you might consider bottle feeding so that you can monitor her intake.

B. Your milk is sufficient if the baby is voiding pale straw-colored urine 6-10 times a day. The urine will be dilute (straw-colored) and frequent (>6 to 10 times/day) (B), if the infant is adequately hydrated. Although a weight gain (A) of 30 grams/day is indicative of adequate nutrition, most home scales do not measure this accurately and this suggestion is likely to make the mother very anxious! (C) causes nipple confusion and diminishes the mother's milk production. (D) does not answer the client's question.

A pregnant client presents to the antepartal clinic complaining of brownish vaginal bleeding. The nurse notes that she has a greatly enlarged uterus and is complaining of severe nausea. The client reports that her last menstrual period was "about 2 and a half months ago." Vital signs are: temperature 98.7°F, (37°C) pulse rate 70 beats/minute, respiratory rate 18 breaths/minute, and blood pressure 190/110 mmHg. Based on these findings, which laboratory value should the nurse review? A. Vaginal secretions culture. B. hCG values. C. Hematocrit. D. Glucose in the urine.

B. hCG values. The presence of brownish vaginal bleeding, greatly enlarged uterus, and severe nausea are symptoms of a molar pregnancy, which is an abnormal pregnancy characterized by the growth of a nonviable fertilized egg that forms a mass of cysts. In a molar pregnancy, hCG levels are much higher than normal for the gestational age and rise more rapidly than in a normal pregnancy.

A postpartal client complains that she has the urge to urinate every hour but is only able to void a small amount. What interventions provides the nurse with the most useful information? A. Initiate a perineal pad count. B. Catheterize for residual urine after next voiding. C. Assess for perineal hematoma. D. Determine the clients usual voiding pattern.

B?

A 4-week-old premature infant has been receiving epoetin alfa (Epogen) for the last three weeks. Which assessment finding indicates to the nurse that the drug is effective? A) Slowly increasing urinary output over the last week. B) Respiratory rate changes from the 40s to the 60s. C) Changes in apical heart rate from the 180s to the 140s. D) Change in indirect bilirubin from 12 mg/dl to 8 mg/dl.

C) Changes in apical heart rate from the 180s to the 140s. Epogen, given to prevent or treat anemia, stimulates erythropoietin production, resulting in an increase in RBCs. Since the body has not had to compensate for anemia with an increased heart rate, changes in heart rate from high to normal (C) is one indicator that Epogen is effective. (A) is not related to Epogen administration. Respiratory rate should decrease rather than increase (B) with Epogen administration. (D) is usually related to resolution of hyperbilirubinemia, treated with phototherapy or increased oral intake in the infant.

A 24-hour-old newborn has a pink papular rash with vesicles superimposed on the thorax, back, and abdomen. What action should the nurse implement? A) Notify the healthcare provider. B) Move the newborn to an isolation nursery. C) Document the finding in the infant's record. D) Obtain a culture of the vesicles.

C) Document the finding in the infant's record. Erythema toxicum (or erythema neonatorum) is a newborn rash that is commonly referred to as "flea bites," but is a normal finding that is documented in the infant's record (C), and requires no further action (A, B, and D).

A 26-year-old, gravida 2, para 1 client is admitted to the hospital at 28-weeks gestation in preterm labor. She is given 3 doses of terbutaline sulfate (Brethine) 0.25 mg subcutaneously to stop her labor contractions. The nurse plans to monitor for which primary side effect of terbutaline sulfate? A) Drowsiness and bradycardia. B) Depressed reflexes and increased respirations. C) Tachycardia and a feeling of nervousness. D) A flushed, warm feeling and a dry mouth.

C) Tachycardia and a feeling of nervousness. Terbutaline sulfate (Brethine), a beta-sympathomimetic drug, stimulates beta-adrenergic receptors in the uterine muscle to stop contractions. The beta-adrenergic agonist properties of the drug may cause tachycardia, increased cardiac output, restlessness, headache, and a feeling of "nervousness" (C). Hypotension, hypertension, and/or drowsiness may occur, but tachycardia, not (A), is a primary side effect. (B and D) are side effects of magnesium sulfate.

The nurse is preparing a client with a term pregnancy who is in active labor for an amniotomy. What equipment should the nurse have available at the client's bedside? (Select all that apply.) A. Litmus paper. B. Fetal scalp electrode. C. A sterile glove. D. An amnihook. E. Sterile vaginal speculum. F. Lubricant.

C, D, F C. A sterile glove. D. An amnihook. F. Lubricant. A single sterile glove (C), an amnihook (D), and lubricant (F) are the necessary equipment for performing an amniotomy. Litmus paper (A) is used to assess for the presence of amniotic fluid. A fetal scalp probe (B) is used to assess fetal heart rates. A sterile vaginal speculum (E) is used to visualize the cervix.

A nurse is teaching a client during the client's first prenatal visit. Which of the following instructions should the nurse include? A. "A fetal stethoscope can first detect your baby's heart rate at 22 weeks." B. "After week 16, we can see if your baby is a boy or a girl." C."A Doppler device can detect your baby's heart rate at 12 weeks." D. "You will first feel the baby move at about 8 weeks."

C. "A Doppler device can detect your baby's heart rate at 12 weeks." The nurse should be able to detect the fetal heartbeat with a Doppler device toward the end of the first trimester, often as early as 10 weeks of gestation.

The nurse implements a teaching plan for a pregnant client who is newly diagnosed with gestational diabetes mellitus. Which statement made by the client indicates a need for further teaching? A. "I should stay on the diabetic diet." B. "I should perform glucose monitoring at home." C. "I should avoid exercise because of the negative effects on insulin production." D. "I should be aware of any infections and report signs of infection immediately to my health care provider."

C. "I should avoid exercise because of the negative effects on insulin production." Exercise is safe for a client with gestational diabetes mellitus and is helpful in lowering the blood glucose level. Dietary modifications are the mainstay of treatment, and the client is placed on a standard diabetic diet. Many clients are taught to perform blood glucose monitoring. If the client is not performing the blood glucose monitoring at home, it is performed at the clinic or obstetrician's office. Signs of infection need to be reported to the obstetrician. - Elsevier's Comprehensive Review for the NCLEX-RN Examination

A nurse is caring for a newborn who has a prescription for phototherapy. The mother asks why the newborn needs to lay under a special light. Which of the following responses should the nurse make? A. "The light helps your baby maintain his body temperature." B. "The light helps your baby establish a regular sleeping pattern." C. "The light will help lower your baby's bilirubin level." D. "The light will help regulate your baby's blood sugar."

C. "The light will help lower your baby's bilirubin level." Jaundice is caused by the breakdown of red blood cells, which release bilirubin. A newborn's immature liver is unable to filter and excrete the bilirubin efficiently, leading to accumulation of bilirubin in the tissues, The ultraviolet light in phototherapy assists in breaking down the bilirubin so that it can be excreted in the urine and feces.

A nurse is providing teaching to a client who has come to the family-planning clinic requesting an intrauterine device (IUD). Which of the following pieces of information should the nurse provide the client? A. "If you lose weight, you will need to have you IUD refitted." B. "An IUD provides protection from certain sexually transmitted infections." C. "Your risk for ectopic pregnancy increases with an IUD." D. "You shouldn't use an IUD if you want to have children later."

C. "Your risk for ectopic pregnancy increases with an IUD." An IUD is a contraceptive device the provider inserts through the cervix into the uterus. The IUD works by changing the lining of the uterus and fallopian tubes, making fertilization in the uterus more difficult. Consequently, an IUD increases the risk of ectopic pregnancy.

The nurse is using the Silverman-Anderson index to assess an infant with respiratory distress and determines that the infant is demonstrating marked nasal flaring, an audible expiratory grunt, and just visible intercostal and xiphoid retractions. Using this scale, which score should the nurse assign? A. 3. B. 4. C. 5. D. 8.

C. 5. The Silverman-Anderson index is an assessment scale that scores a newborn's respiratory status as grade 0, 1, or 2 for each component; it includes synchrony of the chest and abdomen, retractions, nasal flaring, and expiratory grunt. No respiratory distress is graded 0 and a total of 10 indicates maximum respiratory distress. This infant is demonstrating respiratory distress with maximal effort, so a grade 2 is assigned for marked nasal flaring, grade 2 for an audible expiratory grunting, plus grade 1 for just visible retractions, which is a total score of 5 (C). (A, B, and D) are not accurate.

A client at 32-weeks gestation is hospitalized with severe pregnancy-induced hypertension (PIH), and magnesium sulfate is prescribed to control the symptoms. Which assessment finding indicates the therapeutic drug level has been achieved? A. 4+ reflexes. B. Urinary output of 50 ml per hour. C. A decrease in respiratory rate from 24 to 16. D. A decreased body temperature.

C. A decrease in respiratory rate from 24 to 16. Magnesium sulfate, a CNS depressant, helps prevent seizures. A decreased respiratory rate (C) indicates that the drug is effective. (Respiratory rate below 12 indicates toxic effects.) (A) indicates high CNS irritability. Urinary output must be monitored when administering magnesium sulfate and should be at least 30 ml per hour. (B) indicates that the magnesium sulfate is not at a toxic level, but does not indicate that a therapeutic level has been achieved. (D) is not specifically related to magnesium sulfate. (The therapeutic level of magnesium sulfate for a PIH client is 4.8 to 9.6 mg/dl.)

A newborn, whose mother is HIV positive, is scheduled for follow-up assessments. The nurse knows that the most likely presenting symptom for a pediatric client with AIDS is: A. Shortness of breath. B. Joint pain. C. A persistent cold. D. Organomegaly.

C. A persistent cold. A child with AIDS has a decreased ability to defend the body against these infections and often the presenting symptom of a child with AIDS is a persistent cold (C). (A, B, and D) are symptoms of complications which may occur later in the disease process.

A client at 28-weeks gestation arrives at the labor and delivery unit with a complaint of bright red, painless vaginal bleeding. For which diagnostic procedure should the nurse prepare the client? A. Contraction stress test. B. Internal fetal monitoring. C. Abdominal ultrasound. D. Lecithin-sphingomyelin ratio.

C. Abdominal ultrasound. Bright red, painless vaginal bleeding occurring after 20-weeks gestation can be an indicator of placenta previa, which is confirmed by abdominal ultrasound.

Prior to discharge, what instructions should the nurse give to parents regarding the newborn's umbilical cord care at home? A. Wash the cord frequently with mild soap and water. B. Cover the cord with a sterile dressing. C. Allow the cord to air-dry as much as possible. D. Apply baby lotion after the baby's daily bath.

C. Allow the cord to air-dry as much as possible. Recent studies have indicated that air drying or plain water application may be equal to or more effective than alcohol in the cord healing process (C). (A, B, and D) are incorrect because they promote moisture and increase the potential for infection.

The nurse is providing instructions to a pregnant client who is scheduled for an amniocentesis. What instruction should the nurse provide? A. Strict bed rest is required after the procedure. B. Hospitalization is necessary for 24 hours after the procedure. C. An informed consent needs to be signed before the procedure. D. A fever is expected after the procedure because of the trauma to the abdomen.

C. An informed consent needs to be signed before the procedure. An amniocentesis is an invasive diagnostic test that involves the insertion of a needle through the abdominal wall into the uterus to obtain a sample of the amniotic fluid. Like any invasive procedure, it carries certain risks, such as infection, bleeding, and injury to the fetus or surrounding structures. Therefore it is essential to obtain informed consent from the patient, which includes providing information about the risks and benefits of the procedure, as well as alternative options. This allows the patient to make an informed decision about whether or not to proceed with the test. Bed rest is not typically required after an amniocentesis, but the client may be instructed to avoid strenuous activities for the remainder of the day. Hospitalization is generally not necessary after an amniocentesis, unless there are complications that require close monitoring or treatment.

A client in active labor is becoming increasingly fearful because her contractions are occurring more often than she had expected. Her partner is also becoming anxious. Which of the following should be the focus of the nurse's response? A. Telling the client and her partner that the labor process is often unpredictable. B. Informing the client that this means she will give birth sooner than expected. C. Asking the client and her partner if they would like the nurse to stay in the room. D. Affirming that the fetal heart rate is remaining within normal limits.

C. Asking the client and her partner if they would like the nurse to stay in the room. Offering to remain with the client and her partner (C) offers support without providing false reassurance. The length of labor is not always predictable, but (A and B) do not offer the client the support that is needed at this time. (D) may be reassuring regarding the fetal heart rate, but does not provide the client the emotional support she needs at this time during the labor process.

A laboring client's membranes rupture spontaneously. The nurse notices that the amniotic fluid is greenish-brown. What intervention should the nurse implement first? A. Turn the client to her left side. B. Contact the healthcare provider. C. Assess the fetal heart rate. D. Check the cervical dilation.

C. Assess the fetal heart rate.

A nurse is caring for an infant who beings displaying manifestations of neonatal abstinence syndrome (NAS). Which of the following actions should the nurse take? A. Swaddle the infant with arms and legs extended. B. Administer naloxone IM. C. Avoid eye contact during feedings. D. Discourage the mother from handling the infant during the withdrawal phase.

C. Avoid eye contact during feedings The nurse should avoid eye contact and talking during feedings. Infants with NAS have difficulty processing multiple forms of stimulation and can quickly become frustrated. (NAS is a group of conditions caused when a baby withdraws from certain drugs, most commonly opioids, which they're exposed to in the womb before birth.)

The nurse is preparing to administer methylergonovine maleate (Methergine) to a postpartum client. Based on what assessment finding should the nurse withhold the drug? A. Respiratory rate of 22 breaths/min. B. A large amount of lochia rubra. C. Blood pressure 149/90. D. Positive Homan's sign.

C. Blood pressure 149/90. Administering Methergine to a patient with a high blood pressure is contraindicated. While Methergine is used to control postpartum hemorrhage by causing uterine contractions, it can cause hypertension and should therefore not be given to clients with a history of hypertension or who are currently hypertensive.

The nurse is caring for a newborn infant who was recently diagnosed with congenital heart defect. Which assessment finding warrants immediate intervention by the nurse? A. Sweating during feedings. B. Weak peripheral pulse. C. Bluish tinge to the tongue. D. Increased respiratory rate.

C. Bluish tinge to the tongue. A bluish tinge to the tongue can indicate central cyanosis, which is a medical emergency and requires immediate intervention. It can indicate that the newborn is not getting enough oxygenated blood and may require supplemental oxygen or other medical interventions.

The nurse notes on the fetal monitor that laboring client has a variable deceleration. Which action should the nurse implement first? A. Assess cervical dilation. B. Administer oxygen via facemask. C. Change the client's position. D. Turn off the oxytocin infusion.

C. Change the client's position.

While evaluating an external monitor tracing of a woman in active labor, the nurse notes that the fetal heart rate (FHR) for five sequential contractions begins to decelerate late in the contraction, with the nadir of the decelerations occurring after the peak of the contraction. The nurse's first priority is to: A. Notify the care provider. B. Assist with amnioinfusion. C. Change the woman's position. D. Insert a scalp electrode.

C. Change the woman's position. Late decelerations may indicate uteroplacental insufficiency and can be caused by compression of the fetal head, cord compression, or decreased blood flow through the placenta. Changing the woman's position can help alleviate these factors and improve fetal oxygenation.

A newborn infant is brought to the nursery from the birthing suite. The nurse notices that the infant is breathing satisfactorily but appears dusky. What action should the nurse take first? A. Notify the pediatrician immediately. B. Suction the infant's nares, then the oral cavity. C. Check the infant's oxygen saturation rate. D. Position the infant on the right side.

C. Check the infant's oxygen saturation rate. When possible, the nurse should first obtain measurable objective data; an oxygen saturation rate provides such information. The pediatrician should be notified if the oxygen saturation rate is below 90% (A). The infant is not demonstrating signs of an obstructed airway, but if suctioning was required, the oral cavity should be suctioned first to prevent the infant from aspirating pharyngeal secretions (B). (D) facilitates drainage from the mouth and promotes emptying into the small intestine, but at this time, this intervention is not as high a priority as (C).

A mother asks the nurse what to use when changing her newborn's diaper. What substance is best for the nurse to recommend to this mother? A. Talcum powder. B. Baby lotion. C. Clear water. D. Corn starch powder.

C. Clear water. Water is gentle, non-irritating, and effective for cleaning the baby's bottom.

A client at 40-weeks gestation presents to the obstetrical floor and indicates that the amniotic membranes ruptured spontaneously at home. She is in active labor, and feels the need to bear down and push. What information is most important for the nurse to obtain first? A. Estimated amount of fluid. B. Any odor noted when membranes ruptured. C. Color and consistency of fluid. D. Time the membranes ruptured.

C. Color and consistency of fluid. The most important information for the nurse to obtain first in this situation is the color and consistency of the fluid. This is because the presence of meconium-stained fluid, which appears green or brown, may indicate fetal distress and require immediate intervention.

A primigravida arrives at the observation unit of the maternity unit because she thinks she is in labor. The nurse applies the external fetal heart monitor and determines that the fetal heart rate is 140 beats per minute and contraction occurring irregularly every 10 to 15 minutes. Which assessment finding confirms to the nurse that the client is not in labor at this time? A. Membranes are intact. B. 2+ pitting edema in lower extremities. C. Contractions decrease with walking. D. Cervical dilation is 1 centimeter.

C. Contractions decrease with walking.

A mother who is breastfeeding her baby receives instructions from the nurse. Which instruction is most effective to prevent nipple soreness? A. Wear a cotton bra. B. Increase nursing time gradually. C. Correctly place infant on the breast. D. Manually express a small amount of milk before nursing.

C. Correctly place infant on the breast. The most common cause of nipple soreness is incorrect positioning (C) of the infant on the breast, e.g., grasping too little of the areola or grasping only the nipple. (A) helps prevent chafing. (B) is important, but is not necessary for all women. (D) helps soften an engorged breast and encourages correct infant attachment, but is not the BEST answer.

A client with no prenatal care arrives at the labor unit screaming, "The baby is coming!" The nurse performs a vaginal examination that reveals the cervix is 3 centimeters dilated and 75% effaced. What additional information is most important for the nurse to obtain? A. Gravidity and parity. B. Time and amount of last oral intake. C. Date of last normal menstrual period. D. Frequency and intensity of contractions.

C. Date of last normal menstrual period. Evaluating the gestation of the pregnancy (C) takes priority. If the fetus is preterm and the fetal heart pattern is reassuring, the healthcare provider may attempt to prolong the pregnancy and administer corticosteroids to mature the lungs of the fetus. (A, B, and D) are all important to evaluate and incorporate into the plan of care, but establishing gestation takes priority.

The total bilirubin level of a 36-hour, breastfeeding newborn is 14 mg/dL. Based on this finding, which intervention should the nurse implement? A. Provide phototherapy for 30 minutes q8h. B. Feed the newborn sterile water hourly. C. Encourage the mother to breastfeed frequently. D. Assess the newborn's blood glucose level.

C. Encourage the mother to breastfeed frequently. The normal total bilirubin level is 6 to 12 mg/dl after Day 1 of life. This infant's bilirubin is beginning to climb and the infant should be monitored to prevent further complications. Breast milk provides calories and enhances GI motility, which will assist the bowel in eliminating bilirubin. (A) is not indicated at this level. (B) would limit caloric intake, which is essential in preventing jaundice. (D) is not related to bilirubin levels.

A new mother is having trouble breastfeeding her newborn son. The child is making frantic rooting motions and will not grasp the nipple. Which intervention should the nurse implement? A. Encourage frequent use of a pacifier so that the infant becomes accustomed to sucking. B. Hold the infant's head firmly against the breast until he latches onto the nipple. C. Encourage the mother to stop feeding for a few minutes and comfort the infant, and assist the mother to help the baby latch on. D. Provide formula for the infant until he becomes calm, and then offer the breast again.

C. Encourage the mother to stop feeding for a few minutes and comfort the infant, and assist the mother to help the baby latch on. Stopping the feeding for a few minutes and comforting the infant can help calm them and make it easier for them to latch onto the nipple. The nurse can also assist the mother with different breastfeeding positions and techniques to help the infant latch on properly.

A client at 32-weeks gestation is diagnosed with preeclampsia. Which assessment finding is most indicative of an impending convulsion? A. 3+ deep tendon reflexes. B. Periorbital edema. C. Epigastric pain. D. Decreased urine output.

C. Epigastric pain. Epigastric pain (C) is indicative of an edematous liver or pancreas which is an early warning sign of an impending convulsion (eclampsia) and requires immediate attention. (A 4+ reflex in a client with pregnancy-induced hypertension indicates hyperreflexia, which is an indication of an impending seizure.)

A nurse is caring for a client who states, "I think I am pregnant." Which of the following findings should the nurse identify as a positive sign of pregnancy?" A. Positive serum pregnancy test. B. Amenorrhea. C. Fetal heart tones auscultated by Doppler. D. Chadwick signs.

C. Fatal heart tones auscultated by Doppler. Fetal heart tones heard by Doppler are a positive sign of pregnancy. The only possible explanation for hearing fetal heart tones is the presence of a fetus.

The nurse is teaching breastfeeding to prospective parents in a childbirth education class. Which instruction should the nurse include as content in the class? A. Begin as soon as your baby is born to establish a four-hour feeding schedule. B. Resting helps with milk production. Ask that your baby be fed at night in the nursery. C. Feed your baby every 2 to 3 hours or on demand, whichever comes first. D. Do not allow your baby to nurse any longer than the prescribed number of minutes.

C. Feed your baby every 2 to 3 hours or on demand, whichever comes first. Breastfeeding infants should be kept in the room with the mother and fed every 2 to 3 hours or on demand--whichever comes first. (Breastfeeding frequently aids in milk production.)

Which assessment finding following an amniotomy should be conducted first? A. Cervical dilation. B. Bladder distention. C. Fetal heart rate pattern. D. Maternal blood pressure.

C. Fetal heart rate pattern. Fetal heart rate is assessed immediately after amniotomy to detect any changes that may indicate cord compression or prolapse. When the membranes are ruptured, minimal vaginal examinations would be done because of the risk of infection. Bladder distention or maternal blood pressure would not be the first thing to check after an amniotomy.

The nurse is teaching care of the newborn to a group of prospective parents and describes the need for administering antibiotic ointment into the eyes of the newborn. Which infectious organism will this treatment prevent from harming the infant? A. Herpes. B. Staphylococcus. C. Gonorrhea. D. Syphilis.

C. Gonorrhea. Erythromycin ointment is instilled into the lower conjunctiva of each eye within 2 hours after birth to prevent ophthalmia neonatorum and inclusion conjunctivitis. (Erythromycin is less effective against, but may still help prevent chlamydia infections.)

When reading the obstetrical history of a client who is currently at 36-weeks gestation, the nurse notes that she has had one full-term infant, one premature infant born at 32-weeks gestation that survived, and one miscarriage. What is this client's gravidity? A. Gravida 2. B. Gravida 5. C. Gravida 4. D. Gravida 3.

C. Gravida 4. Gravidity refers to the total number of pregnancies a woman has had, including both viable and non-viable pregnancies. In this case, the client has had four pregnancies: - One current pregnancy. - One full-term infant. - One premature infant born at 32 weeks gestation that survived. - One miscarriage.

The nurse instructs a laboring client to use accelerated-blow breathing. The client begins to complain of tingling fingers and dizziness. What action should the nurse take? A. Administer oxygen by face mask. B. Notify the healthcare provider of the client's symptoms. C. Have the client breathe into her cupped hands. D. Check the client's blood pressure and fetal heart rate.

C. Have the client breathe into her cupped hands. Tingling fingers and dizziness are signs of hyperventilation (blowing off too much carbon dioxide). Hyperventilation is treated by retaining carbon dioxide. This can be facilitated by breathing into a paper bag or cupped hands (C). (A) is inappropriate since the CO2 level is low, not O2. (B and D) are not specific for this situation.

One week after missing her menstrual period, a woman performs an OTC pregnancy test and it is positive. Which hormone is responsible for producing the positive result? A. Human placental lactogen. B. Gonadotrophin-releasing hormone. C. Human chorionic gonadotrophin . D. Prostaglandin E2 Aplha.

C. Human chorionic gonadotrophin. Human chorionic gonadotropin (hCG) is a hormone produced by the placenta during pregnancy. The levels of hCG in the blood and urine increase rapidly during the first few weeks of pregnancy.

A client who delivered by cesarean section 24 hours ago is using a patient-controlled analgesia (PCA) pump for pain control. Her oral intake has been ice chips only since surgery. She is now complaining of nausea and bloating, and states that because she had nothing to eat, she is too weak to breastfeed her infant. Which nursing diagnosis has the highest priority? A. Altered nutrition, less than body requirements for lactation. B. Alteration in comfort related to nausea and abdominal distention. C. Impaired bowel motility related to pain medication and immobility. D. Fatigue related to cesarean delivery and physical care demands of infant.

C. Impaired bowel motility related to pain medication and immobility. Impaired bowel motility caused by surgical anesthesia, pain medication, and immobility (C) is the priority nursing diagnosis and addresses the potential problem of a paralytic ileus. (A and B) are both caused by impaired bowel motility. (D) is not as important as impaired motility.

At 14-weeks gestation, a client arrives at the Emergency Center complaining of a dull pain in the right lower quadrant of her abdomen. The nurse obtains a blood sample and initiates an IV. Thirty minutes after admission, the client reports feeling a sharp abdominal pain and a shoulder pain. Assessment findings include diaphoresis, a heart rate of 120 beats/minute, and a blood pressure of 86/48. Which action should the nurse implement next? A. Check the hematocrit results. B. Administer pain medication. C. Increase the rate of IV fluids. D. Monitor client for contractions.

C. Increase the rate of IV fluids. The client is demonstrating symptoms of blood loss, probably the result of an ectopic pregnancy, which occurs at approximately 14-weeks gestation when embryonic growth expands the fallopian tube causing its rupture, and can result in hemorrage and hypovolemic shock. Increasing the IV infusion rate (C) provides intravascular fluid to maintain blood pressure. (A, B, and D) can be implemented after fluid replacement is increased.

At 0600 while admitting a woman for a scheduled repeat c section, the client tells the nurse that she drank a cup of coffee at 0400 because she wanted to avoid getting a headache. What action should the nurse take first? A. Ensure preoperative lab results are available. B. Start prescribed IV with Lactated Ringers. C. Inform the anesthesia care provider. D. Contact the client's obstetrician.

C. Inform the anesthesia care provider. Drinking fluids before a c section increases the risk of aspiration and caffeine may* impact the anesthesia that will be administered.

A nurse is caring for a client in active labor whose fetus is in a persistent occiput posterior position. Which of the following actions should the nurse take to promote rotation of the fetal head? A. Apply counter-pressure to the client's back. B. Place heat on the client's lower back. C. Instruct the client to squat during contractions. D. Encourage the client to ambulate in the hall.

C. Instruct the client to squat during contractions. Measures to encourage rotation of the fetal head to a more anterior position include squatting during contractions, getting on hands and knees during contractions, and lying on the same side as the fetal spine.

An infant born to a heroin addict mother is admitted to the neonatal care unit. What behaviors can the baby exhibit? A. Lethargy and a poor suck. B. Facial abnormalities and microcephaly. C. Irritability and high pitched cry. D. Low birth weight and intrauterine growth retardation.

C. Irritability and high pitched cry. Irritability and a high-pitched cry are common signs of neonatal abstinence syndrome (NAS), which can occur in infants born to mothers who used opioids during pregnancy, including heroin. (A high-pitched cry is thought to be due to the central nervous system hyperirritability and increased muscle tone that can occur in babies experiencing withdrawal.)

The nurse is counseling a woman who wants to become pregnant. The woman tells the nurse that she has a 36-day menstrual cycle and the first day of her last menstrual period was January 8. The nurse correctly calculates that the woman's next fertile period is A. January 14-15. B. January 22-23. C. January 30-31. D. February 6-7.

C. January 30-31. This woman can expect her next period to begin 36 days from the first day of her last menstrual period--the cycle begins at the first day of the cycle and continues to the first day of the next cycle. Her next period would, therefore, begin on February 13. Ovulation occurs 14 days before the first day of the menstrual period. Therefore, ovulation for this woman would occur January 31 (C).

What is the most important assessment for the nurse to conduct following the administration of epidural anesthesia to a client who is at 40-weeks gestation? A. Station of presenting part. B. Level of pain sensation. C. Maternal blood pressure. D. Variability of fetal heart rate.

C. Maternal blood pressure. The most important assessment for the nurse to conduct following the administration of epidural anesthesia is maternal blood pressure. Epidural anesthesia can cause maternal hypotension, which can lead to decreased uterine perfusion, fetal hypoxia, and fetal bradycardia.

A woman who delivered a normal newborn 24 hours ago complains, "I seem to be urinating every hour or so. Is that OK?" Which action should the nurse implement? A. Catheterize the client for residual urine volume. B. Evaluate the normal involution, then massage the fundus. C. Measure the next voiding, then palpate the client's bladder. D. Obtain a specimen for urine culture and sensitivity.

C. Measure the next voiding, then palpate the client's bladder. Frequent urination is expected after childbirth due to the diuretic effects of fluids given during labor and the removal of the pressure on the bladder once the baby is delivered. The nurse should measure the next voiding and palpate the client's bladder to assess for bladder distension or urine retention, which can be a sign of postpartum complications. Catheterization or obtaining a urine culture is not necessary unless there is evidence of urinary retention or infection. Evaluating the normal involution and massaging the fundus is not indicated in this situation as the client is complaining of urinary symptoms, not uterine-related symptoms.

The nurse assess a client admitted to the labor and delivery unit and obtains the following data: dark red vaginal bleeding, uterus slightly tense between contractions, BP 110/68, FHR 110 beats/minutes, cervix 1 cm dilated and uneffaced. Based on these assessment findings, what intervention should the nurse implement? A. Insert an internal fetal monitor. B. Assess for cervical changes q1h. C. Monitor bleeding from IV sites. D. Perform Leopold's maneuvers.

C. Monitor bleeding from IV sites. Monitoring bleeding from peripheral sites (C) is the priority intervention. This client is presenting with signs of placental abruption. Disseminated intravascular coagulation (DIC) is a complication of placental abruptio, characterized by abnormal bleeding. Invasive vaginal procedures (A and B) or (D) can increase the abruption and bleeding, so these interventions are contraindicated.

A 38-week primigravida who works as a secretary and sits at a computer for 8 hours each day tells the nurse that her feet have begun to swell. Which instruction would be most effective in preventing pooling of blood in the lower extremities? A. Wear support stockings. B. Reduce salt in her diet. C. Move about every hour. D. Avoid constrictive clothing.

C. Move about every hour. Pooling of blood in the lower extremities results from the enlarged uterus exerting pressure on the pelvic veins. Moving about every hour (C) will straighten out the pelvic veins and increase venous return. (A) increase venous return from varicose veins in the lower extremities, but are little help with swelling. (B) might be helpful with generalized edema (which could be an indication of PIH) but is not specific for edematous lower extremities. (D) does not specifically address venous return in this particular case.

A client at 30-weeks gestation, complaining of pressure over the pubic area, is admitted for observation. She is contracting irregularly and demonstrates underlying uterine irritability. Vaginal examination reveals that her cervix is closed, thick, and high. Based on these data, which intervention should the nurse implement first? A. Provide oral hydration. B. Have a complete blood count (CBC) drawn. C. Obtain a specimen for urine analysis. D. Place the client on strict bedrest.

C. Obtain a specimen for urine analysis. Obtaining a urine analysis (C) should be done first because preterm clients with uterine irritability and contractions are often suffering from a urinary tract infection, and this should be ruled out first. (A) is important, but gathering objective data through a urine analysis has a higher priority. (B) would be indicated if the client's temperature was elevated. (D) is indicated only if the client is in preterm labor which would be determined by vaginal examination.

A client is admitted with the diagnosis of total placenta previa. Which finding is most important for the nurse to report to the healthcare provider immediately? A. Heart rate of 100 beats/minute. B. Variable fetal heart rate. C. Onset of uterine contractions. D. Burning on urination.

C. Onset of uterine contractions. Total (complete) placenta previa involves the placenta covering the entire cervical os (opening). The onset of uterine contractions (C) places the client at risk for dilation and placental separation, which causes painless hemorrhaging. Although (A, B, and D) should be reported, the risk of hemorrhage is the priority.

A client who delivered an infant an hour ago tells the nurse that she feels wet underneath her buttock. The nurse notes that both perineal pads are completely saturated and the client is lying in a 6-inch diameter pool of blood. Which action should the nurse implement next? A. Cleanse the perineum. B. Obtain a blood pressure. C. Palpate the firmness of the fundus. D. Inspect the perineum for lacerations.

C. Palpate the firmness of the fundus. A firm uterus is needed to control bleeding from the placental site of attachment on the uterine wall. The nurse should first assess for firmness (C) and massage the fundus as indicated. (A, B, and D) can be implemented after (C).

The nurse is reviewing the health care provider's (HCP's) prescriptions for a client admitted for premature rupture of the membranes. Gestational age of the fetus is determined to be 37 weeks. Which prescription should the nurse question? A. Monitor fetal heart rate continuously. B. Monitor maternal vital signs frequently. C. Perform a vaginal examination every shift. D. Administer ampicillin 1 g as an intravenous piggyback every 6 hours.

C. Perform a vaginal examination every shift. Vaginal examinations should not only be done routinely on a client with premature rupture of the membranes because of the risk of infection. The nurse would expect to monitor fetal heart rate, monitor maternal vital signs, and administer an antiobiotic.

Following the vaginal delivery of a 10-pound infant, the nurse assesses a new mother's vaginal bleeding and finds that she has saturated two pads 30 minutes and has a boggy uterus. Which action should the nurse implement first? A. Increase oxytocin IV infusion. B. Have the client empty her bladder. C. Perform fundal massage until firm. D. Inspect the perineum for lacerations.

C. Perform fundal massage until firm.

A mother expresses fear about changing the infant's diaper after circumcision. What information should the nurse include in the teaching plan? A. Cleanse the penis with prepackaged diaper wipes every 3 to 4 hours. B. Wash off the yellow exudate on the glans once every day to prevent infection. C. Place petroleum ointment around the glans with each diaper change and cleansing. D. Apply pressure by squeezing the penis with the fingers for 5 minutes if bleeding occurs.

C. Place petroleum ointment around the glans with each diaper change and cleansing. With each diaper change, the glans penis should be washed with warm water to remove any urine or feces and petroleum ointment (C) should be applied to prevent the diaper from sticking to the healing surface. Prepackaged wipes (A) often contain other products that may irritate the site. The yellow exudate, which covers the glans penis as the area heals and epithelializes, is not an infective process and should not be removed (B). If bleeding occurs at home, the client should be instructed to apply gentle pressure (D) to the site of the bleeding with sterile gauze squares and call the health care provider.

Twenty minutes after a continuous epidural anesthetic is administered, a laboring client's blood pressure drops from 120/80 mm Hg to 90/60 mm Hg. Which action should the nurse take immediately? A. Notify the health care provider or anesthesiologist. B. Continue to assess the blood pressure every 5 minutes. C. Place the client in a lateral position. D. Turn off the continuous epidural.

C. Place the client in a lateral position. The nurse should immediately turn the client to a lateral position (C) or place a pillow or wedge under one hip to deflect the uterus. Other immediate interventions include increasing the rate of the main line IV infusion and administering oxygen by face mask. If the blood pressure remains low after these interventions or decreases further, the anesthesiologist or health care provider should be notified immediately (A). To continue to monitor blood pressure without taking further action (B) could constitute malpractice. (D) may also be warranted, but such action is based on hospital protocol.

Six hours after an oxytocin (Pitocin) induction was begun and 2 hours after spontaneous rupture of the membranes, the nurse notes several sudden decreases in the fetal heart rate with quick return to baseline, with and without contractions. Based on this fetal heart rate pattern, which intervention is best for the nurse to implement? A. Turn the client to her side. B. Begin oxygen by nasal cannula at 2 L/min. C. Place the client in a slight Trendelenburg position. D. Assess for cervical dilation.

C. Place the client in a slight Trendelenburg position. The goal is to relieve pressure on the umbilical cord, and placing the client in a slight Trendelenburg position (C) is most likely to relieve that pressure. The FHR pattern is indicative of a variable fetal heart rate deceleration, which is typically caused by cord compression and can occur with or without contractions. (A) may be helpful but is not as likely to relieve the pressure as the Trendelenburg position. (B) is not helpful with cord compression. (D) is not the priority intervention at this time. After repositioning the client, a vaginal examination is indicated to rule out cord prolapse and assess for cervical change.

A nurse is teaching the guardian of a newborn about car seat safety. Which of the following pieces of information should the nurse include? A. Position the child's car seat forward-facing at 1 year of age. B. Place the retainer clip 2 inches above the newborn's umbilicus. C. Place the shoulder harness in the slots that are level with the newborn's shoulders. D. Position the newborn's car seat at a 20 degree angle in the vehicle.

C. Place the shoulder harness in the slots that are level with the newborn's shoulders. The guardian should place the shoulder harness in the slots that are level or slightly below the newborn's shoulders to ensure the child is restrained in the even of an accident.

Twenty minutes after a continuous epidural anesthetic is administered, a laboring client's blood pressure drops from 120/80 to 90/60. What action should the nurse take? A. Notify the healthcare provider or anesthesiologist immediately. B. Continue to assess the blood pressure q5 minutes. C. Place the woman in a lateral position. D. Turn off the continuous epidural

C. Place the woman in a lateral position. The nurse should immediately turn the woman to a lateral position (C), place a pillow or wedge under the right hip to deflect the uterus, increase the rate of the main line IV infusion, and administer oxygen by face mask at 10-12 L/min. If the blood pressure remains low, especially if it further decreases, the anesthesiologist/healthcare provider should be notified immediately (A). Continued assessment of (B), without taking any further action would constitute malpractice. (D) may also be warranted, but such action is based on hospital protocol.

The nurse observes a new mother is rooming-in and caring for her newborn infant. Which observation indicates the need for further teaching? A. Cuddles the baby close to her. B. Rocks and soothes the infant in her arms. C. Places the infant prone in the bassinet. D. Wraps the baby in a warm blanket after bathing.

C. Places the infant prone in the bassinet. The mother should be instructed to avoid placing the infant prone (C) which is associated with an increased incidence of sudden infant death syndrome (SIDS). (A, B, and D) are bonding and nurturing behaviors.

A nurse is caring for a client who is receiving IV oxytocin for the induction of labor and notes repetitive early decelerations on the electronic fetal heart rate (FHR) tracing. Which of the following actions should the nurse take? A. Increase the rate of intravenous fluid infusion. B. Discontinue the infusion of oxytocin. C. Re-evaluate the FHR tracing in 15 minutes. D. Request a prescription for an amnioinfusion.

C. Re-evaluate the FHR tracing in 15 minutes. Early decelerations are a result of the compression of the fetal head during contractions. They are benign and require no specific intervention. The nurse should reassess the FHR and contraction pattern in 15 minutes due to the infusion of oxytocin.

The nurse is assessing a 2-hour-old infant born by cesarean delivery at 39-weeks gestation. Which assessment finding should receive the highest priority when planning this infant's care? A. Blood pressure 76/42 mmHg. B. Faint heart murmur. C. Respiratory rate of 76 breaths/minute. D. Blood glucose 45 mg/dL.

C. Respiratory rate of 76 breaths/minute. A respiratory rate of 76 breaths/minute in a newborn is abnormal and may indicate respiratory distress. The other options are normal and not a cause for concern.

The nurse who is working at a prenatal clinic notes a woman that is at 18 weeks of gestation has two elevated maternal alpha feto-protein (MSAFP) values. What action should the nurse implement? A. Instruct the client to increase intake of folic acid supplements. B. Request a consultation with genetic counselor. C. Schedule a sonogram in the radiology department. D. Send the client to the laboratory for repeat MSAFP.

C. Schedule a sonogram in the radiology department. - HESI

A couple, concerned because the woman has not been able to conceive, is referred to a healthcare provider for a fertility workup and a hysterosalpingography is scheduled. Which post procedure complaint indicates that the fallopian tubes are patent? A. Back pain. B. Abdominal pain. C. Shoulder pain. D. Leg cramps.

C. Shoulder pain. If the tubes are patent (open), pain is referred to the shoulder from a subdiaphragmatic collection of peritoneal dye/gas. (B) could be caused from uterine cramping, but might also be indicative of gas/dye collecting in the uterus due to occluded tubes. Abdominal pain should be further evaluated; it would not be normal after hysterosalpingography. (A and D) are not related to the procedure.

A 23-year-old client who is receiving Medicaid benefits is pregnant with her first child. Based on knowledge of the statistics related to infant mortality, which plan should the nurse implement with this client? A. Refer the client to a social worker to arrange for home care. B. Recommend perinatal care from an obstetrician, not a nurse-midwife. C. Teach the client why keeping prenatal care appointments is important. D. Advise the client that neonatal intensive care may be needed.

C. Teach the client why keeping prenatal care appointments is important. Regular prenatal visits should begin early in pregnancy to monitor health of the mother and development of the fetus (C). Based on the client's information, (A, B, and D) are not indicated.

A new mother who has just had her first baby says to the nurse "I saw the baby in the recovery room. She sure has a funny looking head". Which response by the nurse is best? A. That is not an unusual shaped head, especially for a first baby. B. It may look funny to you, but newborn babies are often born with heads like your baby's. C. That is normal. The head will return to a round shape within 7-10 days. D. Your pelvis was too small, so the baby's head had to adjust to the birth canal.

C. That is normal. The head will return to a round shape within 7-10 days. (C) reassures the mother that this is normal in the newborn and provides correct information regarding the return to a "normal" shape. Although (A) is correct, it implies that the client should "not worry." Any implied or spoken "don't worry" is usually the wrong answer! (B) is condescending and dismissing--the mother is seeking reassurance and information. (D) is a negative statement and implies that molding is the mother's "fault."

The nurse should encourage the laboring client to begin pushing when: A. There is only an anterior or posterior lip of cervix left. B. The client describes the need to have a bowel movement. C. The cervix is completely dilated. D. The cervix is completely effaced.

C. The cervix is completely dilated. Pushing begins with the second stage of labor, i.e., when the cervix is completely dilated at 10 cm (C). If pushing begins before the cervix is completely dilated (A, B, and D), the cervix can become edematous and may never completely dilate, necessitating an operative delivery. Many primigravidas begin active labor 100% and then proceed to dilate.

The nurse is caring for a client in labor. Which assessment finding indicates to the nurse that the client is beginning the second stage of labor? A. The contractions are regular. B. The membranes have ruptured. C. The cervix is dilated completely. D. The client begins to expel clear vaginal fluid.

C. The cervix is dilated completely. The second stage of labor begins when the cervix is dilated completely (10 cm) and ends with birth of the neonate. Options A, B, and D are not specific assessment findings of the second stage of labor and occur in stage 1.

The nurse is assessing the umbilical cord of a newborn. Which finding constitutes a normal finding? A. Two vessels: one artery and one vein. B. Two vessels: two arteries and no veins. C. Three vessels: two arteries and one vein. D. Three vessels: two veins and one artery.

C. Three vessels: two arteries and one vein. The normal umbilical cord contains three vessels--two arteries and one vein (C). Fewer than three vessels correlates with various congenital anomalies, such as cardiac and renal anomalies. (A, B, and D) would constitute abnormal findings.

The nurse is monitoring a client who is receiving oxytocin (Pitocin) to induce labor. Which assessment finding would cause the nurse to immediately discontinue the oxytocin infusion? A. Fatigue. B. Drowsiness. C. Uterine hyperstimulation. D. Early decelerations of the fetal heart rate.

C. Uterine hyperstimulation. Adverse effects associated with administration of the medication are hyperstimulation of uterine contractions. Oxytocin infusion must be stopped when any signs of uterine hyperstimulation are present. Uterine hyperstimulation is defined as contractions that occur more frequently than every 2 minutes or last longer than 90 seconds, or if there are more than 5 contractions in 10 minutes. This can cause decreased blood flow to the placenta, which can lead to fetal distress and compromise. Fatigue and drowsiness are common side effects of Pitocin, but they are not indications for discontinuing the medication. Early decelerations of the fetal heart rate may be a sign of fetal head compression, but it is not an immediate indication for discontinuing the oxytocin infusion unless other signs of fetal distress are present.

When preparing a class on newborn care for expectant parents, what content should the nurse teach concerning the newborn infant born at term gestation? A. Milia are red marks made by forceps and will disappear within 7-10 days. B. Meconium is the first stool and is usually yellow gold in color. C. Vernix is a white cheesy substance predominantly located in the skin folds. D. Pseudostrabismus found in newborns is treated by minor surgery.

C. Vernix is a white cheesy substance predominantly located in the skin folds. (C) is correct. Vernix, found in the folds of the skin, is a characteristic of term infants. (A) is white, pinpoint spots usually found over the nose and chin which represent blockage of the sebaceous glands. (B) is tarry-black. (D) (crossed eyes) is normal at birth but should be corrected if it persists after 6 to 9 months of age. (Pseudostrabismus a condition where the eyes are well aligned (orthotropic) but appear to be misaligned.)

A nurse is caring for a client who is in labor. A vaginal examination reveals the following findings: 2cm, +1, right occiput anterior (ROA). Based on this information, which of the following fetal positions should the nurse document in the medical record? A. Transverse. B. Breech. C. Vertex. D. Mentum.

C. Vertex. ROA describes the relationship of the presenting part of the fetus to the client's pelvis. In this case, the occipital bone is the presenting part and is located anteriorly on the client's right side. Based on the prescription of the fetus, the position is vertex.

A client who delivered a healthy infant 5 days ago calls the clinic nurse and reports that her lochia is getting lighter in color and asks when the flow will stop. How should the nurse respond? A. 2 weeks. B. 10 days. C. When the placental site has healed. D. After the first time ovulation occurs.

C. When the placental site has healed. The placental site in the uterus usually heals (C) in 3 to 6 weeks, and the lochial flow should cease at that time. Between 2 and 6 weeks after childbirth, lochia alba occurs in most women (A). The client is describing lochia serosa, a normal change in the lochial flow (B) between days 3 and 4 after childbirth, which lasts to about day 10. (D) does not give the client the best information because ovulation varies in the postpartum period and is influenced by lactation and hormonal responses as the client's usual menstrual cycle resumes.

When planning care for a laboring client, the nurse identifies the need to withhold solid foods while the client is in labor. What is the most important reason for this nursing intervention? A. Gastric emptying time decreases during labor. B. Nausea occurs from analgesic used during labor. C. An increased risk for aspiration can occur if general analgesic is needed. D. Autonomic nervous system stimulation during labor decreases peristalsis.

C/D

During a routine prenatal vital a client 32-weeks gestation complains of urinary frequency has increased during the day as well at night. The nurse determines the client is having irregular uterine contractions. What should the nurse implement? A. Ask the client if she had sexual intercourse yesterday. B. Determine if she has change in vaginal discharge. C. Collect urine sample from dipstick analysis. D. Obtain a midstream urine specimen for culture.

C/D?

The nurse is assessing a pregnant client who reports that she smokes one pack of cigarettes per day. The client believes she is six months pregnant, but is unsure of the date of her last menstrual period. Which method of assessment provides the best estimate of gestational age? A. Amniocentesis. B. Nagele's rule. C. Fundal height. D. Ultrasonography.

C/D? Fundal height?

A client at 10 week gestation calls the clinic reporting a low-grade fever with moderate cramping and heavy bright red bleeding. which instruction should the nurse provide the client? A. Come in for an immediate evaluation. B. Ibuprofen... C. D.

Come in for an immediate evaluation. - HESI

A pregnant woman comes to the prenatal clinic for an initial visit. In reviewing her childbearing history, the client indicates that she has delivered premature twins, one full-term baby, and has had no abortions. Which GTPAL should the nurse document in this client's record? A) 3-1-2-0-3. B) 4-1-2-0-3. C) 2-1-2-1-2. D) 3-1-1-0-3.

D) 3-1-1-0-3. (D) describes the correct GTPAL. The client has been pregnant 3 times including the current pregnancy (G-3). She had one full-term infant (T-1). She also had a preterm (P-1) twin pregnancy (a multifetal gestation is considered one birth when calculating parity). There were no abortions (A-0), so this client has a total of 3 living children. (A, B, and C) are inaccurate.

The nurse is planning preconception care for a new female client. Which information should the nurse provide the client? A) Discuss various contraceptive methods to use until pregnancy is desired. B) Provide written or verbal information about prenatal care. C) Ask the client about risk factors associated with complications of pregnancy. D) Encourage healthy lifestyles for families desiring pregnancy.

D) Encourage healthy lifestyles for families desiring pregnancy. Planning for pregnancy begins with healthy lifestyles in the family (D) which is an intervention in preconception care that targets an overall goal for a client preparing for pregnancy. Although (A and B) may be useful for the client, preconception care should focus on measures to assist the client in reducing lifestyle variables that may increase the risk for problems in pregnancy. (C) is important, but not the main objective of maintaining healthy living.

A nurse is preparing to massage the fundus of a client who is postpartum and experiencing uterine atony. In what order should the nurse take the following actions when performed a fundal massage? A. Position hand around the top of the client's fundus. B. Rotate the upper hand to massage the client's uterus. C. Place a hand just above the client's symphysis pubis. D. Ask the client to lie on her back with her knees flexed. E. Use slight downward pressure to compress the client's fundus.

D, C, A, B, E D. Ask the client to lie on her back with her knees flexed. C. Place a hand just above the client's symphysis pubis. A. Position hand around the top of the client's fundus. B. Rotate the upper hand to massage the client's uterus. E. Use slight downward pressure to compress the client's fundus.

A nurse is evaluating a client who has just received instructions about breastfeeding. Which of the following statements should the nurse identify as an indication that the client understands how to prevent mastitis? A. "I will wear an underwire bra to provide support when my milk comes in." B. "I will apply petroleum jelly if my nipples become cracked." C. "I will apply warm compresses to my breasts twice a day." D. "I should avoid waiting too long between feedings."

D. "I should avoid waiting too long between feedings." Mastitis is an inflammation or infection of the breast. Risk factors include insufficient emptying of the breasts during breastfeeding, stress, illness, poor nutrition, and fatigue.

A nurse is providing teaching about newborn care to the parent of a newborn. Which of the following statements by the parent indicates an understanding of the teaching? A. "I will wash my baby's umbilical cord stump with antibacterial soap." B. "I will cover my baby with a lightweight blanket during nap time" C. "I will use a cotton-tipped swab to clean my baby's ear canals." D. "I will place a hat on my baby's head prior to going outside."

D. "I will place a hat on my baby's head prior to going outside." The parents should place a hat or bonnet on the newborn's head to protect the scalp, minimize heat loss, and protect against sunburn.

The nurse prepares to administer a vitamin K injection to a newborn, and the mother asks the nurse why her infant needs the injection. What best response should the nurse provide? A. "Your newborn needs vitamin K to develop immunity." B. "The vitamin K will protect your newborn from being jaundiced." C. "Newborns have sterile bowels, and vitamin K promotes the growth of bacteria in the bowel." D. "Newborns are deficient in vitamin K, and this injection prevents your newborn from bleeding."

D. "Newborns are deficient in vitamin K, and this injection prevents your newborn from bleeding." The primary reason for administering vitamin K to newborns is to prevent bleeding. Vitamin K is necessary for the synthesis of clotting factors, which are essential for blood clotting. Newborns have low levels of vitamin K, and their immature liver function places them at increased risk for bleeding. While the promotion of bowel bacteria may be a beneficial side effect of vitamin K administration, it is not the primary reason for administering it to newborns.

A nurse is preparing to administer meperidine hydrochloride to a client who is in labor. Which of the following statements should the nurse make to the client? A. "This medication can cause your blood pressure to rise." B. "This medication can cause dry mouth." C. "This medication can cause you to urinate excessively." D. "This medication can make you sleepy."

D. "This medication can make you sleepy." Meperidine hydrochloride is an opioid analgesic used for moderate to severe pain during labor. It binds to the brain's opioid receptors and alters the client's response to pain. The client should be informed of the possible adverse effects of this medication such as hypotension, confusion, sedation, headaches, respiratory depression, constipation, and urinary retention.

A nurse is preparing a client who is in labor for the insertion of an intrauterine pressure catheter. The client asks why this type of monitoring is needed. Which of the following responses should the nurse make? A. "This type of monitoring is necessary for timing the frequency of your contractions." B. "This type of monitoring in noninvasive, so it is the best way to monitor your labor contractions." C. "This type of monitor allows us to evaluate your baby's heart rate while you are in labor." D. "This type of monitoring will allow us to measure the intensity of your contractions."

D. "This type of monitoring will allow us to measure the intensity of your contractions." Only an intrauterine pressure catheter can monitor the intensity of contractions. (A tocotransducer can monitor the frequency and duration of contractions)

A nurse is teaching a client who is breastfeeding about strategies for preventing mastitis. Which of the following instructions should the nurse include? A. "Take an herbal galactagogue." B. "Gradually increase the time between feedings." C. "Wear an underwire bra." D. "Use your finger to release suction after feeding."

D. "Use your finger to release suction after feeding." Releasing the newborn's grasp on the nipple with a finger before removing the newborn from the breasts help prevent injury to the nipples, which can lead to mastitis.

The nurse is discussing involution with a post-partum client. Which statement best indicates that the client understands the effect of breastfeeding on the resumption of menstrual cycle? A. "My period will most likely return in 6 to 8 months." B. "I should expect my period to return in 6 to 8 weeks." C. "My period started as soon as the baby was born." D. "While I am breastfeeding, my period may be delayed."

D. "While I am breastfeeding, my period may be delayed." For breastfeeding mothers, the return of menstruation is usually delayed, and it may take several months or longer for periods to resume, especially if the mother is exclusively breastfeeding and feeding her baby on demand. For women who are not breastfeeding, the first menstrual period may occur as early as 6-8 weeks after delivery.

A nurse is caring for a client who is at 35 weeks of gestation and is scheduled to undergo an amniocentesis. Which of the following statements should the nurse make? A. "You will have to drink 3-5 *oz glasses of water to fill your bladder." B. "This procedure will not rupture your membranes or cause premature labor." C. "You might feel light pressure during the collection of a blood sample from the baby." D. "You might feel some like discomfort during the procedure."

D. "You might feel some like discomfort during the procedure." During an amniocentesis, the client might feel slight uterine cramping when the needle comes into contact with the uterus. A local anesthetic is applied to the client's skin, so the client should not feel pain when the needle pierces the skin. (In early pregnancy, a full bladder helps move the uterus into a better position for the procedure. In later pregnancy, the bladder should be empty to lower the risk of puncture with the amniocentesis needle.)

When assessing a client who is at 12-weeks gestation, the nurse recommends that she and her husband consider attending childbirth preparation classes. When is the best time for the couple to attend these classes? A. 16 weeks gestation. B. 20 weeks gestation. C. 24 weeks gestation. D. 30 weeks gestation.

D. 30 weeks gestation. (D) is closest to the time parents would be ready for such classes. Learning is facilitated by an interested pupil! The couple is most interested in childbirth toward the end of the pregnancy when they are psychologically ready for the termination of the pregnancy, and the birth of their child is an immediate concern. (A, B, and C) are not the best times during a pregnancy for the couple to attend childbirth education classes--they will have other teaching needs.

The nurse is preparing to give an enema to a laboring client. Which client requires the most caution when carrying out this procedure? A. A gravida 6, para 5 who is 38 years of age and in early labor. B. A 37-week primigravida who presents at 100% effacement, 3 cm cervical dilatation, and a -1 station. C. A gravida 2, para 1 who is at 1 cm cervical dilatation and a 0 station admitted for induction of labor due to post dates. D. A 40-week primigravida who is at 6 cm cervical dilatation and the presenting part is not engaged.

D. A 40-week primigravida who is at 6 cm cervical dilatation and the presenting part is not engaged. When the presenting part is ballottable (D), it is floating out of the pelvis. In such a situation, the cord can descend before the fetus causing a prolapsed cord, which is an emergency situation. (A, B, and C) do not present problems with administration of an enema.

A primigravida client who is 5 cm dilated, 90% effaced, and at 0 station is requesting an epidural for pain relief. Which assessment finding is most important for the nurse to report to the healthcare provider? A. Cervical dilation of 5 cm with 90% effacement. B. White blood cell count of 12,000/mm3. C. Hemoglobin of 12 mg/dl and hematocrit of 38%. D. A platelet count of 67,000/mm3.

D. A platelet count of 67,000/mm3. Thrombocytopenia (low platelet count) should be reported to the healthcare provider because it places the client at risk for bleeding when an epidural is administered. (A, B, and C) are within the normal parameters for a client in active labor and is not contraindicated for the placement of an epidural. (A normal platelet count in adults ranges from 150,000 to 450,000 platelets.)

A woman who is 38-weeks gestation is receiving magnesium sulfate for severe preeclampsia. Which assessment finding warrants immediate intervention by the nurse? A. Dizziness while standing. B. Sinus tachycardia. C. Lower back pain. D. Absent patellar reflexes.

D. Absent patellar reflexes. Absent patellar reflexes are an indication of magnesium toxicity, and the dose of magnesium should be decreased or stopped immediately.

Fetal distress is occurring with a laboring client. As the nurse prepares the client for a cesarean birth, what is the most important nursing action? A. Slow the intravenous flow rate. B. Place the client in a high Fowler's position. C. Continue the oxytocin (Pitocin) drip if infusing. D. Administer oxygen, 8 to 10 L/minute, via face mask.

D. Administer oxygen, 8 to 10 L/minute, via face mask. Oxygen is administered, 8 to 10 L/minute, via face mask to optimize oxygenation of the circulating blood. Intravenous infusion should be increased to increase the maternal blood volume. Oxytocin stimulation of the uterus is discontinued if fetal heart rate patterns change for any reason. The client is placed in the lateral position with her legs raised to increase maternal blood volume and improve fetal perfusion.

A 40-week gestation primigravida client is being induced with an oxytocin (Pitocin) secondary infusion and complains of pain in her lower back. Which intervention should the nurse implement? A. Discontinue the oxytocin (Pitocin) infusion. B. Place the client in a semi-Fowler's position. C. Inform the healthcare provider. D. Apply firm pressure to sacral area.

D. Apply firm pressure to sacral area. The discomfort of back labor can be minimized by the application of firm pressure to the sacral area (D). (A and C) are not indicated at this time. (B) does not help to alleviate the client's pain.

A community health nurse visits a family in which a 16-year-old unmarried daughter is pregnant with her first child and is at 32-weeks gestation. The client tells the nurse that she has been having intermittent back pain since the night before. What is the priority nursing intervention? A. Ask the client's mother to call an ambulance for transport to the hospital immediately. B. Determine what physical activities the client has performed for the past 24 hours. C. Teach the client how to perform pelvic rock exercises and observe for correct feedback. D. Ask the client if she has experienced any recent changes in vaginal discharge.

D. Ask the client if she has experienced any recent changes in vaginal discharge.

The nurse is assessing a client who is having a non-stress test (NST) at 41-weeks gestation. The nurse determines that the client is not having contractions, the fetal heart rate (FHR) baseline is 144 bpm, and no FHR accelerations are occurring. What action should the nurse take? A. Check the client for urinary bladder distention. B. Notify the healthcare provider of the nonreactive results. C. Have the mother stimulate the fetus to move. D. Ask the client if she has felt any fetal movement.

D. Ask the client if she has felt any fetal movement. The client should be asked if she has felt the fetus move. An NST is used to determine fetal well-being, and is often implemented when postmaturity is suspected. A "reactive" NST occurs if the FHR accelerates 15 bpm for 15 seconds in response to the fetus' own movement, and is "nonreactive" if no FHR acceleration occurs in response to fetal movement. The client should empty her bladder before starting the test, but bladder distention does not impede fetal movement (A). The client should be quizzed about fetal movement before determining that the NST is nonreactive (B). If no movement has occurred in the last 20 to 30 minutes, it is likely that the fetus is sleeping--providing the mother with orange juice often wakes the infant, and then the NST should be conducted again.

What should be the primary focus of nursing care in the transitional phase of labor for a client who anticipates an unmedicated delivery? A. Assessing the strength of uterine contractions. B. Re-evaluate the need for medication. C. Remind her to push 3 times with each contraction. D. Assessing her to maintain control.

D. Assessing her to maintain control. - HESI

A nurse is caring for a client who is postpartum and is having difficulty voiding. Which of the following actions should the nurse take first? A. Place the client's hands in warm water. B. Administer an analgesic to the client. C. Pour water from a squeeze bottle over the client's perineum. D. Assist the client to the bathroom.

D. Assist the client to the bathroom. The greatest risk to this client is an injury from a distended bladder; therefore, the first action the nurse should take is to assist the client to the bathroom to encourage spontaneous voiding. If this is unsuccessful, the nurse can try other techniques to promote voiding.

A nurse is providing teaching to a client who is at 8 weeks gestation about manifestations to report to the provider during pregnancy. Which of the following pieces of information should the nurse include in the teaching? A. Nausea upon awakening. B. Leg cramps while sleeping. C. Increased white vaginal discharge. D. Blurred double vision.

D. Blurred double vision A client who is pregnant should report experiencing blurred or double vision, as these could be a manifestation of gestational hypertension or preeclampsia.

One hour after giving birth to an 8-pound infant, a client's lochia rubra has increased from small to large and her fundus is boggy despite massage. The client's pulse is 84 beats/minute and blood pressure is 156/96. The healthcare provider prescribes Methergine 0.2 mg IM × 1. What action should the nurse take immediately? A. Give the medication as prescribed and monitor for efficacy. B. Encourage the client to breastfeed rather than bottle feed. C. Have the client empty her bladder and massage the fundus. D. Call the healthcare provider to question the prescription.

D. Call the healthcare provider to question the prescription. Methergine is contraindicated for clients with elevated blood pressure, so the nurse should contact the healthcare provider and question the prescription (D). (A) compromises patient safety. While (B) releases endogenous oxytocin, and (C) promotes uterine contraction, questioning the administration of Methergine is a higher priority because it concerns medication safety.

Which assessment finding should the nursery nurse report to the pediatric healthcare provider? A. Blood glucose level of 45 mg/dl. B. Blood pressure of 82/45 mmHg. C. Non-bulging anterior fontanel. D. Central cyanosis when crying.

D. Central cyanosis when crying. An infant who demonstrates central cyanosis when crying is manifesting poor adaptation to extrauterine life which should be reported to the healthcare provider for determination of a possible underlying cardiovascular problem. (A, B, and C) are expected findings.

An oxytocin induction was started for a gravid client 6 hours ago. When assessing the fetal heart rate on the electronic fetal monitor, the nurse notes a "U-shaped" pattern occurring with the peak of the contraction. Which intervention should the nurse implement first? A. Notify the healthcare provider of the fetal heart rate pattern. B. Begin oxygen administration at 8 L/minute via face mask. C. Discontinue the oxytocin infusion. D. Change the position of the client.

D. Change the position of the client. Changing the position of the client can help to alleviate umbilical cord compression and improve fetal oxygenation. While notifying the healthcare provider of the fetal heart rate pattern (A) is important, changing the client's position is the priority intervention. Oxygen administration via face mask (B) and discontinuing the oxytocin infusion (C) may be necessary if the variable decelerations persist or worsen, but they are not the priority intervention at this time. (A "U-shaped" pattern in the fetal heart rate (FHR) during a contraction is known as a variable deceleration. It occurs when there is a temporary decrease in fetal heart rate caused by umbilical cord compression. This can be caused by a variety of factors, including a nuchal cord (cord around the fetal neck), a prolapsed cord, or a compressed cord due to the position of the fetus.)

What action should the nurse implement to decrease the client's risk for hemorrhage after a cesarean section? A. Monitor urinary output via an indwelling catheter. B. Assess the abdominal dressings for drainage. C. Give the Ringer's Lactated infusion at 125 ml/hr. D. Check the firmness of the uterus every 15 minutes.

D. Check the firmness of the uterus every 15 minutes. A client's risk of postpartal hemorrhage is deceased when the uterus is firm after delivery of the infant. Assessment of fundus consistency q15 minutes (D) provides frequent intervals to stimulate the fundus to contract and prevent bleeding. (A, B, and C) are interventions that do not decrease a postpartal client's risk for hemorrhage.

The nurse is evaluating a full-term multigravida who was induced 3 hours ago. The nurse determines that the client is dilated 7 cm and is 100% effaced at 0 station, with intact membranes. The monitor indicates that the FHR decelerates at the onset of several contractions and returns to baseline before each contraction ends. Which action should the nurse take? A. Reapply the external transducer. B. Insert intrauterine pressure catheter. C. Discontinue the oxytocin infusion. D. Continue to monitor labor progress.

D. Continue to monitor labor progress. The fetal heart rate indicates early decelerations, which are not an ominous sign, so the nurse should continue to monitor the labor progress (D) and document the findings in the client's record. There is no reason to reapply the external transducer (A) if the FHR tracings are being captured. (B and C) are not indicated at this time.

A client is receiving oxytocin (Pitocin) to augment early labor. Which assessment is most important for the nurse to obtain each time the infusion rate is increased? A. Pain level. B. Blood pressure. C. Infusion site. D. Contraction pattern.

D. Contraction pattern.

An infant is placed in a radiant warmer immediately after birth. At one hour of age, the nurse finds the infant to be jittery, tachypneic, and hypotonic. What is the first action that the nurse should take? A. Notify the health care provider immediately. B. Increase the temperature of the radiant warmer. C. Assess infant heart rate. D. Determine the infants blood sugar level.

D. Determine the infants blood sugar level.

A nurse is teaching a female client about nutrition during pregnancy. Which of the following instructions should the nurse include in the teaching? A. Plan to double normal caloric intake during the last trimester of pregnancy. B. Expect to gain 10 to 15 lb during pregnancy. C. Restrict your intake of sodium throughout pregnancy. D. Do not eat swordfish, shark, or king mackerel while you are pregnant.

D. Do not eat swordfish, shark, or king mackerel while you are pregnant. The nurse should instruct the client to avoid certain types of fish, including swordfish, shark, and king mackerel, due to their high levels of mercury, which can harm the developing fetus. The client's caloric intake should increase by approximately 300 calories per day during the third trimester, not double. The client should expect to gain between 25 and 35 pounds during pregnancy, not 10 to 15. Sodium intake should be monitored, but not necessarily restricted, during pregnancy.

A client at 32-weeks gestation comes to the prenatal clinic with complaints of pedal edema, dyspnea, fatigue, and a moist cough. Which question is most important for the nurse to ask this client? A. Which symptom did you experience first? B. Are you eating large amounts of salty foods? C. Have you visited a foreign country recently? D. Do you have a history of rheumatic fever?

D. Do you have a history of rheumatic fever? Clients with a history of rheumatic fever (D) may develop mitral valve prolapse, which increases the risk for cardiac decompensation due to the increased blood volume that occurs during pregnancy, so obtaining information about this client's health history is a priority. (A) is not important. Salty foods (B) sometimes cause edema, but this client is experiencing additional cardiac symptoms. (C) assesses for possible exposure to microorganisms, but these symptoms are more indicative of a cardiovascular etiology.

The nurse is caring for a client in labor and is monitoring the fetal heart rate patterns. The nurse notes the presence of episodic accelerations on the electronic fetal monitor tracing. Which action is most appropriate? A. Notify the health care provider of the findings. B. Reposition the mother and check the monitor for changes in the fetal tracing. C. Take the mother's vital signs and tell the mother that bed rest is required to conserve oxygen. D. Document the findings and tell the mother that the pattern on the monitor indicates fetal well-being.

D. Document the findings and tell the mother that the pattern on the monitor indicates fetal well-being. Accelerations are transient increases in the fetal heart rate that often accompany contractions or are caused by fetal movement. Episodic accelerations are thought to be a sign of fetal well-being and adequate oxygen reserve. Options A, B, and C are inaccurate nursing actions and are unnecessary.

A nurse is caring for a newborn immediately following birth. Which of the following actions should the nurse take first? A. Weigh the newborn. B. Instill erythromycin ophthalmic ointment in the newborn's eyes. C. Administer vitamin K to the newborn. D. Dry the newborn.

D. Dry the newborn. The greatest risk to the newborn immediately after birth is heat loss, which can cause cold stress, respiratory distress, and hypoglycemia. Therefore, the first action the nurse should take is to dry the newborn to prevent heat loss from evaporation.

A nurse is providing teaching to the parents of a newborn about how to care for his circumcision at home. Which of the following instructions should the nurse include in the teaching? A. Apply the diaper tightly over the circumcision area. B. Remove the yellow exudate with each diaper change. C. Use prepackaged commercial wipes to clean the circumcision site. D. Encourage non-nutritive sucking for pain relief.

D. Encourage non-nutritive sucking for pain relief. Allowing the newborn to suck on a pacifier is an effective form of non pharmacological pain management.

The nurse is preparing a young couple and their 24-hour-old infant for discharge from the hospital. In conducting discharge teaching, which intervention is most important for the nurse to implement? A. Ensure that they have the pediatric clinic's phone number. B. Provide the results of the infant's hearing test to the parents. C. Request a return demonstration of a diaper change. D. Evaluate infant feeding techniques prior to discharge.

D. Evaluate infant feeding techniques prior to discharge.

A nurse is caring for a client who is in labor. Which of the following methods will determine the frequency of the client's contractions? A. Palpating the firmness of the uterus during a contraction. B. Calculating the time from the end of each contraction to the beginning of the next. C. Measuring the time from the beginning of a contraction to the end of that same contraction. D. Evaluating the time from the beginning of a contraction to the beginning of the next contraction.

D. Evaluating the time from the beginning of a contraction to the beginning of the next contraction. The method for timing contractions is to measure the time from the beginning of tone contraction to the beginning of the next. That time interval is the frequency of contractions at any given point in

The nurse is counseling a client who is at 6 weeks gestation and is experiencing morning sickness but does not want to take any drugs for this discomfort. Which herbal supplement is likely to help this client with the nausea she is experiencing? A. Ginko. B. Chamomile. C. Peppermint. D. Ginger.

D. Ginger. Ginger is a natural remedy that has been shown to be safe and effective in relieving nausea and vomiting during pregnancy.

A mother who is HIV-positive delivers a full-term newborn and asks the nurse if her baby will become HIV-infected. Which explanation should the nurse provide? A. Most infants of HIV-positive women will continue to test positive for HIV antibodies. B. Infants who have HIV-positive mothers carry the virus and will eventually develop the disease. C. Medication taken during pregnancy to reduce the mother's viral load ensures that the infant is HIV-negative. D. HIV infection is determined at 18 months of age, when maternal HIV antibodies are no longer present.

D. HIV infection is determined at 18 months of age, when maternal HIV antibodies are no longer present. All newborns of HIV-positive mothers receive passive HIV antibodies from the mother, so the evaluation of an infant for the HIV virus is determined at 18 months of age, when all the maternal antibodies are no longer in the infant's blood (D). Passive HIV antibodies disappear in the infant within 18 months of age (A). (B) is inaccurate. Although administration of HIV medication during pregnancy (C) can significantly reduce the risk of vertical transmission, treatment does not ensure that the virus will not become manifest in the infant.

Client tells the nurse that she thinks she's pregnant. Which signs or symptoms provide the best indication that the client is pregnant? A. Morning sickness. B. Breast tenderness. C. Amenorrhea. D. Hegar's sign.

D. Hegar's sign Hegar's sign is a softening and compressibility of the lower uterine segment or the isthmus of the uterus, which is a sign of pregnancy. - HESI

The nurse is caring for a newborn who is 18 inches long, weighs 4 pounds, 14 ounces, has a head circumference of 13 inches, and a chest circumference of 10 inches. Based on these physical findings, assessment for which condition has the highest priority? A. Hyperthermia B. Hyperbilirubinemia C. Polycythemia D. Hypoglycemia

D. Hypoglycemia

A nurse is teaching a client who is postpartum about keeping the newborn safe. Which of the following statements should the nurse identify as an indication that the client understands the instructions? A. I will put bumper pads in the crib. B. I will warm my baby's formula in the microwave on a low setting. C. I will place my baby on his stomach to sleep. D. I will purchase a firm mattress for the crib.

D. I will purchase a firm mattress for the crib. A firm mattress that leaves no gaps between it and the crib rails helps prevent suffocation and entrapment. Bumper pads, pillows, stuffed toys, and blankets increase the risk of suffocation. The client should never use a microwave oven to heat refrigerated formula or breast milk. Heating in a microwave is inconsistent and can cause burns. Instead, the client should warm the formula in a pan of hot water and test the temperature with a few drops on the inner aspect of her wrist before feeding it to the newborn. The client should place the newborn on his back to sleep to reduce the risk of sudden infant death syndrome (SIDS).

A 41-week multigravida is receiving oxytocin (Pitocin) to augment labor. Contractions are firm and occurring every 5 minutes, with a 30- to 40-second duration. The fetal heart rate increases with each contraction and returns to baseline after the contraction. Which action should the nurse implement? A. Place a wedge under the client's left side. B. Determine cervical dilation and effacement. C. Administer 10 L of oxygen via face mask. D. Increase the rate of the oxytocin (Pitocin) infusion.

D. Increase the rate of the oxytocin (Pitocin) infusion. The goal of labor augmentation is to produce firm contractions that occur every 2 to 3 minutes, with a duration of 60 to 70 seconds, and without evidence of fetal stress. FHR accelerations are a normal response to contractions, so the oxytocin (Pitocin) infusion should be increased (D) per protocol to stimulate the frequency and intensity of contractions. (A and C) are indicated for fetal stress. A sterile vaginal examination (B) places the client at risk for infection and should be performed when the client exhibits signs of progressing labor, which is not indicated at this time.

A woman in her third trimester of pregnancy has been in active labor for the past 8 hours and has dilated 3 cm. The nurse's assessment findings and electronic fetal monitoring (EFM) are consistent with hypotonic dystocia, and the healthcare provider prescribes an oxytocin drip. Which data is most important for the nurse to monitor? A. Preparation for emergency cesarean birth. B. Client's hourly blood pressure. C. Checking the perineum for bulging. D. Intensity, interval, and length of contractions.

D. Intensity, interval, and length of contractions. Hypotonic dystocia is characterized by weak, inefficient contractions that do not result in significant cervical dilation or effacement. The oxytocin drip is used to strengthen contractions and promote cervical dilation. The nurse should closely monitor the frequency, duration, and strength of contractions.

A client who is in the second trimester of pregnancy tells the nurse that she wants to use herbal therapy. Which response is best for the nurse to provide? A. Herbs are a cornerstone of good health to include in your treatment. B. Touch is also therapeutic in relieving discomfort and anxiety. C. Your healthcare provider should direct treatment options for herbal therapy. D. It is important that you want to take part in your care.

D. It is important that you want to take part in your care. The emphasis of alternative and complementary therapies, such as herbal therapy, is that the client is viewed as a whole being, capable of decision-making and an integral part of the health care team, so (D) recognizes the client's request. (A and B) provide little support for the client's comment about herbal therapy. Although the healthcare provider should address the client's request, (C) dismisses the discussion and assumes the client is not an integral part of the healthcare team.

An expectant father tells the nurse he fears that his wife "is losing her mind". He states she is constantly rubbing her abdomen and talking to the baby, and that she actually reprimands the baby when it moves too much. What recommendation should the nurse make to this expectant father? A. Reassure him that these are normal reactions to pregnancy and suggest that he discuss his concerns with the childbirth education nurse. B. Help him to understand that his wife is experiencing normal symptoms of ambivalence about the pregnancy and no action is needed. C. Ask him to observe his wife's behavior carefully for the next few weeks and report any similar behavior to the nurse at the next prenatal visit. D. Let him know that these behaviors are part of normal maternal/fetal bonding which occur once the mother feels fetal movement.

D. Let him know that these behaviors are part of normal maternal/fetal bonding which occur once the mother feels fetal movement. These behaviors are positive signs of maternal/fetal bonding (D) and do not reflect ambivalence (B). No intervention is needed. Quickening, the first perception of fetal movement, occurs at 17 to 20 weeks gestation and begins a new phase of prenatal bonding during the second trimester. Although (A) is not wrong, it dismisses the father's concerns. (C) is not indicated. (ambivalence - having mixed feelings or contradictory ideas about something or someone.)

The nurse is caring for a female client, a primigravida with preeclampsia. Findings include +2 proteinuria, BP 172/112 mmHg, facial and hand swelling, complaints of blurry vision and a severe frontal headache. Which medication should the nurse anticipate for this client? A. Clonidine hydrochloride. B. Carbamazepine. C. Furosemide. D. Magnesium sulfate.

D. Magnesium sulfate. Patient is showing signs of preeclampsia.

A client who gave birth to a healthy 8 pound infant 3 hours ago is admitted to the postpartum unit. Which nursing plan is best in assisting this mother to bond with her newborn infant? A. Encourage the mother to provide total care for her infant. B. Provide privacy so the mother can develop a relationship with the infant. C. Encourage the father to provide most of the infant's care during hospitalization. D. Meet the mother's physical needs and demonstrate warmth toward the infant.

D. Meet the mother's physical needs and demonstrate warmth toward the infant. It is most important to meet the mother's requirement for attention to her needs so that she can begin infant care-taking (D). Nurse theorist Reva Rubin describes the initial postpartal period as the "taking-in phase," which is characterized by maternal reliance on others to satisfy the needs for comfort, rest, nourishment, and closeness to families and the newborn. (A) could impede development of maternal bonding. (B) is important but not the priority. (C) might encourage paternal bonding, but does not specifically encourage maternal bonding.

A client who gave birth to a healthy 8lb infant 3 hours ago is admitted to the postpartum unit. Which nursing plan is best in assisting this mother to bond with her newborn infant? A. Encourage the mother to provide total care for her infant. B. Provide privacy so the mother can develop a relationship with the infant. C. Encourage the father to provide most of the infant's care during hospitalization. D. Meet the mother's physical needs and demonstrate warmth toward the infant.

D. Meet the mother's physical needs and demonstrate warmth toward the infant. It is most important to meet the mother's requirement for attention to her needs so that she can begin infant care-taking.

The nurse observes that an antepartum client who is on bed rest for preterm labor is eating ice rather than the food on her breakfast tray. The client states that she has a craving for ice and then feels too full to eat anything else. Which is the best response by the nurse? A. Remove all ice from the client's room. B. Ask the client what foods she might consider eating. C. Remind the client that what she eats affects her baby. D. Notify the health care provider.

D. Notify the health care provider. The health care provider should be notified (D) when a client practices pica (craving for and consumption of nonfood substances). The practice of pica may displace more nutritious foods from the diet, and the client should be evaluated for anemia. (A) is overreacting and may be perceived as punishment by the patient. (B) allows the dietary department to customize the client's tray but fails to address physiologic problems associated with not consuming nutritious foods in pregnancy. (C) is judgmental and blocks further communication.

The nurse is assessing a newborn who was precipitously delivered at 38 weeks gestation. The newborn is tremulous, tachycardic, and hypertensive. Which assessment action is most important for the nurse to implement? A. Determine reactivity of neonatal reflexes. B. Perform gestational age assessment. C. Weight and measure the newborn. D. Obtain a drug screen for cocaine.

D. Obtain a drug screen for cocaine. - HESI

Assessment findings of a 4-hour-old newborn include: axillary temperature of 96.8°F (35.8°C), heart rate of 150 beats/minute with a soft murmur, irregular respiratory rate at 64 breaths/minute, jitteriness, hypotonic, and weak cry. Based on these findings, which action should the nurse implement? A. Swaddle the infant in a warm blanket. B. Place a pulse oximeter on the heel. C. Document the findings in the record. D. Obtain a heel stick blood glucose level.

D. Obtain a heel stick blood glucose level. - HESI

One hour following a normal vaginal delivery, a newborn infant boy's axillary temperature is 96° F, his lower lip is shaking and, when the nurse assesses for a Moro reflex, the boy's hands shake. Which intervention should the nurse implement first? A. Stimulate the infant to cry. B. Wrap the infant in warm blankets. C. Feed the infant formula. D. Obtain a serum glucose level.

D. Obtain a serum glucose level. This infant is demonstrating signs of hypoglycemia, possibly secondary to a low body temperature. The nurse should first determine the serum glucose level (D). (A) is an intervention for a lethargic infant. (B) should be done based on the temperature, but first the glucose level should be obtained. (C) helps raise the blood sugar, but first the nurse should determine the glucose level.

The nurse is caring for a client who is 10-weeks gestation and palpates the fundus at 2 fingerbreadths above the pubis symphysis. The clients reports nausea, vomiting, and scant dark brown vaginal discharge. Which action should the nurse take? A. Measure vital signs. B. Recommend bed rest. C. Collect urine sample for urinalysis. D. Obtain human chorionic gonadotropin levels.

D. Obtain human chorionic gonadotropin levels. - HESI (Molar pregnancy)

Which finding for a client in labor at 41-weeks gestation requires additional assessment by the nurse? A. Cervix dilated 2 cm and 50% effaced. B. Score of 8 on the biophysical profile. C. Fetal heart rate (FHR) of 116 beats per minute. D. One fetal movement noted in an hour.

D. One fetal movement noted in an hour. Healthcare providers generally suggest that women should feel at least 5 fetal movements within one hours. The other options are normal findings during labor and do not require additional assessment by the nurse.

The nurse in a birthing room is monitoring a client with dysfunctional labor for signs of fetal or maternal compromise. Which assessment finding should alert the nurse to a compromise? A. Maternal fatigue. B. Coordinated uterine contractions. C. Progressive changes in the cervix. D. Persistent nonreassuring fetal heart rate.

D. Persistent nonreassuring fetal heart rate. This finding may indicate fetal distress, which requires immediate intervention to prevent potential harm to the fetus. Maternal fatigue is a common finding during labor. Coordinated uterine contractions and progressive changes in the cervix are important indications of normal labor progression and do not necessarily indicate fetal or maternal compromise.

At 39-weeks gestation, a multigravida is having a nonstress test (NST). The fetal heart rate has remained non-reactive during 30 minutes of evaluation. Based on this finding, what action should the nurse implement? A. Initiate an intravenous (IV) infusion. B. Observe the FHR pattern for 30 more minutes. C. Schedule a biophysical profile. D. Place an acoustic stimulator on the abdomen.

D. Place an acoustic stimulator on the abdomen. A non-reactive fetal heart rate (FHR) should firstly result in the nurse suspecting fetus may be sleeping. An acoustic stimulator can be used to wake up the fetus and provoke fetal movement, which may improve the FHR pattern. The other options are not appropriate at this time.

The nurse is examining an infant for possible cryptorchidism. Which exam technique should be used? A. Place the infant in side-lying position to facilitate the exam. B. Hold the penis and retract the foreskin gently. C. Cleanse the penis with an antiseptic-soaked pad. D. Place the infant in a warm room and use a calm approach.

D. Place the infant in a warm room and use a calm approach. For possible cryptorchidism, the testes will need to be examined and a cold room keeps the testes pulled up to the body.

The nurse is assessing a 35-week primigravida with a breech presentation who is experiencing moderate uterine contractions every 3-5 minutes. During the examination the client tells the nurse, "I think my water just broke." Inspection of the perineal area reveals the umbilical cord protruding from the vagina. After activating the call bell system for assistance, what intervention should the nurse implement? A. Administer oxygen at 10 liters via face mask. B. Don gloves and push the cord back into the vagina. C. Wrap the umbilical cord with sterile gauze. D. Position the client into a knee-chest position.

D. Position the client into a knee-chest position. This situation describes a prolapsed umbilical cord, which is an obstetric emergency that requires immediate action to prevent fetal hypoxia and death. The nurse should immediately place the client in a knee-chest position to relieve pressure on the cord and improve fetal oxygenation.

A client at 9-weeks gestation tells the nurse that while she has "cut down," she still has at least one alcoholic drink every evening before bedtime. Which intervention should the nurse implement? A. Notify child protective services of the client's illicit drug use and probable child endangerment. B. Insist that the client stop all alcohol use and draw a blood alcohol level at each prenatal visit. C. Refer the client to an outpatient alcohol abuse program for disulfiram therapy. D. Praise the client for her actions and offer to discuss ways to decrease consumption even more.

D. Praise the client for her actions and offer to discuss ways to decrease consumption even more. The nurse should educate the client about the risks of alcohol use during pregnancy and encourage her to stop all alcohol use. The nurse should praise the client for cutting down on alcohol use and offer support and resources to help her stop completely. B and C is not appropriate as drawing a blood alcohol level at each prenatal visit is not a practical approach and disulfiram therapy is not recommended for pregnant women. A is not appropriate because the client's alcohol use does not necessarily constitute child endangerment and reporting the client to child protective services should not be the first action.

A multiparous client with active herpes lesions is admitted to the unit with spontaneous rupture of membranes. Which action should the nurse implement? A. Obtain blood cultures. B. Cover the lesion with a dressing. C. Administer penicillin. D. Prepare for a cesarean section.

D. Prepare for a cesarean section.

The nurse is preparing to administer phytonadione to a newborn. Which statement made by the parents indicates understanding why the nurse is administering this medication? A. Improve insufficient dietary intake. B. Stimulates the immune system. C. Help an immature liver. D. Prevent hemorrhagic disorders.

D. Prevent hemorrhagic disorders. - HESI

The nurse administers erythromycin ointment (0.5%) to the eyes of a newborn and the mother asks the nurse why this is performed. Which explanation is best for the nurse to provide about neonatal eye prophylaxis? A. Protects the newborn's eyes from possible infections acquired while hospitalized. B. Prevents cataracts in the newborn born to a woman who is susceptible to rubella. C. Minimizes the spread of microorganisms to the newborn from invasive procedures during labor. D. Prevents an infection called ophthalmia neonatorum from occurring after delivery in a newborn born to a woman with an untreated gonococcal infection.

D. Prevents an infection called ophthalmia neonatorum from occurring after delivery in a newborn born to a woman with an untreated gonococcal infection. Erythromycin ointment (0.5%) is one of the most commonly used antibiotics for neonatal eye prophylaxis and it is effective against many of the bacteria that can cause ophthalmia neonatorum. This infection can occur in newborns born to women with untreated gonococcal or chlamydial infections, and it can cause severe eye damage or even blindness. The ointment is typically applied to the lower conjunctival sac of each eye shortly after birth, usually within the first hour.

Which client should the nurse report to the healthcare provider as needing a prescription for Rh Immune Globulin (RhoGAM)? A. Woman whose blood group is AB Rh-positive. B. Newborn with rising serum bilirubin level. C. Newborn whose Coombs test is negative. D. Primigravida mother who is Rh-negative.

D. Primigravida mother who is Rh-negative. Rh immune globulin (RhoGAM) is given to Rh-negative women to prevent the development of antibodies to Rh-positive blood. If an Rh-negative woman is pregnant with an Rh-positive fetus, her immune system can recognize the fetus's Rh-positive blood as foreign and produce antibodies against it. These antibodies can then cross the placenta and attack the fetus's red blood cells.

Just after delivery, a new mother tells the nurse, "I was unsuccessful breastfeeding my first child, but I would like to try with this baby." Which intervention is best for the nurse to implement first? A. Assess the husband's feelings about his wife's decision to breastfeed their baby. B. Ask the client to describe why she was unsuccessful with breastfeeding her last child. C. Encourage the client to develop a positive attitude about breastfeeding to help ensure success. D. Provide assistance to the mother to begin breastfeeding as soon as possible after delivery.

D. Provide assistance to the mother to begin breastfeeding as soon as possible after delivery. Infants respond to breastfeeding best when feeding is initiated in the active phase soon after delivery. (A and B) might provide interesting data, but gathering this information is not as important as providing support and instructions to the new mother. While (C) is also true, this response by the nurse might seem judgmental to a new mother.

An off-duty nurse finds a woman in a supermarket parking lot delivering an infant while her husband is screaming for someone to help his wife. Which intervention has the highest priority? A. Use a thread to tie off the umbilical cord. B. Provide as much privacy as possible for the woman. C. Reassure the husband and try to keep him calm. D. Put the newborn to breast.

D. Put the newborn to breast. Putting the newborn to breast will help contract the uterus and prevent a postpartum hemorrhage--this intervention has the highest priority. It is not necessary to tie off the umbilical cord (A), the infant can be transported attached to the placenta. Providing privacy (B) is an important psychosocial need, but does not have the priority of (D). Although the husband is an important part of family-centered care, he is not the most important concern at this time (C).

A 30-year-old multiparous woman who has a 3-year-old boy and an newborn girl tells the nurse, "My son is so jealous of my daughter, I don't know how I'll ever manage both children when I get home." How should the nurse respond? A. Tell the older child that he is a big boy now and should love his new sister. B. Ask friends and relatives not to bring gifts to the older sibling because you do not want to spoil him. C. Let the older child stay with his grandparents for the first six weeks to allow him to adjust to the newborn. D. Regression in behaviors in the older child is a typical reaction so he needs attention at this time.

D. Regression in behaviors in the older child is a typical reaction so he needs attention at this time. Preschool-aged children frequently regress in habits or behaviors, such as toileting and sleep habits, as a method of seeking attention (D), so the parents should distribute their attention between the children and include the preschooler during infant care. (A) is a negative approach. Providing small gifts to give to the preschooler (B) is a strategy that can facilitate sibling acceptance of the new infant. Removing the preschooler from the home when the new infant arrives may enhance his negative behaviors (C) when he is separated from his mother.

A 6-month old child who had a cleft-lip repair has elbow restraints in place. What nursing intervention should the nurse plan to implement? A. Remove restraints q4h for 30 minutes and place gloves on the child's hands. B. Record observations of the restraints q2h and ensure that they are in place at all times. C. Obtain the HCP advice as to when the restraints should be removed. D. Remove restraints one at a time to provide ROM exercises.

D. Remove restraints one at a time to provide ROM exercises.

The nurse is calculating the estimated date of confinement (EDC) using Nägele's rule for a client whose last menstrual period started on December 1. Which date is most accurate? A. August 1. B. August 10. C. September 3. D. September 8.

D. September 8. Calculation of a client's EDC provides baseline data to monitor fetal gestation. Nägele's rule uses the formula: subtract 3 months and add 7 days to the first day of the last normal menstrual period, so December 1 minus 3 months + 7 days is September 8 (D). (A, B, and C) are incorrect use of this formula.

After each feeding, a 3-day-old newborn is spitting up large amounts of Enfamil Newborn Formula, a nonfat cow's milk formula. The pediatric healthcare provider changes the neonate's formula to Similac Soy Isomil Formula, a soy protein isolate based infant formula. What information should the nurse provide to the mother about the newly prescribed formula? A. The new formula is a coconut milk formula used with babies with impaired fat absorption. B. Enfamil Formula is a demineralized whey formula that is needed with diarrhea. C. The new formula is a casein protein source that is low in phenylalanine. D. Similac Soy Isomil Formula is a soy-based formula that contains sucrose.

D. Similac Soy Isomil Formula is a soy-based formula that contains sucrose. The nurse should explain that the newborn's feeding intolerance may be related to the lactose found in cow's milk formula and is being replaced with the soy-based formula that contains sucrose (D), which is well-tolerated in infants with milk allergies and lactose intolerance.* (A) describes Portagen Formula, a formula prescribed for malabsorption syndromes. (B) does not explain that cow's milk intolerance is the reason for the formula change. (C) describes Lofenalac Formula, a formula prescribed for phenylketonuria (PKU). *Important

A nurse is caring for a client who is in labor and is reporting intense pain during contractions. The client has no previous knowledge of nonpharmacologic comfort measures. Which of the following nursing interventions should the nurse implement? A. Self-hypnosis. B. Biofeedback. C. Acupuncture. D. Slow-paced breathing.

D. Slow-paced breathing. Slow-paced breathing is an easy technique for the client to learn quickly and practice immediately. It provides distraction, which can help reduce the perception of pain. The pattern is in 2-3-4/out 2-3-4/In 2-3/Out 2-3-4. Repeating this cycle slows the client's breathing to about half of its usual rate, which can help relax the client and improve oxygenation.

The nurse is reviewing the serum laboratory finding for a 5-day-old infant with congenital adrenal hyperplasia. Which laboratory results should be reported to the healthcare provider (HCP) immediately? A. Bilirubin of 1.5 mg/dl. B. Glucose of 80 mg/dl. C. Potassium of 4.5 mEq/L. D. Sodium of 119 mEq/L.

D. Sodium of 119 mEq/L. The normal range for serum sodium levels in a neonate is typically between 135 to 145 mEq/L. In infants with congenital adrenal hyperplasia, low sodium levels can be a sign of adrenal crisis, a potentially life-threatening complication that requires prompt treatment with intravenous fluids and medications. Potassium, bilirubin, and glucose levels are within normal limits for a neonate.

A 39-week gestational multigravida is admitted to labor and delivery with spontaneous rupture of membranes (SROM) and contractions occurring every 2 to 3 minutes. A vaginal exam indicates that the cervix is dilated 6 cm, 50% effaced, and the fetus is at a -2 station. During the last 45 minutes the fetal heart rate (FHR) has ranged between 170 and 180 beats/minute. Which action should the nurse implement? A. Straight catheterize the client. B. Send amniotic fluid for analysis. C. Obtain a blood specimen for hemoglobin. D. Take an oral maternal temperature.

D. Take an oral maternal temperature. Maternal temperature changes are associated with prolonged ruptured membranes for over 24 hours. Routine monitoring (usually every 2 to 4 hours) is indicated. (Elevated FHR indicates fever.)

A woman who had a miscarriage 6 months ago becomes pregnant. Which instruction is most important for the nurse to provide this client? A. Elevate lower legs while resting. B. Increase caloric intake by 200 to 300 calories per day. C. Increase water intake to 8 full glasses per day. D. Take prescribed multivitamin and mineral supplements.

D. Take prescribed multivitamin and mineral supplements. A client who has had a spontaneous abortion or still birth in the last 1½ years should take multivitamin and mineral supplements (D) and maintain a balanced diet because the previous pregnancy may have left her nutritionally depleted. (A, B, and C) are sound instructions to provide any pregnant client, but do not have the priority of (D) for this particular client who had a miscarriage 6 months ago.

A breastfeeding infant, screened for congenital hypothyroidism, is found to have low levels of thyroxine (t4) and high levels of thyroid stimulating hormone (TSH). What is the best explanation for this finding? A. The thyroxine level is low because the TSH level is high. B. High thyroxine levels normally occur in breastfeeding infants. C. The thyroid gland does not produce normal levels of thyroxine for several weeks after birth. D. The TSH is high because of the low production of t4 by the thyroid.

D. The TSH is high because of the low production of t4 by the thyroid. The thyroid gland does not produce enough thyroxine (t4), which stimulates the production of thyroid-stimulating hormone (TSH) by the pituitary gland.

The nurse is caring for a 2-day-old neonate who has not passed meconium and has a swollen abdomen. The healthcare provider reviews the flat plate x-ray of the abdomen and makes a tentative diagnosis of Hirschsprung's disease. Which pathophysiological process is consistent with this neonate's clinical picture? A. Partial imperforated anal opening limits the passage of stools. B. Peristalsis moving against the intestinal obstruction causing hyperactivity of the bowel. C. Pyloric obstruction causes reflux and projective hematemesis. D. The congenital absence of parasympathetic ganglion cells to large intestine produces no peristalsis.

D. The congenital absence of parasympathetic ganglion cells to large intestine produces no peristalsis.

While breastfeeding, a new mother strokes the top of her baby's head and asks the nurse about the baby's swollen scalp. The nurse responds that the swelling is caput succedaneum. Which additional information should the nurse provide this new mother? A. The infant should be positioned to reduce the swelling. B. The swelling is a subperiosteal collection of blood. C. The pediatrician will aspirate the blood if it gets larger. D. The scalp edema will subside in a few days after birth.

D. The scalp edema will subside in a few days after birth. Caput succedaneum is edema of the fetal scalp that crosses over the suture lines and is caused by pressure on the fetal head against the cervix during labor; it subside in a few days after birth without treatment (D). (B) describes a cephalohematoma, a subperiosteal collection of blood that does not cross the suture lines and is a common benign birth injury. (A and C) are not necessary for caput or cephalohematoma.

A nurse is performing a physical assessment of a male newborn. Which of the following findings should the nurse report to the provider? A. Superficial cracking and peeling are evident on the skin of the hands and feet. B. The Palmar grasp occurs spontaneously when newborn is sucking. C. The bulge of the testes is palpable in the inguinal canal. D. There is decreased abdominal movement with breathing.

D. There is decreased abdominal movement with breathing. The nurse should report this finding to the provider. Decreased abdominal movement with breathing is a deviation from an expected finding and could indicate phrenic nerve palsy or a congenital diaphragmatic hernia. The nurse should expect the newborn to have diaphragmatic breathing with synchronous abdominal and chest movements.

A new mother is afraid to touch her baby's head for fear of hurting the "large soft spot." Which explanation should the nurse give to this anxious client? A. Some care is required when touching the large soft area on top of your baby's head until the bones fuse together. B. That's just an 'old wives' tale' so don't worry, you can't harm your baby's head by touching the soft spot. C. The soft spot will disappear within 6 weeks and is very unlikely to cause any problems for your baby. D. There's a strong, tough membrane there to protect the baby so you need not be afraid to wash or comb his/her hair.

D. There's a strong, tough membrane there to protect the baby so you need not be afraid to wash or comb his/her hair. (D) provides correct information and attempts to alleviate anxiety related to knowledge deficit. The anterior fontanel or "large soft spot" has a strong epidermal membrane present, which can be touched (A). (B) dismisses the client's concerns. The anterior fontanel normally closes at 12 to 18 months of age, not 6 weeks (C). The posterior fontanel closes at 8 to 12 weeks of age.

A couple has been trying to conceive for nine months without success. Which information obtained from the clients is most likely to have an impact on the couple's ability to conceive a child? A. Exercise regimen of both partners includes running four miles each morning. B. History of having sexual intercourse 2 to 3 times per week. C. The woman's menstrual period occurs every 35 days. D. They use lubricants with each sexual encounter to decrease friction.

D. They use lubricants with each sexual encounter to decrease friction. The use of lubricants (D) has the potential to affect fertility because some lubricants interfere with sperm motility. While excessive heat can affect sperm production, bicycling, rather than running (A) is more likely to concentrate heat in the groin area. While having intercourse too frequently has been implicated as a cause for decreased numbers of sperm, 2 to 3 times per week (B) is not considered excessive. (C) should not affect fertility.

Following a minor motor vehicle collision, a client at 36-weeks gestation is brought to the emergency center. She is lying supine on a backboard, is awake, and denies any complaints. Her blood pressure is 80/50 mmHg and her heart rate is 130 beats/minute. Which action should the nurse implement first? A. Palpate the abdomen for contractions. B. Infuse 1,000 mL normal saline using a large bore IV. C. Obtain a blood sample for complete blood count. D. Tilt the backboard sideways to displace the uterus laterally.

D. Tilt the backboard sideways to displace the uterus laterally.

A client who is in active labor is receiving magnesium sulfate and begins to experience slurred speech and decreased reflexes. Which action should the nurse implement first? A. Obtain a serum magnesium level. B. Measure the client's hourly urinary output. C. Provide an emesis basin for vomiting. D. Turn off the magnesium sulfate infusion.

D. Turn off the magnesium sulfate infusion. The client is showing signs of magnesium toxicity, which can be serious and even life-threatening. The first action the nurse should take is to discontinue the magnesium sulfate infusion to prevent further accumulation of the medication in the client's system.

A 35-year-old primigravida client with severe preeclampsia is receiving magnesium sulfate via continuous IV infusion. Which assessment data indicates to the nurse that the client is experiencing magnesium sulfate toxicity? A. Deep tendon reflexes 2+. B. Blood pressure 140/90. C. Respiratory rate 18/minute. D. Urine output 90 mL/4 hours.

D. Urine output 90 mL/4 hours. Urine outputs of less than 100 ml/4 hours (D), absent DTRs, and a respiratory rate of less than 12 breaths/minute are cardinal signs of magnesium sulfate toxicity. (A, B, and C) do not indicate a magnesium sulfate toxicity.

A nurse is caring for a client in the third trimester of pregnancy who reports difficulty sleeping. Which of the following instructions should the nurse produce? A. Eat a high-fat snack before bed. B. Exercise in the evening before bed. C. Sleep in the supine position. D. Use additional pillows to support extremities and abdomen.

D. Use additional pillows to support extremities and abdomen. Finding a comfortable position for sleeping during the last 3 months of pregnancy can be difficult due to fetal growth. Using additional pillows promotes a more comfortable sleeping position.

A client who had her first baby three months ago and is breastfeeding her infant tells the nurse that she is currently using the same diaphragm that she used before becoming pregnant. Which information should the nurse provide this client? A. After ceasing breastfeeding the diaphragm should be resized B. Avoid intercourse during ovulation until the size of the diaphragm has been evaluated. C. If no more than 20 pounds was gained during pregnancy, the diaphragm is safe to use. D. Use an alternate form of contraception until a new diaphragm is obtained.

D. Use an alternate form of contraception until a new diaphragm is obtained. The client should be informed that the diaphragm used before pregnancy may no longer fit properly due to changes in the body's size and shape. She should be advised to use an alternate form of contraception until a health care provider can assess her body and properly fit a new diaphragm.

The nurse is monitoring a client in labor. The nurse suspects umbilical cord compression if which is noted on the external monitor tracing during a contraction? A. Variability. B. Accelerations. C. Early decelerations. D. Variable decelerations.

D. Variable decelerations. Variable decelerations are a common type of fetal heart rate deceleration that occur during contractions in labor. They are called "variable" because they do not have a consistent shape or timing, and can vary in their depth and duration. Variable decelerations are caused by compression or occlusion of the umbilical cord, which reduces blood flow and oxygen to the fetus.

A nurse is teaching a client who is at 12 weeks gestation and has HIV. Which of the following statements should the nurse include in the teaching? A. Breastfeed newborn to provide passive immunity. B. Abstain from sexual intercourse throughout the pregnancy. C. You will be in isolation after delivery. D. You should continue to take zidovudine throughout your pregnancy.

D. You should continue to take zidovudine throughout the pregnancy. The nurse should inform the client that taking prescription antiviral medication every day decreases the risk of transmission of HIV to her newborn. (Zidovudine can be started after 1st trimester - 12 weeks)

In developing a teaching plan for expectant parents, the nurse plans to include information about when the parents can expect the infant's fontanels to close. The nurse bases the explanation on knowledge that for the normal newborn, the: A. anterior fontanel closes at 2-4 months and posterior by the end of the first week. B. anterior fontanel closes at 5-7 months and posterior by end of second week. C. anterior fontanel closes at 8-11 months and posterior by end of first month. D. anterior fontanel closes at 12-18 months and posterior by end of second month.

D. anterior fontanel closes at 12-18 months and posterior by end of second month. In the normal infant the anterior fontanel closes at 12 to 18 months of age and the posterior fontanel by the end of the second month (D).

A nurse is caring for a client who is refusing a blood transfusion for religious reasons. The client's partner wants the client to have the blood transfusion. Which of the following actions should the nurse take? A. Request prescription for oxytocin. B. Administer oxygen at 2 L/min via nasal cannula. C. Prepare for the insertion of intrauterine balloon. D. Reposition the client from side to side.

D?

The nurse is caring for a client who delivered 6 hours ago. assessment findings reveal a boggy uterus that is displaced above and to the right of the umbilicus. which action should the nurse take?

Encourage voiding. - HESI

Breast engorgement

Form fitting bra - HESI

Which action should the nurse take if an infant who was born yesterday weighing 7.5 pounds (3,402 grams), weighs 7 pounds (3,175) today?

Inform and assure the mother that this is a normal weight loss. - HESI

Yellow tint... A. Prepare infant for phototherapy B. Measure bilirubin levels using transcutaneous bilirubinometer. C. D.

Measure bilirubin levels using transcutaneous bilirubinometer. - HESI

During newborn admission assessment, the nurse palpates the newborn's scrotum and does not feel the testicles. Which assessment technique should the nurse perform next to verify absence of testes? A. Palpate... B. Perform transillumination of scrotal sac to visualize shadow of testes. C. D.

Perform transillumination of scrotal sac to visualize shadow of testes. - HESI

A father watching the admission of his newborn to the nursery notices that eye ointment is placed in the infants eyes. he asks the nurse what the purpose of the ointment is. the nurse would be correct in stating that the purpose for using the ointment is to

Prevent eye infection. - HESI

Which physical assessment data should the nurse consider a normal finding for a primigravida client who is 12 hours postpartum?

Pulse rate of 56 beats/minute. - HESI (Postpregnancy heartrate)

During a routine prenatal health assessment for a client in her third trimester, the client reports that she had fluid leakage on her way to the appointment. which technique should the nurse implement to calculate the leakage?

Test the fluid with a nitrazine strip. - HESI

A gravida 3 para 3 who is Rh-negative delivers a full-term infant at home with the assistance of a nurse-midwife. Two days later, the client calls the clinic to ask if it is necessary to see the healthcare provider since the infant is healthy, and she is not having any complications. The woman's history indicates that both previously born infants were Rh-negative. Which response should the nurse provide? A. It is likely that the husband is Rh negative, and if so Rho (D) immune globulin injection is not needed. B. Rho (D) immune globulin is not indicated since both previous babies were Rh-negative. C. The newborn's blood type should be tested to determine the need for RhoGAM. D. Rho (D) immune globulin injections must be administered within 24 hours after delivery.

The newborn's blood type should be tested to determine the need for RhoGAM. (Rhogam is given to Rh-negative mothers during pregnancy to prevent complications that can occur if their fetus is Rh-positive.)

A client who is receiving oxytocin to augment early labor begins to experience tachysystole or tetanic contractions with variable fetal heart decelerations. which action should the nurse implement?

Turn off the oxytocin. - HESI


Set pelajaran terkait

Ch. 23 Disruptive Behavior Disorders M.C.

View Set

GMAT - Divisibility, Inequalities, Min Max Stats 5

View Set

Environmental Toxicology Final Exam Review

View Set

Cardiovascular Drugs: Drugs 31-60 Drug Trade Name, Drug Generic Name and Drug Category

View Set

Pharmacology Chapter 9: Antibiotics

View Set